Download as pdf or txt
Download as pdf or txt
You are on page 1of 123

CHARTERED ACCOUNTANCY PROFESSIONAL II

(CAP-II)

Suggested Answer
December 2018

The Institute of Chartered Accountants of Nepal


Suggested Answer- December 2018

Paper 1: Advanced Accounting

The Institute of Chartered Accountants of Nepal 2


Suggested Answer- December 2018

Advance Account
Suggested
Roll No……………. Maximum Marks - 100
Total No. of Questions - 6 Total No. of Printed Pages - 14
Time Allowed - 3 Hours
Marks
Attempt all questions. Working notes should form part of the answer.
1. Ram, the owner of Ram Ltd. and Sita, the owner of Sita Ltd., got
married. So, they agreed to amalgamate their business. The scheme
envisaged a share capital, equal to the combined capital of Ram Ltd. &
Sita Ltd. for the purpose of acquiring the assets, liabilities and
undertakings of the two companies in exchange for share in Ram & Sita
Ltd.
The Summarized Balance Sheets of Ram Ltd. & Sita Ltd. as on 32nd
Ashadh, 2075 (the date of amalgamation) are given below:
Summarized Balance Sheets as on 32nd Ashadh, 2075
Equity & Liabilities Ram Ltd. Sita Ltd. Assets Ram Ltd. Sita Ltd.
(Rs.) (Rs.) (Rs.) (Rs.)
Shareholders Fund: Non-current Assets:
a. Share Capital 6,00,000 8,40,000 Fixed Assets,
b. Reserves 10,20,000 6,00,000 excluding goodwill 7,20,000 10,80,000

Current Assets:
Current Liabilities: Inventories 3,60,000 6,60,000
Bank Overdraft - 5,40,000 Trade Receivables 4,80,000 7,80,000
Trade Payables 2,40,000 5,40,000 Cash at Bank 3,00,000 -
18,60,000 25,20,000 18,60,000 25,20,000
The consideration was to be based on the net assets of the companies as
shown in the Balance Sheet above, but subject to an additional payment
to Ram Ltd. for its goodwill to be calculated as its weighted average net
profits for the three years ended 32nd Ashadh, 2075.
The profit had been
2072/73 Rs. 3,00,000; 2073/74 Rs. 5,25,000; 2074/75 Rs. 6,30,000.
The shares of Ram & Sita Ltd. were to be issued to Ram Ltd. & Sita Ltd.
at a premium and in proportion to the agreed net assets value of these
companies.
In order to raise working capital, Ram & Sita Ltd. proceeded to issue
72,000 shares of Rs. 10 each at the same rate of premium as issued for
discharging the purchase consideration to Ram Ltd. & Sita Ltd.
You are required to prepare: 20
a) Calculate the number of shares issued to Ram Ltd. & Sita Ltd.
b) Prepare required journal entries in the books of Ram & Sita Ltd.
c) Prepare the Balance Sheet of Ram & Sita Ltd. after recording the
necessary journal entries.

The Institute of Chartered Accountants of Nepal 3


Suggested Answer- December 2018

Solution:1
a) Calculation of number of shares issued to Ram Ltd. & Sita Ltd.
Amount of share capital as per Balance Sheet Rs.
Ram Ltd. 6,00,000
Sita Ltd. 8,40,000
14,40,000
Share of Ram Ltd. = 14,40,000*[21,60,000/(21,60,000+14,40,000) ]
= Rs. 8,64,000 or 86,400 Shares
Securities Premium= 21,60,000 - 8,64,000 = Rs. 12,96,000
Premium per share = 12,96,000/86,400 = Rs. 15 per share

Share of Sita Ltd. = 14,40,000*[14,40,000/(21,60,000+14,40,000) ]


= Rs. 5,76,000 or 57,600 Shares
Securities Premium= 14,40,000-5,76,000= Rs. 8,64,000
Premium per share = 8,64,000/57600= Rs. 15 per share

b) Journal Entries in the books of Ram & Sita Ltd.


i) Business Purchase Account Dr. 36,00,000
To, Liquidator of Ram Ltd. 21,60,000
To, Liquidator of Sita Ltd. 14,40,000
(Being the amount of purchase consideration payable to liquidator of Ram
Ltd. & Sita Ltd. for assets taken over)

ii) Goodwill Account Dr. 5,40,000


Fixed Assets Account Dr. 7,20,000
Inventory Account Dr. 3,60,000
Trade Receivables Account Dr. 4,80,000
Cash at Bank Account Dr. 3,00,000
To, Trade Payables Account 2,40,000
To, Business Purchase Account 21,60,000
(Being assets & liabilities of Ram Ltd. taken over)

iii) Fixed Assets Account Dr. 10,80,000


Inventory Account Dr. 6,60,000
Trade Receivables Account Dr. 7,80,000
To, Trade Payables Account 5,40,000
To, Bank Overdraft Account 5,40,000
To, Business Purchase Account 14,40,000
(Being assets & liabilities of Sita Ltd. taken over)

iv) Liquidator of Ram Ltd. Account Dr. 21,60,000


To, Equity Share Capital Account 8,64,000
To, Securities Premium Account 12,96,000
(Being the allotment of shares as per agreement for discharge of purchase
consideration)

v) Liquidator of Sita Ltd. Account Dr. 14,40,000


To, Equity Share Capital Account 5,76,000
To, Securities Premium Account 8,64,000

The Institute of Chartered Accountants of Nepal 4


Suggested Answer- December 2018

(Being the allotment of shares as per agreement for discharge of purchase


consideration)

vi) Bank Account Dr. 18,00,000


To, Equity Share Capital Account 7,20,000
To, Securities Premium Account 10,80,000
(Being Equity Share Capital raised for working capital )

c) Balance Sheet of Ram & Sita Ltd. as on 32-03-2075 after amalgamation


Equity & Liabilities Amount Assets Amount (Rs.)
(Rs.)
Shareholders Fund Non-current Assets
a. Share Capital 21,60,000 Fixed Assets 18,00,000
b. Securities Premium 32,40,000 Goodwill 5,40,000

Current Assets
Current Liabilities Inventories 10,20,000
Bank Overdraft 5,40,000 Trade Receivables 12,60,000
Trade Payables 7,80,000 Cash at Bank 21,00,000
67,20,000 67,20,000

Working Notes:
1. Calculation of Goodwill of Ram Ltd.
Year Profit Weight Weighted Amount
2072/73 3,00,000 1 3,00,000
2073/74 5,25,000 2 10,50,000
2074/75 6,30,000 3 18,90,000
Total 6 32,40,000
Weighted Average Amount = 32,40,000/6 = 5,40,000
Goodwill = Rs. 5,40,000

2. Calculation of Net Assets


Ram Ltd. Sita Ltd.
Assets
Goodwill 5,40,000 -
Fixed Assets 7,20,000 10,80,000
Inventory 3,60,000 6,60,000
Trade Receivables 4,80,000 7,80,000
Cash at Bank 3,00,000 -
Less: Liabilities
Bank Overdraft - 5,40,000
Trade Payables 2,40,000 5,40,000
Net Assets or Purchase Consideration 21,60,000 14,40,000

2.
a) From the following particulars, you are required to calculate the
amount of claim for Godawari Ltd., whose business premises was
partly destroyed by fire: 10
Sum insured (from 31st Chaitra, 2073) Rs. 4,00,000

The Institute of Chartered Accountants of Nepal 5


Suggested Answer- December 2018

Period of indemnity 12 months


Date of damage 1st
Baishakh, 2074
Date on which disruption of business ceased 31st Magh,
2074
The subject matter of the policy was gross profit but only net profit
and insured standing charges are included.
The books of account revealed:
i) The gross profit for the financial year 2073 was Rs. 3,60,000.
ii) The actual turnover for financial year 2073 was Rs. 12,00,000
which was also the turnover in this case.
iii) The turnover for the period from 1 st Baishakh to 31st Magh, in
the year preceding the loss, was Rs. 10,00,000.
During dislocation of the position, it was learnt that in Falgun-
Chaitra 2074, there has been an upward trend in business
(compared to the figure of the previous years) and it was stated that
had the loss not occurred, the trading results for 2074 would have
been better than those of the previous years.
The Insurance company official who was appointed to assess the loss
accepted this view and adjustments were made to the pre-damaged
figures to bring them up to the estimated amounts which would have
resulted in 2074.
The pre-damaged figures together with agreed adjustments were:
Period Pre damaged Adjustment to Adjusted Standard
figure be added Turnover
Rs. Rs. Rs.
Baishak 90,000 10,000 1,00,000
Jestha to Magh 9,10,000 50,000 9,60,000
Falgun to Chaitra 2,00,000 10,000 2,10,000
Total 12,00,000 70,000 12,70,000
Gross Profit 3,60,000 46,400 4,06,400
Rate of Gross Profit 30% (actual for 2073), 32% (adjusted for 2074).
Increased cost of working amounted to Rs. 1,80,000.
There was a clause in the policy relating to savings in insured
standard charges during the indemnity period and this amounted to
Rs. 28,000.
Standing Charges not covered by insurance amounted to Rs. 20,000
p.a. The annual turnover for Baishakh was nil and for the period
Jestha to Magh 2074 Rs. 8,00,000.
b) A company issued 1,50,000 shares of Rs. 10 each at a premium of
Rs. 10. The entire issue was underwritten as follows:
X-90,000 shares (firm underwriting 12,000 shares)
Y- 37,500 shares (firm underwriting 4,500 shares)
Z- 22,500 shares (firm underwriting 15,000 shares)

The Institute of Chartered Accountants of Nepal 6


Suggested Answer- December 2018

Total subscriptions received by the company (excluding firm


underwriting and marked applications) were 22,500 shares.
The marked applications (excluding firm underwriting) were as
follows:
X-15,000 shares
Y- 30,000 shares
Z- 7,500 shares
Commission payable to underwriters is at 5% of the issue price. The
underwriting contract provides that credit for unmarked applications
be given to the underwriters in proportion to the shares underwritten
and benefit of firm underwriting is to be given to individual
underwriters.
Required: 10
i) Determine the liability of each underwriter (number of shares)
ii) Compute the amounts payable or due from underwrites; and
iii) Pass Journal Entries in the books of the company relating to
underwriting.

Solution-2(a)
1) Short Sales
Period Adjusted Standard Turnover Actual Turnover Shortage
Rs. Rs. Rs.
Baishak 1,00,000 - 1,00,000
Jestha to Magh 9,60,000 8,00,000 1,60,000
10,60,000 8,00,000 2,60,000
2) Gross profit ratio for the purpose of insurance claim on loss of profit
Gross profit - Insured Standing Charges - Uninsured standing charges = Net profit Or

Net profit +Insured Standing Charges = Gross profit - Uninsured standing charges
= 4,06,400 – 20,000 = 3,86,400
=3,86,400/12,70,000×100%= 30.425%
3) Amount allowable in respect of additional expenses is least of the following
(i) Actual expenses 180,000
(ii) Gross profit on sales during 10 months period =8,00,000×30.425%=243,400
(iii)

Least i.e. 1,71,142 is admissible

The Institute of Chartered Accountants of Nepal 7


Suggested Answer- December 2018

4) Amount of Claim
Particulars Rs.
Gross profit on short sales = 2,60,000× 30.425% 79,105
Add: Amount allowable in respect of additional 1,71,142
expense
2,50,247
Less: Savings in Insured Standing Charges (28,000)
2,22,247
On the amount of final claim, the average clause will not apply since the amount of the
policy Rs.4,00,000 is higher than gross profit on annual adjusted turnover Rs.3,86,400.
Therefore the insurance claim will be Rs.2,22,247

Solution-2(b)
(i) Computation of total liability of underwriters in shares
(in shares)
X Y Z Total
Gross liability 90,000 37,500 22,500 1,50,000
Less: Marked applications
(excluding firm underwriting ) (15,000) (30,000) (7,500) (52,500)
75,000 7,500 15,000 97,5000
Less: Unmarked applications
in the ratio of gross liabilities
of 12:5:3 (excluding firm
underwriting ) (13,500) (5,625) (3,375) (22,500)
Less: Firm underwriting 61,500 1,875 11,625 75,000
(12,000) (4,500) (15,000) (31,500)
Less: Surplus of Y and Z 49,500 (2,625) (3,375) 43,500
adjusted in X's balance
(2,625+3,375) (6,000) 2,625 3,375 00
Net liability 43,500 - - 43,500
Add: Firm underwriting 12,000 4,500 15,000 31,500
Total liability 55,500 4,500 15,000 75,000

(ii) Calculation of amount payable to or due from underwriters


X Y Z Total
Total Liability in shares 55,500 4,500 15,000 75,000

Amount receivable @ Rs 20
from Underwriter (in Rs) 11,10,000 90,000 3,00,000 15,00,000
Less: Underwriting
Commission Payable @ 5% of (90,000) (37,500) (22,500) (1,50,000)
Rs 20 (in Rs)
Net amount receivable (in Rs)
10,20,000 52,500 2,77,500 13,50,000

The Institute of Chartered Accountants of Nepal 8


Suggested Answer- December 2018

(iii) Journal Entries in the books of the company (relating to underwriting)


Rs Rs
1 X Dr. 11,10,000
Y Dr. 90,000
Z Dr. 3,00,000
To Share capital A/c 7,50,000
To Securities Premium A/c 7,50,000
(Being allotment of shares to
underwriters)

2 Underwriting commission A/c Dr. 1,50,000


To X 90,000
To Y 37,500
To Z 22,500
(Being amount of underwriting
commission payable)

3 Bank A/c Dr. 13,50,000


To X 10,20,000
To Y 52,500
To Z 2,77,500
(Being net amount received by
underwriting for shares allotted less
underwriting commission)

3.
a) The Balance Sheets of X Co. Ltd. and Y Co. Ltd. as on 31st Chaitra,
2074 are as follows:
Balance sheet of X Co. Ltd.
Liabilities Rs. Assets Rs.
Share Capital: Fixed Assets:
Authorised Capital: Goodwill 80,000
10,000 shares of Other 8,00,000 8,80,000
Rs. 100 each 10,00,000 Current Assets,
Loans and Advances 9,00,000
Issued Capital:
10,000 shares of Rs.100 each
fully paid 10,00,000
Reserves and Surplus:
Capital Reserve 2,00,000
General Reserve 70,000 2,70,000
Unsecured Loans 2,00,000
Current Liabilities and
Provisions :
Sundry Creditors 3,10,000
17,80,000 17,80,000

The Institute of Chartered Accountants of Nepal 9


Suggested Answer- December 2018

Balance sheet of Y Co. Ltd.


Liabilities Rs. Assets Rs.
Share Capital: Fixed Assets: 16,00,000
Authorised Capital: Current Assets, Loans
2,00,000 share of Rs.10 and Advances:
each 20,00,000 Bank 2,00,000
Other 6,60,000 8,60,000
Issued Capital:
80,000 shares of Rs.10
each fully paid 8,00,000
Reserves and Surplus:
General Reserve 8,00,000
Secured Loans 5,00,000
Current Liabilities and
Provisions:
Sundry Creditors 3,60,000
24,60,000 24,60,000
It was proposed that X Co. Ltd. should be taken over by Y Co. Ltd.
The following arrangement was accepted by both the companies:
a. Goodwill of X Co. Ltd. is considered valueless.
b. Arrears of depreciation in X Co. Ltd. amounted to Rs. 40,000.
c. The holder of every 2 shares in X Co. Ltd. was to receive:
(i) as fully paid at per 10 shares in Y Co. Ltd. and
(ii) so much cash as in necessary to adjust the rights of
shareholders of both the companies in accordance with the
intrinsic value of the shares as per their balance sheets
subjects to necessary adjustments with regards to goodwill
and depreciation in X Co. Ltd.‘s Balance Sheet.
You are required to: 10
(a) Determine the composition of purchase consideration; and
(b) Show the Balance Sheet after absorption.
b) How will you disclose following items while preparing Cash Flow
Statement of Thapathali Ltd. as per Nepal accounting standard for
the year ended 32nd Ashadh 2075? 5
(i) 10% Debentures issued As on 1.4.2074 Rs. 110,000
As on 32.3.2075 Rs. 77,000
(ii) Debentures were redeemed at 5% premium at the end of the
year. Premium was charged to the profit & Loss account for
the year.
(iii) Unpaid interest on debentures As on 1.4.2074 Rs. 275
As on 32.3.2075 Rs. 1,175
(iv) Debtors of Rs. 36,000 were written off against the provision
for doubtful debts account during the year.
(v) Investment in 10% Bonds As on 1.4.2074 Rs. 350,000
As on 32.3.2075 Rs. 350,000
(vi) Accrued interest on investments As on 32.3.2075 Rs. 10,500

The Institute of Chartered Accountants of Nepal 10


Suggested Answer- December 2018

Solution-3(a)

a)
(a)Computation and Composition of Purchase Consideration
(i) Valuation of shares of X Co. Ltd. Rs.
Share Capital 10,0,000
Capital Reserve 2,00,000
General Reserve 70,000
12,70,000
Less: Goodwill,being valueless 80,000
Arrear of Depreciation 40,000 1,20,000
Value of Net Assets 11,50,000
No. of Shares 10,000
Intrinsic value per share Rs.115
(ii) Valuation of Shares of Y Co. Ltd.
Share Capital 8,00,000
General Reserve 8,00,000
16,00,000
No. of Shares 80,000
Value per share Rs. 20
On the basis of intrinsic values, every holder of two shares in X Co. Ltd. will
receive 10 shares in Y Co. Ltd. plus cash for the balance. The intrinsic value of the
two shares in X Co. Ltd. is Rs. 230 and that of 10 shares in Y Co. Ltd. is Rs.200 .
Therefore, for each lot of two shares in X Co. Ltd. , a shareholder will receive
Rs.30 in cash (Rs.230-200).

Y Co . Ltd. will therefore satisfy the purchase considerations as follows:


50,000 Shares of Rs.10 each issued at Rs. 20 each
10,00,000
Cash
1,50,000

11,50,000

(b)
Y Co. Ltd.(after absorption)
BALANCE SHEET
as on 31st Chaitra, 2074
Liabilities Rs. Liabilities Rs.
Share Capital: Fixed Assets 16,00,000
Authorised Addition on acquisition
2,00,000 shares of 7,60,000 23,60,000
Rs.10 each 20,00,000
Investment -
Issued and Subscribed Current Assets,Loans 15,60,000
1,30,000 Shares of &Advances
Rs.10 each fully paid Cash at Bank 50,000
(Issued for

The Institute of Chartered Accountants of Nepal 11


Suggested Answer- December 2018

consideration other
than cash:50,000
Shares of Rs.10 each
fully paid ) 13,00,000
Reserve and Surplus :
Share Premium 5,00,000
General Reserve 8,00,000
Secured Loans 5,00,000
Unsecured Loans 2,00,000
Current Liabilities &
Provisions :
Sundry Creditors 6,70,000
39,70,000 39,70,000

Solution-3(b)

Cash flow statement of Thapathali Ltd. for the year ended 32nd Ashad 2075

A Cash flow from operating activities


Net profit as per Profit & Loss a/c --------
Add: Premium on redemption of debentures 1,650
Add: interest on 10% debentures 11,000
Less: interest on 10% investments (35,000)
B Cash flow from investing activities
Interest on investments (35,000 – 10,500) 24,500
C Cash flow from financing activities
Interest on debentures paid [11,000 – (1,175-275)] (10,100)
Redemption of debentures [(110,000-77000) at 5% premium] (34,650)
Writing of debtors against provision for bad debt:
It will not be included in cash flow statement

4.
a) You are the Financial Accountant for Kathmandu Ltd., a company
that manufactures household furniture. Kathmandu Ltd. has
experienced both a reduction in sales revenue and cash flow during
the last financial period. You are provided with the following
information regarding Kathmandu Ltd. for the years 2072/73 and
2073/74:
Statement of Profit or Loss 2072/73 2073/74
(Rs.'000) (Rs.'000)
Revenue 500 700
Cost of sales (300) (350)
Gross profit 200 350
Operating expenses (75) (140)
Operating profit 125 210
Interest on debentures (30) (50)
Profit before tax 95 160
Tax (12) (20)

The Institute of Chartered Accountants of Nepal 12


Suggested Answer- December 2018

Profit after tax 83 140


Statement of Financial Position
Ashadh End 2073 Ashadh End 2074
Non-Current Assets: (Rs.'000) (Rs.'000)
Property, Plant and Equipment 1,160 1,200
Intangible assets 650 400
1,810 1,600
Current Assets:
Inventory 41 39
Trade Receivables 69 67
Bank - 150
110 256
Total Assets 1,920 1,856
Equity & Liabilities:
Issued Share Capital 800 800
Retained Earnings 612 529
1,412 1,329
Non-Current Liabilities:
10% Debenture 300 500
Current Liabilities:
Bank Overdraft 171 -
Trade Payables 37 27
508 527
1,920 1,856

Required: 10
Calculate the following ratios for both years:
i) Operating profit margin
ii) Current ratio
iii) Acid test ratio
iv) Inventory days
v) Receivable days
vi) Payable days
vii) Return on capital employed
viii) Gearing Ratio
b) Following is the information related to loan loss provisions (LLP) of
Big Bank Ltd. as on 32.03.2075.
Classification LLP (in lakhs)
Pass loan 2,500
Watchlist 500
Re-scheduled 350
Sub-standard 150
Doubtful 400
Bad 1,000
Sub-standard loan is fully insured with DCG Fund. Find out the level
of Non Performing Loan (NPL) as on 32.03.2075 of the bank. 5

The Institute of Chartered Accountants of Nepal 13


Suggested Answer- December 2018

Solution-4(a)

i) Operating Profit Margin 125/500*100=25%


210/700*100=30%
=Operating Profit/Sales*100

ii) Current Ratio 110:208= 0.53:1 256:27=9.48:1


=Current Assets: Current Liabilities

iii) Acid Test Ratio (110-41):208=0.33:1 (256-


39):27=8.04:1
=(Current Assets-Inventories): Liabilities

iv) Inventory Days 41/300*365= 50 Days 39/350*365= 41


Days
Inventory/Cost of Sales*365 Days

v) Receivable Days 69/500*365= 50 Days 67/700*365=35


Days
Receivable/Revenue*365 Days

vi) Payable Days 37/300*365= 45 Days 27/350*365=28


Days
Payable/Revenue*365 Days

vii) Return on capital employed 125/1712*100=7.3%


210/1829*100=11.48
=Operating Profit/(Total Assets-Current Liabilities)

viii) Gearing Ratio 300/1712*100=17.52%


500/1829*100=27.34%
or
Debt Equity Ratio
= Long term liabilities/(Total Assets- Current Liabilities)

Solution-4(b)
Statement of calculation of NPA as on 31.03.2075.
(Amount in Lakhs)
Classification LLP LLP rate Loan amount
Pass loan 2,500 1% 250,000
Watchlist 500 5% 10,000
Re-scheduled 350 12.50% 2,800
6.25%(insured so 25% only required) 2,400
Sub standard 150
Doubtful 400 50% 800
Bad 1,000 100% 1,000

The Institute of Chartered Accountants of Nepal 14


Suggested Answer- December 2018

Total 267,000
NPA 7,000
NPA% 2.62
5.
a) Tinkune Ltd.‘s head office building is the only building it owns.
Using professional valuers, it revalued the building on 1st Shrawan
2074, at Rs. 21,00,000. Tinkune Ltd. has adopted a
revaluation policy for buildings from this valuation date and has
decided that the original useful life of buildings has not changed as a
result of the revaluation. The building was acquired on 1st Shrawan
2064. The cost of the building on acquisition was Rs. 25,00,000 and
the accumulated depreciation to the Ashadh end, 2074 amounted to
Rs. 5,00,000. The depreciation up to 1st Shrawan 2074 was
depreciated evenly since acquisition. The professional valuer
believes that the residual value on the building would be Rs.
6,00,000 at the end of its useful life.
Required: 5
Calculate the depreciation amount of the building for the year ended
32nd Ashadh 2075 based on the information provided in the above
scenario.
b) A machine having expected useful life of 6 years is leased for 4
years. Both the cost and fair value of the machinery are Rs.
17,00,000. The amount will be paid in 4 equal installments and at the
termination of lease, lessor will get back the machinery. The
unguaranteed residual value at the end of the 4th year is Rs.
1,70,000. The IRR of investment is 10%. The present value of
annuity factor of Rs. 1 due at the end of 4th year at 10% IRR is 3.169.
The present value of Rs. 1 due at the end of 4th year at 10% rate of
interest is 0.683.
State with reason on the basis of your calculation, whether the lease
constitutes finance lease or not. 5
c) Rahul Trading gives the following information relating to items
forming part of inventory as on 32-3-2075. His factory produces
Product X using Raw material A.
i) 600 units of Raw material A (Produce @ Rs. 120). Replacement
cost of raw material A as on 32-3-2075 is Rs. 90 per unit.
ii) 500 units of partly finished goods in the process of producing X
and cost incurred till date Rs. 260 per unit. These units can be
finished next year by incurring additional cost of Rs. 60 per unit.
iii) 1500 units of finished product X and total cost incurred Rs. 320
per unit. Expected selling price of Product X is Rs. 300 per
unit.
Determine how each item of inventory will be valued as on 32-3-
2075. Also calculate the value of total inventory as on 32-3-2075. 5
Solution-5(a)
The depreciation amount is as follow:

To calculate the new depreciation amount, we use the following depreciation formula.

The Institute of Chartered Accountants of Nepal 15


Suggested Answer- December 2018

Revalued cost of asset-residual value 21,00,000-6,00,000


Expected useful life of asset 40 Years
Depreciation per year 37,500

Working Note 1
Building-original cost 25,00,000
Building- Accumulated Depreciation 5,00,000
Accumulated Depreciation/Cost= 20%
Building has been depreciated by 20 % over 10 years, so annual rate of depreciation has
been 2 % i.e. 20%/10 years, as asset has been depreciated evenly since acquisition.
Therefore, the original useful life is 50 years and the remaining useful life is 40 years.
Solution-5(b)
As per NAS 17 on "Leases", one of the situations that individually or in combination
would normally lead to a lease being classified as a finance lease is that if at the
inception of the lease the present value of the minimum lease payment amounts to at
least substantially all of the fair value of the leased asset.
Determination of nature of lease Rs.
Fair value of asset is Rs. 17,00,000 and unguaranteed residual value is Rs.1,70,000
Present value of residual value at the end of 4th year =1,70,000*0.683 =1,16,110
Present value of lease payment recoverable = 17,00,000–1,16,110 =15,83,890
The percentage of present value of lease payment to
fair value of the asset is =(15,83,890/17,00,000)*100% =93.17%
Since the present value of minimum lease payment substantially cover the major
portion of fair value of leased assets and life of the asset, the lease transaction meets the
definition of finance lease as per NAS -17. Hence, it constitutes a finance lease.
Solution-5(c)
As per NAS 2 ‗ Inventories‘ are valued at lower of cost and net realizable value.
Materials and other supplies held for use in the production of inventories are not written
down below cost if the finished products in which the will be incorporated are expected
to be sold at cost or above cost. However, when there has been a decline in the price of
materials and it is estimated that the cost of the finished products will exceed net
realizable value, the materials are written down to net realizable value. In such
circumstances, the replacement cost of the materials may be the best available measure
of their net realizable value. In the given case, selling price of product X is Rs. 300 and
total cost per unit for production is Rs.320.
Hence the valuation will be done as under:
i) 600 units of raw material will be written down to replacement cost as market value
of finished product is less than its cost, hence value at Rs. 90 per unit.
ii) 500 units of partly finished goods will be valued at 240 per unit i.e. lower of cost
Rs. 320 (Rs. 260+ additional cost Rs. 60) or Net estimated selling price Rs. 240(
Estimated selling price Rs. 300 per unit less additional cost of Rs. 60)
iii) 1500 units of finished product X will be valued at NRV of Rs. 300 per unit since it
is lower than cost Rs. 320 of product X.

The Institute of Chartered Accountants of Nepal 16


Suggested Answer- December 2018

Valuation of Total Inventory as on 32-03-2075


Units Cost NRV/ Value=units x cost or
(Rs.) Replacement NRV whichever is
cost less(Rs.)
Raw material A 600 120 90 54,000
Partly finished goods 500 260 240 1,20,000
Finished goof X 1,500 320 300 4,50,000
Value of inventory 6,24,000
6. Write short notes on: (5×3=15)
a) Unexpired Risk Reserve
b) Financial Instrument
c) Receipt and Expenditure Account
d) Debt Service Coverage Ratio
e) Elements of Financial Statements

Solution-6
a) Unexpired Risk Reserve
As per rule 15 of the Insurance Regulation 2049, every Insurer operating Non Life
Insurance Business shall transfer an amount not less than fifty percent of the Net
Insurance Premium show in Revenue Account to the ―Unexpired Risk Reserve"
account. Such amount shall be allocated for every category of Insurance the Insurer
operating. e.g. An insurer operating Non Life Insurance Business and accepting risk
for Fire Insurance, Marine Insurance, Motor Insurance and Aviation Insurance, then
the insurer shall maintain the Unexpired Risk Reserve For each of the fire, marine
motor and aviation insurance.
Such Unexpired Risk Reserve shall be recognized as income in next year except the
Unexpired Risk Reserve maintained for Maine Insurance. In case of Marine
Insurance, Unexpired Risk Reserve maintained for it shall not be recognized as
income for at least three years.
b) Financial Instrument
Financial instrument is any contract that gives rise to a financial asset to one entity
and a financial liability or equity instrument to another entity. Hence, financial
instruments include financial assets, financial liability and equity instrument. This
means that financial assets of one entity shall be financial liabilities or equity
instruments of another entity and financial liabilities or equity instrument of one
entity shall be financial assets of another entity. For example, bond, debenture or
bank loan is financial liabilities of entity issuing such bond or debenture or raising
loan and it is financial assets for holder of debenture or bond holder or provider of
loan. Similarly, share capital is equity instrument for share issuing entity and it is
financial assets of holder of equity.

c) Receipt and Expenditure Account


Receipt and Expenditure Account also can be taken as part of Financial Statements.
Some non-profit making organization like professional firms, educational institutes
etc. prefers to prepare Receipts and Expenditure account instead of Income and
Expenditure account as part of Financial Statements. Such an account includes all
expenses on accrual basis but incomes are recorded on cash basis. In other words, to

The Institute of Chartered Accountants of Nepal 17


Suggested Answer- December 2018

find out the result, all outstanding expenses are taken into account but the incomes
that are outstanding are not considered. The main reason behind this kind of
practice is that professionals consider it imprudent and risky to recognize the
outstanding incomes.

d) Debt Service Coverage Ratio


The ratio is a key financial ratio for the lenders.
 Debt servicing means timely payment of principal amount of instalments plus
interest.
 Borrower should be able to service the debt out of the profits. Profit means the
profit available for debt servicing.
 This ratio is calculated as:
Profit available for Debt Servicing
Loan instalments +Interest

 This ratio normally should be 1.33 but a higher coverage is of advantage to the
business as it improves its strength to service the debts promptly

e) Elements of Financial Statements


The framework classifies items of financial statements can be classified in five
broad groups depending on their economic characteristics: Asset, Liability,
Equity, Income/Gain and Expense/Loss.
Assets Resource controlled by the enterprise as a result of past events
from which future economic benefits are expected to flow to
the enterprise
Liability Present obligation of the enterprise arising from past events,
the settlement of which is expected to result in an outflow of a
resource embodying economic benefits.
Equity Residual interest in the assets of an enterprise after deducting
all its liabilities.
Income/Gain Increase in economic benefits during the accounting period
in the form of inflows or enhancement of assets or decreases
in liabilities that result in increase in equity other than those
relating to contributions from equity participants
Expenses/Loss Decrease in economic benefits during the accounting period
in the form of outflows or depletions of assets or incurrence
of liabilities that result in decrease in equity other than those
relating to distributions to equity participants.

The Institute of Chartered Accountants of Nepal 18


Suggested Answer- December 2018

Specific Comments on the performance of the students


Batch: - December 2018
Level: - CAP-II
Subject: Advanced Accounting
Question No. 1
Many students failed to exactly calculate no. of shares to be issued. Seems to lack of
proper conceptual knowledge in amalgamation problems.
Question No. 2
(a) Students lack proper understanding in determining short sales and the amount of
claim. None of the student solved this problem accurately.
(b) No clear concepts were found in Journal entries.
Question No. 3
(a) Calculation of purchase consideration and its settlement was not done well by
majority of students. Most of the students failed in computing intrinsic value per
shares and amount of PC.
(b) Students did not understand to present the items in the CFS. Further students lack
proper knowledge in journal entries and Balance sheet.
Question No. 4
(a) Calculation of ROCE and Gearing ratio is not done well by majority of students.
Almost all students could not be able to compute Receivable days, Payable days.
(b) Students are not well aware of calculation of NPA. Almost all students could not
find out loan amount given the LLP rate.
Question No. 5
(a) Calculation of original useful life is done well by students.
(b) Students did not understand to calculate the PV of leased asset.
(c) Major students fail to calculate value of WIP and in some cases RM.
Question No. 6
(a) Lack of concept of Insurance business.
(b) Some of the students did not understand the term FI.
(c) Students are confused with R & P A/c.
(d) Lack of concept of Debt service coverage ratio.

The Institute of Chartered Accountants of Nepal 19


Suggested Answer- December 2018

Paper 2:

Audit & Assurance

The Institute of Chartered Accountants of Nepal 20


Suggested Answer- December 2018

Audit & Assurance


Suggested
Roll No……………. Maximum Marks - 100

Total No. of Questions- 7 Total No. of Printed Pages- 10

Time Allowed - 3 Hours


Marks
Attempt all questions.

1. As an auditor, give your opinion with explanations on the following cases: (45=20)
a) Haribhakti Sugar Mills Limited is closed from last 11 months due to
outdated technology and has no sales as old technology produces inferior
products. The company will take minimum 2 years' time with substantial
modification of technology to restart production and make sales. Cost is very
substantial for upgrading technology. Due to factory closure, company
defaults in loan repayment and bank has issued notice for auction if loan is
not repaid within 3 months' time. Management is doubtful that funding can
be arranged.
CFO is of view that financial statement shall be prepared in going concern
basis as there is little hope that funding will be received.
b) The annual general meeting of Nepal Hydropower Limited failed to appoint
the auditor for the fiscal year 2074/75 due to time constraint and delegated
power to the board under the terms recommended by the audit committee.
The board of directors appoints M/s ABC & Co., Chartered Accountants as
auditor. Do you think the appointment is valid?
c) The total assets of Rs. 250 million of Y & Z Limited includes inventory
amounting to Rs. 50 million. The inventories were valued at cost. The
market price of the inventories was Rs. 42 million. The company has
disclosed this fact in the notes to accounts.
d) A Co. Ltd. has not included in the Balance Sheet as on 32-03-2075 a sum of
Rs. 1,500,000 being amount in the arrears of salaries and wages payable to
the staff for the last 2 years because the negotiations were going since last 18
months which concluded on 30-04-2075. The auditor wants to sign the said
financial statement and give the audit report on 31-05-2075. The auditor
came to know the result of the negotiations on 15-05-2075.
Answer:
a) As per NAS 1, an entity shall prepare its financial statements on a going concern
basis unless management intends to liquidate the entity or to cease trading, or that it
has no realistic alternative but to do so. When preparing Financial Statements,
management shall make an assessment of an entity's ability to continue as a going
concern. In making assessment, management is aware of material uncertainties
related to events or conditions that may cast significant doubt upon the entity's
ability to continue as a going concern, the entity shall disclose those uncertainties.
When financial statements are not prepared on going concern basis, It shall disclose
the fact, together with the basis on which it prepared the Financial Statements and
the reasons of doing so.

The Institute of Chartered Accountants of Nepal 21


Suggested Answer- December 2018

NSA 570 ―Going Concern‖ requires that the auditor shall consider whether there are
events or conditions that may cast significant doubt on the entity‘s ability to
continue as a going concern. In the given case, entity's inability to operate the
business for more than 11 months' time and significant doubt on availability of the
fund is one such example. If the financial statements have been prepared on a going
concern basis but, in the auditor‘s judgment, management‘s use of the going
concern assumption in the financial statements is inappropriate, the auditor shall
express a modified opinion.

In the given case, as there is significant doubt that entity will be operational and
going concern assumptions is questionable. The auditor shall evaluate the data on
which basis the management has made assessment and come to the conclusion of
the appropriateness of use of going concern assumption. Based on evaluation, the
auditor should form an appropriate opinion of the Financial Statements.

b) According to Section 110 of the Companies Act 2063, every company shall appoint
auditor under the act to have its accounts audited. As per the Section 111, the
general meeting shall appoint the auditor of the company from the amongst the
auditors licensed to carry out audit under the prevailing laws subject to Chapter 18
of the act in case of a public limited company. The act also provides that the board
of directors may appoint the auditor prior to holding first annual general meeting.
There is no any provision to delegate the authority to anyone for appointment of an
auditor. In the case of a public limited company the annual general meeting has
authority to appoint the auditor under the terms and conditions as recommended by
the Audit Committee as per Section 165 of the act.
As per section 113 of the act, in case of failure to appoint an auditor in the general
meeting of the company for any reason or where annual general meeting cannot be
held, the auditor is appointed by the Company Registrar‘s Office at the request of
the board of directors of the company.
Hence, the companies Act does not have any provision of delegating power of the
appointment of the auditor and no one can appoint auditor except the annual general
meeting and Company Register's Office in case of failure to appoint the auditors by
the AGM.
In the above context, the appointment of M/s ABC & Co., Chartered Accountants
by the board of Nepal Hydropower Limited is not valid. The Company Register's
Office can only appoint the auditor at the request of board of directors where annual
general meeting fails to appoint auditor.

c) As per Nepal Accounting Standards (NAS) 2, Inventories should be measured at


lower of cost and net realizable value. In the present case the cost price of the
inventories is Rs. 50 million and net realizable value is Rs. 42 million and hence the
inventories should be presented at Rs. 42 million in the balance sheet. However, the
company has presented the inventories at Rs. 50 million and disclosed in the notes
to accounts that the inventories have been presented at cost although its net
realizable value is lower than the cost. Mere disclosure of this fact in the notes
however does not result into compliance with the accounting standard.
Hence as an auditor, I will qualify my audit report because inventory in the present
case represents material item of the assets of the company and it has been materially
misstated in the balance sheet.

The Institute of Chartered Accountants of Nepal 22


Suggested Answer- December 2018

d) As per NAS 10 ―Events after the reporting period‖, adjustments to assets and
liabilities are required for events after the reporting date that provide additional
information materially affecting the determination of the amounts relating to
conditions existing at the reporting date. Similarly as per NAS 37 "Provisions,
Contingent liabilities and Contingent Assets", future events that may affect the
amount required to settle an obligation should be reflected in the amount of a
provision where there is sufficient objective evidence that will occur.
The amount of Rs 1,500,000 is a material amount and it is the result of an event,
which has occurred after the reporting date. The facts have become known to the
auditor before the date of issue of the Audit Report and Financial Statements. The
auditor has to perform the procedure to obtain sufficient, appropriate evidence about
the events occuring from the date of the financial statements i.e. 32-3-2075 to the
date of Auditors Report i.e. 31-05-2075. It is observed that as a result of long
pending negotiations a sum of Rs. 1,500,000 representing arrears of salaries of last
two years have not been included in the financial statements. It is quite clear that the
obligation requires provision for outstanding expenses. So the auditor should
request the management to adjust the sum of Rs. 1,500,000 by making provision for
expenses. If the management does not accept the request the auditor should qualify
the audit report.

2. Give your comments on the following cases: (45=20)


a) Mr. A, an auditor of Cold Drink Company has obtained trade secret formula
during audit process. A Case was lodged against Cold Drink Company for
putting non-edible components in the drink. Subsequently, auditor was
called by Supreme Court to provide documents and his knowing in the
formulae. He shared the information he has received on formula of Cold
Drink Company. Mr. B, lodged complaints to ICAN that, Mr. A has violated
Code of Ethics on the ground of breach of confidentiality.
b) National Company Limited had definite plan of its business being closed
within a short period from the close of the accounting year ended on
32ndAshadh, 2075. The Financial Statements for the year ended 32ndAshadh,
2075 had been prepared on the same basis as it had been in earlier periods
with an additional note that the business of the Company shall cease in near
future and the assets shall be disposed off in accordance with a plan of
disposal as decided by the management. The Statutory Auditors of the
Company indicated this aspect in "Key Audit Matters" only by a reference as
to a possible cessation of business and making of adjustments, if any, thereto
to be made at the time of cessation only. Comment on the reporting by the
Statutory Auditor as above.
c) Mr. Ajay, a practicing Chartered Accountant receives commission from Mr.
Sanjay, another practicing Chartered Accountant Rs. 175,000 being 25% of
the audit fee for the referral of statutory audit of company limited, a listed
company.
d) You are the auditor of Special Mart Ltd. for FY 2074/75. Your audit team
has approached to you on how to judge whether the particular risk is
significant or not. As a principal auditor how do you guide your audit team?

The Institute of Chartered Accountants of Nepal 23


Suggested Answer- December 2018

Answer:
a) Section 140.1 of Code of Ethics of ICAN, specifies about the Confidentiality to be
observed by members and professional accountants. The principle of confidentiality
imposes an obligation on all professional accountants to refrain from: (a) Disclosing
outside the firm or employing organization confidential information acquired as a
result of professional and business relationships without proper and specific
authority or unless there is a legal or professional right or duty to disclose; and (b)
Using confidential information acquired as a result of professional and business
relationships to their personal advantage or the advantage of third parties.
However, section 140.07, outlines circumstances where professional accountants is
required to disclose confidential information or when such disclosure is appropriate
and once such circumstances arise as required by law, for example i.e production of
documents or other provision of evidence in the course of legal proceedings.
Hence, in given case, complaint of Mr. B is not valid, as Mr. A has well followed
the code of Ethics and release confidential information on trade secret only upon
order of supreme court and such release of trade secret is allowed by Ethical Code
of ICAN and cannot be construed as release of confidential information by
breaching Ethical Code of ICAN for professional accountants.

b) As per NSA 570 ―Going Concern‖, management intentions to liquidate the entity or
to cease operations is one of the event or condition that may cast significant doubt
on the entity‘s ability to continue as going concern. If events or conditions have
been identified that may cast significant doubt on the entity‘s ability to continue as a
going concern but, based on the audit evidence obtained the auditor concludes that
no material uncertainty exists, the auditor shall evaluate whether, in view of the
requirements of the applicable financial reporting framework, the financial
statements provide adequate disclosures about these events or conditions
Further, as per NSA 701 ―Communicating Key Audit Matters in the Independent
Auditor‘s Report‖, when matters relating to going concern may be determined to be
key audit matters, and explains that a material uncertainty related to events or
conditions that may cast significant doubt on the entity‘s ability to continue as a
going concern is, by its nature, a key audit matter. NSA 701 also emphasizes on
auditor‘s responsibility to communicate key audit matters in the auditor‘s report.

As per the facts given in the case, intention of the National Company Limited had
definite plan of its business being closed down within short period from 32ndAshad,
2075. However, financial statements for the year ended 32ndAshad, 2075 had been
prepared on the same basis as it had been in earlier periods with an additional note.
Thus, management intentions to liquidate the entity or to cease operations is one of
the event or condition that may cast significant doubt on the entity‘s ability to
continue as going concern is a key audit matter. Therefore, the auditor is required to
communicate the Key Audit Matters in accordance with NSA 570 in above stated
manner. Simple reference as to a possible cessation of business and making of
adjustments, if any, be made at the time of cessation only by the auditor in his report
is not sufficient.

c) Section 240.5 of Code of Ethics of ICAN, accepting certain a referral fee or


commission creates a self-interest threat to objectivity and professional competence
and due care. According to Code of Ethics issued by ICAN, the payment or receipt

The Institute of Chartered Accountants of Nepal 24


Suggested Answer- December 2018

of commission by a professional accountant in public practice could impair


objectivity and independence. A professional accountant in public practice should
not therefore pay a commission to obtain a client nor should a commission be
accepted for referral of a client to a third party. The payment and receipt of
commission are permitted only for such engagements for which independence is not
required and the professional accountant in practice should nonetheless disclose the
facts to the client.

In the above case since the above assignments requires independence, Mr. Ajay and
Mr. Sanjay both are not complying with ethical requirements under Code of Ethics
issued by ICAN. The disciplinary action can be taken against Mr. Ajay and Mr.
Sanjay.

d) As per NSA 315 (Identifying and Assessing the Risks of Material Misstatement
through Understanding the Entity and Its Environment), in exercising judgment as
to which risks are significant risks, the auditor shall consider at least the following:
 Whether the risk is a risk of fraud;
 Whether the risk is related to recent significant economic, accounting or other
developments and, therefore, requires specific attention;
 The complexity of transactions;
 Whether the risk involves significant transactions with related parties;
 The degree of subjectivity in the measurement of financial information related to
the risk, especially those measurements involving a wide range of measurement
uncertainty; and
 Whether the risk involves significant transactions that are outside the normal
course of business for the entity, or that otherwise appear to be unusual. (Ref:
Para. A141–A145)
In the light of the aforesaid provision of NSA 315, I will guide my audit team to
ensure the risk identified by them are significant risk or otherwise.

3. Answer the following: (35=15)


a) In the light of NSA 315, explain understanding the entity and its
environment.
b) ―The auditor shall exercise professional judgment in planning and
performing an audit of financial statements‖. Comment.
c) What are the assertions with which an auditor is concerned with while
obtaining audit evidence from substantive procedures?

Answer:
a) The auditor‘s understanding of the entity and its environment consists of an
understanding of the following aspects:
 Industry, regulatory, and other external factors, including the applicable financial
reporting framework.
 Nature of the entity, including the entity‘s selection and application of accounting
policies.
 Objectives and strategies and the related business risks that may result in a
material misstatement of the financial statements.
 Measurement and review of the entity‘s financial performance.

The Institute of Chartered Accountants of Nepal 25


Suggested Answer- December 2018

 Internal control.

b) Nepal Standard on Auditing 300 (Planning an audit of financial statements) stated


that the manner in which the auditor emphasizes to engagement team members
the need to maintain a questioning mind and to exercise professional skepticism
in gathering and evaluating audit evidence.
Professional Judgment is the application of relevant training, knowledge and
experience, within the context provided by auditing, accounting and ethical
standards, in making informed decisions about the courses of action that are
appropriate in the circumstances of the audit engagement. That the auditor shall
exercise professional judgment in planning and performing an audit of financial
statements. Exercise of professional judgment depends on facts & circumstances
known to the auditor. It is to be exercised throughout the audit and to be
appropriately documented. It is important when deciding about:

 Materiality & audit risk.


 Nature, time and extent of audit procedures.
 Evaluating sufficiency & appropriateness of audit procedures.
 Evaluating management judgment in applying applicable Financial Reporting
Framework.
 Drawing conclusions based on audit evidence.

c) Nepal Standard on Auditing 500 (Audit Evidence) prescribes audit procedures for
obtaining audit evidences. Accordingly an auditor is concerned with following
assertions while obtaining audit evidences from substantive procedures:
 Existence: That an asset or liability exists at a given date.
 Rights and obligations: That an asset is a right of the concern and a liability is
an obligation at a given date.
 Occurrence: That a transaction or event which took place pertains to the entity
during the relevant period.
 Completeness: That there are no unrecorded assets, liabilities or transaction.
 Valuation: That an asset or liability is recorded at an appropriate carrying value.
 Measurement: That a transaction is recorded in the proper amount and revenue
or expense is allocated to the proper period.
 Presentation and disclosure: That an item is disclosed classified and described in
accordance with recognized accounting policies and practices and relevant
statutory

4. Answer/Comment on the following: (35=15)


a) Significant Familiarity and self-interest threats are noted due to using of
same senior personnel on an audit engagement over a long period of time.
What are the safeguards to be applied to eliminate such threats or reduce
them to an acceptable level?
b) Explain situation resulting in threat to objectivity and threat to fundamental
principles arising from conflict of interest with examples.
c) During the fiscal year 2073/74 ABC & Co., Chartered Accountants, a
proprietor Firm have done the following Statutory Audits.

The Institute of Chartered Accountants of Nepal 26


Suggested Answer- December 2018

Listed Company: 3
Limited Company: 10
Private Limited Company: 75
INGOs: 5
In the fiscal year 2074/75, one existing listed company did not continue as
auditor and other two companies has approached to the firm to appoint as
auditor. In addition, 2 other limited company and 2 NGOs approached to the
firm for the audit of FY 2074/75.

Answer:
a) Section 290.148 of ICAN Code of Ethics deals with this matter. The significance of
threat depends on time duration the individual has been a member of audit team,
role given to her/him, structure of Firm, nature of audit engagement, whether the
entity's management team is same or changed and the nature or complexity of the
Client's accounting and reporting issues. Accordingly, the safeguards to be applied
to eliminate such threats or reduce them to an acceptable level are:
 Rotating the senior personnel off the audit team;
 Having a professional accountant who was not a member of the audit team
review the work of the senior personnel; or
 Regular independent internal or external quality reviews of the engagement.

b) Section 220.1 of ICAN Code of Ethics deals with this matter. A professional
accountant in public practice may be faced with a conflict of interest when
performing a professional service. A conflict of interest creates a threat to
objectivity and may create threats to the other fundamental principles. Such threats may
be created when:
i. The professional accountant provides a professional service related to a
particular matter for two or more clients whose interests with respect to that
matter are in conflict; or
ii. The interests of the professional accountant with respect to a particular matter
and the interests of the client for whom the professional accountant provides a
professional service related to that matter are in conflict.
Examples of Conflicts of interest may include but not limited to:
 Providing a transaction advisory service to a client seeking to acquire an audit
client of the firm, where the firm has obtained confidential information during
the course of the audit that may be relevant to the transaction.
 Advising two clients at the same time who are competing to acquire the same
company where the advice might be relevant to the parties‘ competitive
positions.
 Providing services to both a vendor and a purchaser in relation to the same
transaction.
 Preparing valuations of assets for two parties who are in an adversarial position
with respect to the assets.
 Representing two clients regarding the same matter that is in a legal dispute
with each other, such as during divorce proceedings or the dissolution of a
partnership.

The Institute of Chartered Accountants of Nepal 27


Suggested Answer- December 2018

 Providing an assurance report for a licensor on royalties due under a license


agreement when at the same time advising the licensee of the correctness of the
amounts payable.
 Advising a client to invest in a business in which, for example, the spouse of the
professional accountant in public practice has a financial interest.
 Providing strategic advice to a client on its competitive position while having a
joint venture or similar interest with a major competitor of the client.
 Advising a client on the acquisition of a business which the firm is also
interested in acquiring.
 Advising a client on the purchase of a product or service while having a royalty
or commission agreement with one of the potential vendors of that product or
service.

c) 194th ICAN Council meeting dated 16th February 2015 had revised the earlier
council decision for the number of audits that can be carried out by the ICAN COP
holder with effect from 17th July 2015. As per revised decision, a member in
practice can conduct the audit of 100 organizations in a fiscal year including the
maximum 10 public companies.
In the given case the number of audit engagements of ABC & Co for the fiscal year
2074/75 as are follows:

Listed Company 3-1+2= 4


Limited Company: 10+2= 12
Private Limited Company: 75
INGOs: 5
NGOs: 2
Total: 98
Public Limited Company: 4+12 = 16.

As per the provision, a member in practice can audit 100 numbers of organizations
however the public limited company shall not exceed 10. In the given case, the total
number is below 100 however the public limited company has exceeded 10. So he
can accept only 10 audit engagements of public companies. He should not accept
the audit of either 6 public companies.
5. Answer the following: (25=10)
a) Miss Shristi, a Chartered Accountant, has been appointed as an auditor in the
22nd AGM of M/s Kantipur Ltd. She was removed by Board of Directors
when she was abroad for her personal visit.
b) Mr. Kumar KC is practicing as a Chartered Accountant from his
proprietorship firm. He nominated Mr. LK Khatri as partner on profit
sharing basis. Mr. Khatri, is not the member of ICAN.
Answer:
a) Section 119 (1) of the Company Act, 2063 provides that no auditor appointed
pursuant to Companies Act shall be removed pending the completion of audit of
accounts of any financial year for which he/she was appointed as the auditor.
As per Sub-section (2), notwithstanding anything contained in sub-section (1), if
any auditor breaches the code of conduct of auditors or does any act against the
interest of the company which has appointed him/her as the auditor or commits any

The Institute of Chartered Accountants of Nepal 28


Suggested Answer- December 2018

act contrary to the prevailing law, such auditor may be removed through the same
process whereby he/she was appointed as auditor, by giving prior information to the
ICAN, and with the approval of the regulatory authority, if any authorized by the
prevailing law for the regulation of business of the company concerned , and if there
is no such authority, with the approval of the Office of Registrar. While removing
an auditor pursuant to sub-section (2) above, the auditor shall be provided with a
reasonable opportunity to defend him/herself.
Thus, Board of Directors cannot remove if auditor has been appointed through
AGM. Further, reasonable opportunity to defend should be provided.

b) Chapter VIII of Section 34.3 of Nepal Chartered Accountant Act, 1997 states that
one shall not share or distribute as profit the auditing fees or remuneration with any
person other than a member of the Institute; and shall not pay any commission,
brokerage etc. out of the professional fees earned to any person or member.

In the given instance; Mr. Kumar, has nominated Mr. Khatri as a partner, who is not
member of ICAN on profit sharing basis, which is against the code of conduct
prescribed by ICAN.
Given the facts, Mr. Kumar is not allowed to nominate Mr. Khatri, who is not
member of ICAN as partner on profit sharing basis. Upon complaint to the Institute
against Mr., Kumar, for not upholding the conduct mentioned in this Act or the
Regulations framed under this Act, the Executive Director shall, if he finds
convincing information that proves Mr. Kumar, is not observing the conduct, submit
the proposal along with the related facts to the Council for further action against
such member or member holding Certificate of Practice, and Mr. Kumar, may face
disciplinary action by ICAN.

6. Write short notes on the following:


(42.5=10)
a) Employment with an Audit Client
b) Professional Skepticism
c) Peer Review
d) Independence of Internal Auditor
Answer:
a) Employment with an audit client may create self-interest and self-review threats.
Later on, familiarity or intimidation threats may be created if a director or officer of
the audit client, or an employee in a position to exert significant influence over the
preparation of the client‘s accounting records or the financial statements on which
the firm will express an opinion, has been a member of the audit team or partner of
the firm.

b) Professional skepticism is an attitude which includes a questioning mind, being alert


to conditions which may indicate possible misstatement due to error or fraud, and a
critical assessment of evidence. Therefore, the auditor should recognize the fact
that circumstances may exist that may cause the financial statements to be
materially misstated throughout the audit process.

The Institute of Chartered Accountants of Nepal 29


Suggested Answer- December 2018

c) This is a critical independent review of one public accounting firm's practices by


another public accounting firm. It is a review of the firm‘s accounting and auditing
practices. It is intended that the review be done by practitioners upon fellow
practitioners. Such an external review offers a more objective evaluation of the
quality of performance than could be done by self-review.

Peer review studies the adequacy of the firm's established quality control policies
and tests to determine the extent of the firm's compliance to these policies.
Suggestions for improvement to the system are outlined in a letter of comments
issued by peer reviewers to the reviewed firm. If a firm fails to take appropriate
corrective action, various actions may be imposed e.g. suspension from
membership.

In carrying out the review it is limited to:-

 Professional aspects of the practice.


 The overall total quality control policies.
 Professional aspect of the firms accounting and auditing practices

d) The concept of independence is equally relevant for internal auditor too. Internal
auditing is an independent, objective assurance and consulting activity designed to
add value and improve an organization‘s operations.
Internal auditor is part of the management but s/he evaluates the functioning of the
management at different levels. Therefore, to be efficient and effective, the internal
auditor must have adequate independence. It may be noted that by its very nature,
the internal audit function cannot be expected to have the same degree of
independence as is essential when the external auditor expresses his opinion on the
financial information. To ensure his independence, he is made responsible directly
to the Board of Directors through audit committee.
Such a channel of communication provides an independence whereby an internal
auditor can communicate and share his views on the scope of internal audit,
findings, etc. If internal auditor is made subordinate to lower level, his
independence will be effected which will affect his functioning and effectiveness.

7. Distinguish between: (25=10)


a) Audit Report and Audit Certificate
b) Test Checking and Routine Checking
Answer:
a)
Basis Audit Report Audit Certificate
Meaning An Audit Report is an expression of Certificate is a written
opinion on the true and fair view confirmation of the accuracy of
presented by financial statements the fact stated therein and does
not involve any estimate of
opinion.

The Institute of Chartered Accountants of Nepal 30


Suggested Answer- December 2018

Utility The term audit report is used when The term certificate is used when
the auditor expresses his opinion on the auditor verifies certain exact
the financial statements fact e.g. Royalty payment made
to foreign collaborators, value of
import/exports of a company
during a financial year.
Implication Audit report implies that the auditor A certificate implies that the
- Has examined relevant records in Auditor
accordance with generally accepted - Has verified certain precise
auditing standards; and figures; and
- Is expressing an opinion whether or - Is in a position to vouch their
not the financial statements accuracy as per the examination
representing a true and fair view of of documents and books of
the state of affairs and of the account produced before him.
working results of the enterprise.

Accuracy The Auditor is responsible for The Auditor is responsible for the
ensuring that the report is based on factual accuracy of what is stated
factual data that his opinion is in therein.
accordance with facts, and that it is
arrived at by application of due care
& Skill.

b)
Criteria Test Checking Routine Checking
Concept Test checking involves selecting a Routine checking involves
few transactions on the basis of checking of books and records on
auditor‘s judgment and examining regular basis.
them.
User Generally Auditor (Internal/external) Generally Accountants (Lower &
etc. Middle level).
Objectives The main object of test checking is to The main object of routine
form an opinion on the financial checking is ensuring arithmetical
statements on the basis of accuracy of the entries in the
original books and ledgers and
examination of selected sample.
posting to correct ledgers
accounts.
Scope Limited Wide
Time Lesser time consuming Higher time consuming
Reliance Certain reliance can be taken on Reliance cannot be taken on test
routine checking checking
Risk Higher risk of improper result if Lesser risk of improper result if
internal control system is weak. internal control system is strong.

The Institute of Chartered Accountants of Nepal 31


Suggested Answer- December 2018

Specific Comments on the performance of the students


Batch: - December 2018
Level: - CAP-II
Subject: Audit and Assurance
Question No. 1
(a) Provision of NAS -1 not written in majorities of cases. Majority of the students
failed to explain the concept of going concern.
(b) Fairly attempted.
(c) About 10% of the student not aware of the Provision of NAS -2. Overall
impression on this question was good. Well attempted by students.
(d) NAS 37 Provisions, not written in majorities of the cases.
Question No. 2
(a) The concept of confidentiality as a code of conduct was well attempted.
(b) The students failed to explain the KAM & going concern issues – poorly
attempted. Failed to explain the Provisions of NAS 701 and conclusions were not
proper. Students are not able to provide auditors conclusion.
(c) Fairly attempted.
(d) In general, this question was not attempted by the students – poor preparation.
Answer is too generalized and wrong in majorities of the cases. Students are not
able to provide auditors conclusion.
Question No. 3
(a) Fairly attempted.
(b) Well attempted. Most candidates answered generally W/O stating the areas where
"Professional judgment" is applied. Professional judgments are not appropriately
explained.
(c) Some of the student misunderstands the question. About 5% student could not
answer the question.
Question No. 4
(a) Specific safeguards weren't mentioned by most of the students. The safeguard for
the related threats were not explained with the examples general answer were
seen.
(b) Most of the students didn't explain and wrote the examples of situation of conflict
of interest properly.
(c) Few were unaware of the limit of Public complies and some were confused with
limited company. Mostly students answered as 15 Public Ltd. company instead of
10. Mostly confused with listed companies.
Question No. 5
(a) Fairly attempted. However, Answer is not correct in most cases.
(b) Fairly attempted but most of the students did not explain disciplinary action as
recommended by suggested answer.
Question No. 6
Majorities cannot give the concept. About 50% could not catch the relationship
between IA and Management. Students were confused with internal review. Students
were confused regarding audit and employment and Peer review.
Question No. 7
Majorities are confused about "Audit Certificate". There is a confusion on "Audit
report and Audit certificate" for most of the students. The distinction with the routine
checking was not understood by majority of the candidates.

The Institute of Chartered Accountants of Nepal 32


Suggested Answer- December 2018

Paper 3:

Corporate & Other Laws

The Institute of Chartered Accountants of Nepal 33


Suggested Answer- December 2018

Corporate & Other Laws


Suggested
Roll No……………. Maximum Marks - 100

Total No. of Questions - 7 Total No. of Printed Pages -14


Time Allowed - 3 Hours
Marks
Attempt all questions.
1. Answer the following questions: (5×5=25)
a) You are the legal advisor of the Dev Limited. Mr. Ram Lal, chairman
of the company wants to know the following:
i) When a public limited company is required to issue the prospectus?
ii) Who shall sign the prospectus?
iii) What are the procedures of publication of prospectus?
b) Mr. Salil is interested to carry some business. He has no knowledge of
the business organizations. However he could notice that some of his
friends are carrying business by incorporating a company. So he
approached you regarding the incorporation of a company as an expert
of the company law. How would you suggest him regarding the
process of the incorporation of a company as to his interest as per the
prevailing law?
c) The Board of Directors of Spring Water Pvt. Ltd. had proposed Mr.
Karna as an auditor but the resolution failed and there is no chance of
appointing of another auditor. Therefore, it was suggested appointing
the auditor by the Office of Company Registrar. Give your opinion.
i) When the office can appoint the auditor of a company?
ii) Can auditor be appointed by the office in the above situation?
d) Sagarmatha Securities Limited, incorporated in 2075, under the
Companies Act, 2063. This company has objectives of undertaking
securities business in Nepal. The company is going to convene the first
annual general meeting any appropriate time and duration. What is the
procedure for annual general meeting of a public company?
e) Mr. CP Mishra, shareholder of ABC Limited, a public company, holds
2% ordinary shares out of total paid up capital of Rs. 300 million. Mr.
Mishra is of the opinion that since his holding in less than 5%, he is not
the substantial shareholder of the ABC Limited and not required to
give any information to the company. Critically examine the opinion of
Mr. Mishra citing relevant provision of the Companies Act, 2063.
Answer:
a)
i) Public Limited Company is required to issue prospectus prior to issuing
shares to general public. Section 23 of the Companies Act, 2063 provides
that a public company shall publish its prospectus prior to issuing its
securities publicly.

The Institute of Chartered Accountants of Nepal 34


Suggested Answer- December 2018

ii) While publishing the prospectus, the prospectus must be signed by all
directors of the board of a company and same has to be submitted, along
with a written application made to the Securities Board for approval.
iii) If it appears that the prospectus submitted omits any important matter or
contains any unnecessary matter the Securities Board shall cause such
prospectus to be amended or altered as required and grant approval to
publish it in accordance with law. Prior to the approval the company shall
also make a declaration before the Securities Board that the provisions of
the Companies Act have been complied with; and the Securities Board
may, if it deems necessary, seek opinion of Office of Company Registrar
on that matter.
After receipt of approval from the Securities Board, company shall give
information thereof to Company Registrar Office in writing, accompanied by a
copy of the approval letter of the Securities Board.
On receipt of that information the Office of Company Registrar shall register
the prospectus. However if it appears that any matter contained in Companies
Act has not been complied with, the Office of Company Registrar may refuse
to register it.
In publishing the prospectus company shall mention that the prospectus has
been approved by the Securities Board and registered with the Company
Registrar Office and the date thereof. The covering page of prospectus shall
also mention that Securities Board or Office of Company Registrar shall not
be liable to bear any kind of responsibility in respect of matters mentioned
therein.

b) To carry business under the name of a company it is mandatory to register


them. According to section 5 (5) of the Companies Act, 2063 a person cannot
use the name of company to carry any kind of transaction by the name of any
institution or firm. Thus registration of a company is compulsory to carry any
business under its name.
The process regarding registration of a company has been prescribed in
Sections 3, 4 and 5 of the Companies Act. The first amendment made in
Companies Act as on 2074.1.19 allows incorporation of a company via
electronic transmission with digital recording.
According to Section 3 of the Companies Act, 2063. Any person desiring to
incorporate a company which may be private sector, public or company not
distributing profit under chapter IX can incorporate them.
According to the Section 4 of the Companies Act, 2063 the person or intended
promoters desiring to incorporate company must propose the name of the
proposed company for the approval of it. On receipt of the approval an
application is to be submitted along with the documents specified thereof via
digital transmission. The documents include:
i) Memorandum of Association /Articles of Association of the proposed
company.
ii) An agreement of the promoters in case of public company and consensus
agreement in case of private company if any.
iii) Prior approval of the concern line agencies where it is required.
iv) Certified copy of the citizenship certificate in case of Nepalese promoters,
and in case of organized institution their registration certificate

The Institute of Chartered Accountants of Nepal 35


Suggested Answer- December 2018

v) In case of Foreign investor of Company, prior approval of the foreign


investment or carrying business
vi) In case of foreign promoter certified documents indicating the citizenship
of such person.
vii) In case of Foreign Company or entity the registration documents of such
company or incorporation document.
According to Section 5 of the Act, on receipt of the application, if the
Registrar is satisfied with the legal compliance of the application, the
office will register such company within seven days after the date of
application and grant company registration certificate to the applicant and
becomes able to carry business under the name of a company.

c)
i) Companies Act, 2063 seeks to ensure that the appointment of an auditor is
not in the hands of the directors. That is why it is vested in the general
meeting of shareholders.
Section 113 provides that where the annual general meeting of a company
fails to appoint an auditor for any reason or where the annual general
meeting itself cannot be held or where the auditor appointed pursuant to
this Act ceases to continue his office for any reason, the Office may, at
the request of the board of directors of the company, appoint another
auditor. Therefore, the casual vacancy of auditor is fulfilled by the Office
of the Company Registrar whether the auditor is vacant by reason of
resignation or other causes.
ii) In the given situation where the company's annual general meeting fails to
appoint an auditor, the Office of Company Registrar may appoint another
auditor under the section 113 of the Companies Act, 2063.

d) Section 76 of the Companies Act, 2063 provide the procedure of the Annual
General Meetings.
1) The public company requires conducting the first annual general meeting
within 1 year of license of conducting business there after the annual
general meeting shall be held within six (6) months from the end of a
fiscal year.
2) If any public company fails to call the annual general meeting even within
three months after the expiry of the time-limit referred, the Office of
Company Registrar may give direction to call the annual general meeting
of such company.
3) If the company fails to call the annual general meeting even within three
months after the receipt of the direction, any shareholder may make a
petition, setting out the matter, to the court. Where such petition is made,
the Court may either cause to hold the annual general meeting or issue
any other appropriate order.
4) The shareholders present in the general meeting called pursuant to the
order of the court under (3) above shall be deemed to be a quorum.

e) As per section 50(1) of the Companies Act, 2063, If any person subscribes
ordinary shares with full voting rights that are five percent or more of the paid

The Institute of Chartered Accountants of Nepal 36


Suggested Answer- December 2018

up capital of any public company of which shares that person has held in
his/her name or through his/her agent, that person shall be deemed to have
his/her substantial shareholding in such company.
Provided, however, that in the case of a company having the paid up capital of
more than two hundred fifty million rupees, any person shall be deemed to
have the substantial shareholding if such person has subscribed one percent or
more of the total paid-up capital of such company.
Also as per sub-section (2), a substantial shareholder of every public company
shall give the company information setting out his/her name, address as well
as full particulars of the shares registered in his/her or his/her agent‘s name,
within thirty days after the knowledge of being a substantial shareholder of
that company.
So, as per the above mentioned provision Mr. Mishra is a substantial
shareholder of the company and need to give required information to the
company.

2. Answer the following questions: (3×5=15)


a) How does Nepal Rastra Bank mobilize Foreign Exchange Reserve?
Explain it in the light of Nepal Rastra Bank Act, 2058.
b) An application was filed to NRB by a proposed Samaj Bikas Bank for
the prior approval for its incorporation. It was found that one of the
proposed bank's major investor was convicted for 2 years in jail in a
case of assault. The concerned authority of NRB asked for your
opinion whether NRB can refuse to grant the prior approval on the said
ground. Referring the relevant provisions of the Banks and Financial
Institutions Act, 2073, give your advice in the following matters:
i) What are the grounds of refusal to give prior approval for the
incorporation of BFIs?
ii) Can NRB refuse to grant the prior approval on the said ground?
c) Ms. Pramila Pradhan shareholder of Hamro Bank Ltd. having adequate
number of shares required to be a director of the company. Her date of
birth is 2050 Falgun 5,and filed candidacy for Director of the bank for
the coming annual general meeting to be held on 2075 Mangsir 25. If
the candidacy filed by Ms. Pradhan is valid as per the provision of
Banks and Financial Institutions Act, 2073? State when Ms. Pramila
Pradhan becomes eligible to be a candidacy of a Director of the
company?
Answer:
a) In pursuance to section 66 of the Nepal Rastra Bank (NRB)Act, 2058,
NRB will mobilize Foreign Exchange Reserve in the following
manner:
i) NRB will mobilize the foreign exchanges reserve. Such reserve
will be denominated in the respective foreign exchange and such
reserve will consist of the following assets:-
(a) Gold and other precious metals held by or for the account of NRB;
(b) Foreign currencies held by or for the account of NRB;
(c) Foreign currencies held in the accounts of NRB on the books of a
foreign central bank or other foreign banks;

The Institute of Chartered Accountants of Nepal 37


Suggested Answer- December 2018

(d) Special drawing rights (SDR) held by NRB at the International


Monetary Fund;
(e) Bill of exchange, promissory note, certificate of deposit, bonds, and
other debt instrument payable in convertible foreign currencies
issued by any debtor or liability holder and held by NRB;
(f) Any forward purchase or repurchase agreements of NRB concluded
with or guaranteed by foreign central banks or public international
financial institutions, and any futures and option contracts of NRB
providing for payment in freely convertible foreign currency.
ii) While selecting the assets referred to in Sub-section (1), due
consideration should be given to NRB's capital and liquidity to
maximize earnings.
iii) NRB will maintain international reserve at a level, which will be
adequate for the execution of monitory and exchange rate policies
and for the prompt settlement of the international transaction.
iv) If international reserves have declined or, in the opinion of Bank,
are in danger of declining to such an extent as to jeopardize the
execution of the monetary or exchange rate policies in the prompt
settlement of the country's international transactions, NRB shall
submit to Government of Nepal a report on the international
reserves position and the causes which have led or may lead to
such a decline, together with such recommendations as it
considers necessary to remedy the situation.
v) Until such time as, the situation referred in Sub-section (4) has
been rectified, NRB will make further such report and
recommendations to Government of Nepal.
vi) NRB will hold the foreign exchange reserve referred to in sub-
section (1) in its balance sheet.
b)
i) Section 4 (1) of the Bank and Financial Institutions Act (BAFIA), 2073
has provided a mandatory provision that a prior approval from the NRB
shall be obtained to incorporate the bank and financial institutions.
Subsection (3) reads that no prior approval shall be granted to incorporate
bank and financial institutions by the company where the following
persons or their family members are the major investors:
(a) Regulatory action taken by NRB.
(b) Punished in banking offences.
(c) Punished in fraud, forgery, deceit.
(d) Punished in money laundering and financing in terrorist activities.
(e) Punished in corruption related offences
(f) Punished in the grievous offences like rape, human trafficking,
abduction, hostage etc.
ii) As the assault has not been included under subsection (3), the NRB
cannot refuse to grant the prior approval for the incorporation of the
proposed bank on the said ground.

c) As per the Section 18 (1) (a) of the Banks and Financial Institution Act, 2073
any person below 25 years of age is disqualified from being Director of The
Bank and Financial Institution.

The Institute of Chartered Accountants of Nepal 38


Suggested Answer- December 2018

In given case Ms. Pramila Pradhan is below the 25 years of age at the date of
Annual general meeting i.e. 2075 Mangsir 25. So as per section 18 (1) (a) of
the Banks and Financial Institution Act 2073 Ms. Pramila Pradhan cannot be
appointed the director of Hamro Bank Ltd. and candidacy filled by her is not
valid.
Ms. Pramila Pradhan becomes eligible to be candidacy of a director of the
company if the Annual General Meeting is to be held on or after 2075 Falgun
15. Because Ms. Pramila Pradhan will completed the 25 years of age at the
date of Annual general meeting and disqualification as per section 18 (1) (a) of
the Banks and Financial Institution Act 2073 will not apply. Ms. Pramila
Pradhan can be appointed the director of Hamro Bank Ltd, and The candidacy
filed by her will be valid.

3. Answer the following questions: (2×5=10)


a) Sun Life Insurance was going to provide a loan guarantee to Lion
Airlines while purchasing an aircraft where Mrs. Neeta Chand the wife
of Rakesh Chand, a director of the insurance company was working as
a managing director. One of the director objected that Insurance Board
may impose a ban on insurance business. Give your advice when a ban
may be imposed on insurance business by the Insurance Board
pursuant to the Insurance Act, 2049.
b) To regulate and manage the activities of the securities markets and
persons involved in the business of dealing in securities by regulating
the issuance, purchase, sale and exchange of securities for the purpose
of protecting the interests of investors in securities, Securities Act,
2063 is enacted. Explain how a body corporate issues securities under
the Act.

Answer:
a) Pursuant to Section 12A(1) of the Insurance Act, 2049, the Insurance Board
may impose a ban entirely or partially or may cancel any type of business
being operated by the Insurer under the Insurance Business in any of the
following cases:
i) If the directives provided by the Insurance Board time to time regarding
the procedures to be followed by the Insurer during the operation of the
Insurance Business has been violated.
ii) If the Insurer provides loan to any corporate body in which any of its
Directors or his/her family is working as a Managing Agent or partner
or provides guarantee or security of any kind for any loan provided to
him/her by others by violating section 14.
iii) If the Insurer does not provide information to the Insurance Board to be
provided pursuant to section 15.
iv) If the Insurer does not maintain the accounts and record to be
maintained pursuant to section 19.
v) If the insurer does not maintain separate accounts and records to be
maintained separately pursuant to section 20.
vi) If the Insurer does not maintain the fund to be maintained by it pursuant
to section 21 or bears liability of one Insurance Business from the fund
maintained for another business.

The Institute of Chartered Accountants of Nepal 39


Suggested Answer- December 2018

vii) If the Insurer does not maintain the compulsory reserve fund to be
maintained by it pursuant to section 22.
viii) If the Insurer accepts the insurance risk without receiving the insurance
premium pursuant to section 22.
ix) If the Insurer does not re-insure pursuant to section 28.

In the above situation Sunlife insurance is going to provide gurantee to Lion


Airlines where Mrs. Neeta Chand wife of Rakesh Chand Managing Director
of Insurance company so the Insurance board may impose ban on insurance
business under clause 2 of Section 12A (1) of the Insurance Act, 2049.

b) The body corporate will issue its securities under the Securities Act, 2063 as
follows:

Under section 27(1) a body corporate shall have to register securities to be


issued by it with the Board prior to their issuance. For this a body corporate
shall have to make an application in the prescribed format, accompanied by
its memorandum of association, articles of association, documents related
with such securities, and the prescribed fees, to the Securities Board for
registering securities pursuant to Sub-section (1).
Where an application is received the Securities Board shall make necessary
inquiry into the matter and, if it considers appropriate to register such
securities, register such securities in the register as prescribed, indicating the
details of such securities and issue the securities registration certificate in the
prescribed format to the concerned body corporate.
 Under section 28 where a body corporate allots or sells securities after
registering such securities, the body corporate shall have to give a notice
along with the details of securities so allotted or sold to the Securities
Board within seven days.
 Upon receipt of a notice as referred as above, where it appears necessary to
make the allotment and sale of such securities fair and informative for the
interests of investors and the body corporate, the Securities Board may
give necessary directive to the concerned body corporate. It shall be the
duty of the concerned body corporate to abide by such directive.
 Under section 29 where a body corporate is to sell and distribute securities
to more than fifty persons at a time, it shall make public issue for the sale
and distribution of such securities. The period to be open for making
application of the securities to be issued as above shall be as prescribed.
The provisions relating to the value and allotment of securities for which
public issue has to be made shall be as prescribed.
Where securities for which public issue has been made once could not be sold
and have to be re-issued again within one year, the body corporate which so
issues the securities may, with the approval of the Securities Board, issue
such securities by mentioning the matters which are different than the matters
set forth in the previously published prospectus and the prospectus previously
published.

The Institute of Chartered Accountants of Nepal 40


Suggested Answer- December 2018

4. Answer the following questions: (2×5=10)


a) Some employees demanded for bonus in advance to the manager of a
company which was refused by him and asked the board of directors to
handle the matter. The board became unable to decide the matter as no
balance sheet and loss and profit accounts have been prepared. Can
bonus be distributed in advance? Give your answer stating the
provisions of the Bonus Act, 2030.
b) Ms. X is an entrepreneur. She has consulted you to know the additional
facilities that an industry owned by the woman has provided. Suggest
her on such additional facilities provided to an industry owned by
woman in the Industrial Enterprises Act, 2073.
Answer:
a) Section 11 of the Bonus Act, 2030 has provided the provisions for the
distribution of bonus in Advance. If any enterprise fails not submit the
balance-sheet and statement of profit and loss of such enterprise within the
time limit of six month of completion of Fiscal Year or if bonus could not be
distributed within the time limit of eight month from the close of fiscal year
by the reasons of inquiry or examination conducted by the concerned Labour
Office on the balance sheet and statement of profit and loss submitted the
management of such enterprise, after making tentative computation, shall
have to distribute at least five percent of the net, income as bonus.
After preparation of balance-sheet and statement of profit and loss or after
final assessment is made in this matter. If the amount of bonus distributed as
mentioned above is found less than the amount to be distributed as bonus, the
difference amount shall have to be redistributed to the employees
proportionately having considered previously distributed amount.
If the amount, distributed as bonus is found excess to the amount assessed for
distribution of bonus the excess amount, whatever may be, shall be deducted
for recovery it, while assessing the net income of the enterprise in the next
fiscal year.

b)
i) The Industrial Enterprise Act, 2073 has provided the additional facilities
to the company owned by the woman, pursuant to the Section 25 of
Industrial Enterprise Act, 2073 as follows:
(a) An industry is registered with ownership of woman only, there shall
be 35 percent exemption in the registration fee calculated and levied
in accordance with the law.
(b) An industry is registered with ownership of woman only; there shall
be 20 percent exemption in the amount of registration fee for
industrial property used in the same.
(c) The place shall be provided in priority as prescribed to the woman
entrepreneur if she is desirous to establish an industry in the
industrial region.
(d) An industry, owned to the woman entrepreneur only, apply for
export of the industrial production, shall be provided pre export loan
on the basis of financial condition.

The Institute of Chartered Accountants of Nepal 41


Suggested Answer- December 2018

5.
a) Mr. Ganesh Jha, a Chartered Accountant, certified from the Institute of
Chartered Accountants of India (ICAI), has applied in the Institute of
Chartered Accountants of Nepal (ICAN) for recognition. As you are
concern authority engaged in the ICAN, can he obtain the recognition
in accordance with the prevailing law of Nepal. Provide the
recognition provision for such person and foreign institute as provided
in the Nepal Chartered Accountant Act, 2053. 5
b) What are the functions, duties and powers of Auditing Standards Board
(AuSB) under the Nepal Chartered Accountants Act, 2053? 5

Answer:
a) Recognition of foreign certification is the act of regulatory body that is
Institute of Chartered Accountant of Nepal (ICAN). The Nepal Chartered
Accountant Act, 2053 has provided the authority to act for the recognition of
such person or foreign institute. The recognition provision to person
certification from foreign institute as well as recognition of foreign institute
has given below as provided in the section 26 and 27 of the Nepal Chartered
Accountant Act, 2053 :
Recognition to the person: Section 26
A person, having passed the Chartered Accountancy or equivalent
examination and has received training from any foreign accounting body
should submit an application, in a prescribed format, to the Institute for
recognition of such examination and training. The Council, upon receipt of
such application, makes a decision to grant or not to grant recognition to such
examination and training. Any condition that the Council deems necessary
that the applicant should accomplish before recognition of such examination
and training, shall be notified to the applicant. In case of notification of any
condition, the application for membership of the Institute pursuant to Nepal
Chartered Accountant Act, 2053, could be tendered only after
accomplishment of such condition.

Notwithstanding above in case of a person passed chartered accountancy


course from ICAI, he/she could not get the membership from ICAN unless
completed one year article training in the professional chartered accountants
firm registered in ICAN and passed the Chartered Accountant Membership
Examination conducted by ICAN. There are two papers for the examination.

Through applying the above provision Mr. Jha can obtain the recognition
form ICAN.
Recognition to the Foreign Institute: Section 27.
The Council with the prior approval of Government of Nepal may grant
recognition to foreign accounting bodies and recognize the examinations and
training conducted by such accounting bodies Nepal. The Council shall
maintain a list of such recognized foreign accounting bodies as per the given
provision above. The procedures as per section 26 need not to be followed

The Institute of Chartered Accountants of Nepal 42


Suggested Answer- December 2018

regarding the recognition of examinations and training provided by the


accounting bodies mentioned in such list.

b) According to Section 15(D) of the Act, Government of Nepal shall form an


Auditing Standard Board in order to systematize and regulate the
accountancy and auditing under the chairmanship of a person nominated by
Government of Nepal from amongst the Fellow Chartered Accountants. The
Functions, duties and powers of Auditing Standards Board to be formed
pursuant to Section 15D are given under Section 15E of the Nepal Chartered
Accountants Act, 2053 as follows:
(a) To provide for standards on auditing, also based on international
standards on auditing, in order to systematize and regulate the
accountancy and financial reports;
(b) To prepare appropriate modalities in order to develop standards on
auditing and publish materials relating to standards on auditing;
(c) To amend, improve and revise standards on auditing;
(d) To interpret standards on auditing;
(e) To perform other acts relating to standards on auditing.

6. Answer the following questions: (5×4=20)


a) What is promissory note? Show the difference between promissory
note and bill of exchange.
b) What do you understand by social welfare activities as referred in
Social Welfare Act, 2049? How and what special programmes are
operated by the Government of Nepal as per the Act?
c) List out the duties of workers about the occupational safety and health
under the Labour Act, 2074.
d) What is the object behind the World Trade Organization (WTO)?
e) State the circumstances when an agent becomes personally liable by
his act as per the part V chapter 10 of the Muluki Dewani (Samhita)
Ain, 2074.
Answer:
a) As per Section 2 (f) of the Negotiable Instruments Act, 2034 A promissory
note means an instrument in writing except government or bank note
containing an unconditional under taking, signed by the maker, to pay a
certain sum of money to, or to order of, a certain person, or to the bearer of the
instrument.
The person who makes the promissory note and undertakes obligations to pay
a certain sum of money mentioned thereon is called Maker. The person to
whom the payment is to be made is called Payee.
Difference between a promissory Note and a Bill of Exchange
Basis of Difference Promissory Note Bill of Exchange
1. Number of There are two parties There are three parties
parties
2. Maker and payee Maker and payee cannot Maker/drawer and payee
be can be the same and single
the same and single person
person.
3. Promise and There is a promise to pay a There is an order to pay a

The Institute of Chartered Accountants of Nepal 43


Suggested Answer- December 2018

order sum of money sum of money


4. Acceptance Acceptance is not required Acceptance is necessary
as it is signed by the by the drawee before it
person liable to pay on it can be presented for
payment
5. Nature of Liability of the maker of Liability of the maker of a
liability promissory note is bill is secondary
primary

b) Social Welfare Act, 2049 Section 2(a) defines ―Social welfare activity‖ means
the welfare activity oriented towards the economic and social upliftment and
self-reliance to the weak, helpless and disable individuals.
Under Section 3 of the Act, Government of Nepal, by means of different
activities relating to the social welfare work, to support the overall
development of the country may operate the social welfare programme
through the concerned Ministry and Social organizations and institutions.
Pursuant to Section 4 of the Act, Government of Nepal may operate special
Programmes, relating to the social welfare activity and social service, in the
following matters:
(a) To serve interest and render welfare to the children, old age, helpless or
disabled people.
(b) To foster participation in development and to promote and protect the
welfare, rights and interest of the women.
(c) To rehabilitate and help to lead a life of dignity to the victims of social
mischief's and also to juvenile delinquency, drug addicts and similar
people involved in other kind of addictions.
(d) To help to lead a life with dignity to the jobless, poor and illiterate people.
(e) To manage religious places and the activities of the trust Guthi
institutions.
(f) To take effective management and actions for the welfare of the backward
communities and classes.

c) Section 73 of the Labour Act, 2074 has imposed the various duties to be
followed by the workers at to the matter of occupational safety and health. The
duties are as under-
i) Not to do such act at the workplace knowingly or recklessly causing adverse
effect to safety and health of ones or others.
ii) To provide necessary help to the employer or concerned person to fulfill the
duties stated under this chapter.
iii) To get directives, advice and other information for the purpose of using or
operating safely and cautiously the materials, goods and equipment to be used
and operated at the work place.
iv) To operate and use those workplace, materials, goods and equipment safely and
carefully as per the directives, advice and other information made for the safety
use and handling of the workplace, materials, goods and equipment.
v) To use the personal safety equipments provided by the employer.

d) The World Trade Organization (WTO) is the only global international


organization dealing with the rules of trade between nations. At its heart are

The Institute of Chartered Accountants of Nepal 44


Suggested Answer- December 2018

the WTO agreements, negotiated and signed by the bulk of the world‘s trading
nations and ratified in their parliaments. The goal is to help producers of goods
and services, exporters, and importers conduct their business.
The WTO agreements are lengthy and complex because they are legal texts
covering a wide range of activities. But a number of simple, fundamental
principles run throughout all of these documents. These principles are the
foundation of the multilateral trading system in the member countries It
provides guidelines in trading activities on basis of following principles:
Non-discrimination
A country should not discriminate between its trading partners and it should
not discriminate between its own and foreign products, services or nationals.
More open
Lowering trade barriers is one of the most obvious ways of encouraging trade;
these barriers include customs duties (or tariffs) and measure such as import
bans or quotas that restrict quantities selectively.
Predictable and transparent
Foreign companies, investors and governments should be confident that trade
barriers should not be raised arbitrarily. With stability and predictability,
investment is encouraged, jobs are created and consumers can fully enjoy the
benefits of competition — choice and lower prices.
More competitive
Discouraging ‗unfair‘ practices, such as export subsidies and dumping
products at below cost to gain market share; the issues are complex, and the
rules try to establish what is fair or unfair, and how governments can respond,
in particular by charging additional import duties calculated to compensate for
damage caused by unfair trade.
More beneficial for less developed countries
Giving them more time to adjust, greater flexibility and special privileges;
over three-quarters of WTO members are developing countries and countries
in transition to market economies. The WTO agreements give them transition
periods to adjust to the more unfamiliar and, perhaps, difficult WTO
provisions.
Protect the environment
The WTO‘s agreements permit members to take measures to protect not only
the environment but also public health, animal health and plant health.
However, these measures must be applied in the same way to both national
and foreign businesses. In other words, members must not use environmental
protection measures as a means of disguising protectionist policies.

e) In the absence of any contract to that effect, an agent cannot personally


enforce contract entered into by him on behalf of his principal, nor is he
personally bound by them.
However, there are certain circumstances under which an agent becomes
personally liable by his act.
Section 595 of the Nepal Civil Code Act, 2074 prescribes the personal liability
of an agent. Except when otherwise provided for in the contract, the agent
shall be personally responsible for transaction enter by him/her on behalf of
principal in the following circumstances:

The Institute of Chartered Accountants of Nepal 45


Suggested Answer- December 2018

a. In case of express agreement: Where a contract made by an agent


specifically provides for the personal liability of the agent, the agent will
be personally liable.
b. In case of undisclosed principal: Where the contract is made by an agent
for an undisclosed principal, in the absence of any contract to the contrary,
it is presumed that the agent is personally liable.
c. In case not possible to file a case against the principal for any reason:
Where it becomes impossible to file a case against the principal for any
reason thereof, it is presumed that the agent is personally liable.
d. In case of contract made in his own name: Where a contract is made by an
agent in his own name without disclosing that he is contracting as an
agent, the agent is personally liable.
e. In case of an act done beyond the authority given thereof: Where an act is
done by an agent beyond the authority, the agent becomes personally
liable.
f. In case of fraud or misrepresentation: Where there involves fraud or
misrepresentation at the time of agency dealing, an agent becomes
personally liable.
g. In case of custom or usage of trade: Where there is a custom or usage or
trade making the agent personally liable, in the absence of any contract to
the contrary, agent is personally liable.
h. In case of involvement of benefit for the agent: Where there is a
involvement of benefit for agent himself the agent is personally liable.

7. Write short notes on the following: (2×5=10)


a) 'Cottage Industry' as per Industrial Enterprises Act, 2073
b) Explain the liability for untrue statement in prospectus
Answer:
a) Section 15 (1) (b) of the Industrial Enterprises Act, 2073 has defined the term
cottage industry. As per the Act the industry with the following conditions
called as a cottage industries-
(a) Based on traditional skill and technology.
(b) Labour intensive and based on specific skill or local raw materials and art,
culture and technology.
(c) If machine, engine or equipment is used, maximum capacity of electric
energy is 10 KW
(d) Industries as mentioned in Annex 2.

b) To protect the interests of prospective investors in the shares or debentures of


a company, the law prescribes a wider meaning to this term. The word untrue
statement is defined as a statement included in a prospectus shall be deemed
to be untrue, if the statement is misleading in the form and context in which it
is included. Where the omission from a prospectus of any matter is calculated
to mislead, the prospectus, shall be deemed, in respect of such omission, to be
a prospectus in which an untrue statement in included. Where an untrue
statement occurs in a prospectus, there may arise (i) civil liability (ii)
Criminal liability. Every person who is a director of the company at the time

The Institute of Chartered Accountants of Nepal 46


Suggested Answer- December 2018

of the issue of the prospectus, every promoter of the company and every
person, including an expert, who has authorized the issue of a prospectus,
shall be liable. The liability of these people is to the allotted of shares.
A company is responsible for a statement in prospectus if it was issued by
the company or by the Board of directors with the authority of the company
or by the promoters and the Board ratifies and adopts the issue of
prospectus. The following persons are liable to pay compensation for loss or
damages sustained by reason of untrue statement included in a prospectus:
However a director who resigns before the decisions made by the company to
publish the prospectus or who on becoming aware of any untrue statement in
the prospectus, publish a notice of that matter in the information of the
general public prior to the sale or allotment of securities or who pros that he
did not know that the prospectus contained any untrue statement shall not be
liable to bear such compensation.
The Companies Act, 2063, Section 24, has provided the liability for matters
contained in prospectus. Regarding the untrue statement, subsection (3) of
section 24, has ascertained as the prospectus contains false statements made
maliciously or deliberately and any person sustains any loss or damage by
reason of his/her subscription of securities on the faith of that prospectus, the
directors who have signed that prospectus shall be personally liable to pay
compensation for the actual loss or damage so sustained.

The Institute of Chartered Accountants of Nepal 47


Suggested Answer- December 2018

Specific Comments on the performance of the students


Batch: - December 2018
Level: - CAP-II
Subject: Corporate & Other Laws
Question No. 1
Students had not mentioned the Procedures regarding publication of Prospectus as per
legal framework. The students are unable to know the question clearly in case of
giving the answer on the basis of example. Provision V/s 113 has not covered
adequately. Students had confusion in regular AGM and first AGM.
Question No. 2
Students were confused in age limit. Amendments not updated. Students were
confused in stating the grounds of refusing the approval properly as mentioned in
Section 4(3) of BAFIA.
Question No. 3
Provision of registration cancellation/renewal not mentioned. Answer of 3(a) was well
written by few students with prefer section. Most of the students have written general
answer. In part (b) student could not give well answer.
Question No. 4
Almost student thought bonus is distributed in only the event of completion of
balance sheet. Students were unknown about the facilities for women
entrepreneurship. Students had rate confusion whether 5% or 10%. Similarly, from
which year profit bonus need to mention. They had also confusion on rate/ rebate and
concession percentage.
Question No. 5
Student did not elaborate process for membership, only examination process
mentioned. Students gave own answer rather than Provision of the Act. Students were
confused in recognition of foreign institute of CA.
Question No. 6
Students failed to answer properly about the duties of worker. They emphasize on
employer's duty. Most of the students gave general answer rather than based on Act.
Question No. 7
Students had not properly defined cottage industries as per the section 15 (1) (b) of
Industrial Enterprises Act 2073. Most of the students do not have good knowledge on
question (a).

The Institute of Chartered Accountants of Nepal 48


Suggested Answer- December 2018

Paper 4:

Financial Management

The Institute of Chartered Accountants of Nepal 49


Suggested Answer- December 2018

Financial Management
Suggested
Roll No……………. Maximum Marks - 100
Total No. of Questions – 7 Total No. of Printed Pages – 17
Time Allowed – 3 Hours
Marks
Attempt all questions.
Working notes should form part of the answer. Make assumptions wherever
necessary.

1. A company engaged in manufacture of household electrical goods has


taken a strategic decision to diversify operations and to make a major
investment in facilities for the manufacture of office equipment. The
new investment is being appraised over a four-year time horizon. The
company's finance director has prepared a revenue forecast.
She predicts that it will generate Rs. 2 million operating cash flows
before marketing costs in Year 1, Rs. 14.50 million in Year 2, Rs. 15.50
million in Year 3 and Rs. 15.80 million in year 4.
Marketing costs are predicted to be Rs. 9 million in Year 1 and Rs. 2
million in each of Years 2 to 4.
The new investment will require immediate expenditure on facilities of
Rs. 30.60 million. Tax allowable depreciation will be available on the
new investment at an annual rate of 25% reducing balance basis. It can
be assumed that there will either be a balancing allowance or charge in
the final year of the appraisal. The finance director believes that the
facilities will remain viable after four years, and therefore a realisable
value of Rs. 13.50 million can be assumed at the end of the appraisal
period.
The new facilities will also require an immediate initial investment in
working capital of Rs. 3 million. Working capital requirements will
increase every year by 5% for the next three years and any working
capital at the start of Year 4 will be assumed to be released at the end of
the appraisal period.
The company pays tax at an annual rate of 30%. Tax is payable with a
year‘s time delay. Any tax losses on the investment can be assumed to
be carried forward and written off against future profits from the
investment.
The company has been considering following two choices for financing
all of the Rs. 30.60 million needed for the initial investment in the
facilities:
Option1: Obtaining subsidised loan from a government loan scheme,
with the loan repayable at the end of the fourth year. Issue costs of 4% of
the gross finance would be payable. Interest would be payable at a rate
of 30 basis points below the risk free rate of 2.5%. In order to obtain the

The Institute of Chartered Accountants of Nepal 50


Suggested Answer- December 2018

benefits of the loan scheme, the company would have to fulfill various
conditions, including locating the facilities in a remote part of country
where unemployment is high.
Issue costs for the subsidised loan would be paid out of available cash
reserves. Issue costs are not allowable as a tax-deductible expense.
Option 2: Issuing loan notes with interest payable at 5%, which is
company's normal cost of borrowing.
In initial discussions, the majority of the board of directors favoured
using the subsidised loan. However, the chairman argued strongly in
favour of the loan notes, as, in his view, operating costs will be lower if
the company does not have to fulfill the conditions laid down by the
government. The company's finance director is sceptical, however, about
whether the other shareholders would approve the issue of loan notes on
the terms suggested. The directors will decide which method of finance
to use at the next board meeting.
Assume the discount rate to be 9% to calculate present value of the cash flows.
Required: 20
Calculate the adjusted net present value for the investment on the basis
that it is financed by the subsidized loan and conclude whether the
project should be accepted or not. Show all relevant calculations.

Answer:
Calculation of NPV
(Rs. in
million)
Year 0 1 2 3 4 5
Operating cash flows 2.00 14.50 15.50 15.80 -
excluding marketing costs
Marketing costs (9.00) (2.00) (2.00) (2.00) -
Cash flow before tax (7.00) 12.50 13.50 13.80 -
Taxation (WN 1) (0.39) (3.96)
Investment (30.60) 13.50 -
Working Capital (WN 2) (3.00) (0.15) (0.16) (0.17) 3.48 -
Cash flows (33.60) (7.15) 12.34 13.33 30.39 (3.96)
Discount Factor at 9% 1 0.917 0.841 0.771 0.707 0.648
Discounted cash flows (33.60) (6.56) 10.38 10.28 21.49 (2.57)
Net Present Value (Base (0.58)
case)
Add: Financing effect 1.57
(WN 3,4,5 & 6)
Adjusted NPV 0.99
WN 1: Taxation (Rs. in million)
Year Amount
Investment 30.60
Tax allowable Depreciation @ 25% on (7.65)
1
reducing balance
Balance 22.95

The Institute of Chartered Accountants of Nepal 51


Suggested Answer- December 2018

Tax allowable Depreciation @ 25% on (5.74)


2 reducing balance
Balance 17.21
Tax allowable Depreciation @ 25% on (4.30)
3 reducing balance
Balance 12.91
Balancing (gain) 0.59
4
13.50
Year 1 2 3 4
Cash flow before tax (7.00) 12.50 13.50 13.80
Tax allowable depreciation (7.65) (5.74) (4.30) 0.59
Adjusted Cash flow (14.65) 6.76 9.20 13.21
Offset of previous losses (14.65) (7.89)
Carried forward losses (14.65) (7.89)
Taxable cash flow 1.31 13.21
Tax @ 30% (0.39) (3.96)
Tax payable in Year 4 5
WN 2 : Calculation of working capital (Rs. in million)
Year 1 = 3x0.05=0.15, Year 2=(3+0.15)x0.05=0.16,
Year 3=(3+0.15+0.16)x0.05=0.17 and Year 4 = (3+0.15+0.16+0.17) =3.48
WN 3 : Issue costs
Debt: (Rs. 30.60 m/0·96) = Rs. 31.88 million
Debt issue costs: Rs. 31.88 million x 0·04 = Rs. 1.28 million
WN 4 : Tax shield on subsidised loan
Use PV of an annuity (PVA) for years 2 – 5 at 5% (assume 5% is cost of debt).
(Note: The risk-free rate of 2·5% could also be used for discounting.)
Tax shield: Rs. 30.60 million x (0·025 –0·003) x 0·3 x (4·326 – 0·952) = Rs.
0.68 million
WN 5: Subsidy Benefit
Benefit = Rs. 30.60 million x (0·05 – 0·022) x 3·543 = Rs. 3.04 million
Tax relief lost = Rs. 30.60 million x (0·05 – 0·022) x 0·3 x (4·326 – 0·952) =
Rs. 0.87 million.
WN 6: Financing side effect (Rs. in million)
Issue cost (1.28)
Tax Shield on subsidised loan 0.68
Subsidy Benefit 3.04
Tax relief lost on subsidy benefit (0.87)
Total benefit on financing side 1.57
Conclusion: If base case net present value is used, the project has a negative net
present value of Rs. 0.58 million and, on that basis it should be rejected.
However, the financing side effects add Rs. 1.57 million to the value of the
project, giving a positive adjusted net present value of Rs. 0.99 million. On that
basis, the project should be accepted. Further, the company should also consider
the financial implication of conditions attached to subsidized loan.

Q (1): Alternative Solution:

The Institute of Chartered Accountants of Nepal 52


Suggested Answer- December 2018

Calculation of Adjusted NPV for Subsidies Loan Financing


(Rs. in million)
Year 0 1 2 3 4 5
Operating cash flows excluding marketing
-
costs 2.00 14.50 15.50 15.80

Marketing costs -
(9.00) (2.00) (2.00) (2.00)
Interest cost {30.6 M*(2.5%-0.3%)}
(0.67) (0.67) (0.67) (0.67)
Cash flow before tax -
(7.67) 11.83 12.83 13.13
Taxation (WN 1)
(3.55)
Issue cost (WN 3) -
(1.28)
Sale of investment
13.50
Working Capital (WN 2) -
(3.00) (0.15) (0.16) (0.17) 3.48
Repayment of loan (30.60)
Cash flows (4.28) (7.82) 11.67 12.66 (0.49) (3.55)
Cash flows excluding interest and
repayment of loans (A) (4.28) (7.15) 12.34 13.33 30.78 (3.55)
Cash flows for interest and repayment of
loans (B) (0.67) (0.67) (0.67) (31.27)
Discount Factor at 9% (Assumed rate is
1.00 0.92 0.84 0.77 0.71 0.65
ke)( C )

Discounted cash flows(Excluding interest


and Repayment of loans) (D=A*C) (4.28) (6.56) 10.39 10.29 21.81 (2.31)
Discount Factor at 5% (Assumed Market
1.00
cost of Debt) ( E ) 0.95 0.91 0.86 0.82 0.78
Discounted cash flows(Excluding interest
and Repayment of loans) (F=B* E) (0.64) (0.61) (0.58) (25.73)

Total Discounted Cash Flows (D+F)


(4.28) (7.20) 9.78 9.71 (3.92) (2.31)
Adjusted Net Present Value
1.78

Calculation of Adjusted NPV for Loan Financing


(Rs. in million)
Year 0 1 2 3 4 5
Operating cash flows excluding marketing
2.00 14.50 15.80 -
costs 15.50

The Institute of Chartered Accountants of Nepal 53


Suggested Answer- December 2018

Marketing costs -
(9.00) (2.00) (2.00) (2.00)
interest cost
(1.53) (1.53) (1.53) (1.53)
Cash flow before tax -
(8.53) 10.97 11.97 12.27
Taxation (WN 1)
(2.52)
Issue cost - -
Sale of investment
13.50
Working Capital (WN 2) -
(3.00) (0.15) (0.16) (0.17) 3.48
Repayment of loan
(30.60)
Cash flows
(3.00) (8.68) 10.81 11.80 (1.35) (2.52)
Cash flows excluding interest and
repayment of loans (A) (3.00) (7.15) 12.34 13.33 30.78 (2.52)
Cash flows for interest and repayment of
loans (B) (1.53) (1.53) (1.53) (32.13)
Discount Factor at 9% (Assumed that
0.92
given discount rate is ke)( C ) 1.00 0.84 0.77 0.71 0.65

Discounted cash flows(Excluding interest


and Repayment of loans) (D=A*C) (3.00) (6.56) 10.39 10.29 21.81 (1.64)

Discount Factor at 5% (Assumed Market


0.95
cost of Debt) ( E ) 1.00 0.91 0.86 0.82 0.78

Discounted cash flows(Excluding interest


and Repayment of loans) (F=B* E) (1.46) (1.39) (1.32) (26.43)

Total discounted Cash Flows (D+F)


(3.00) (8.02) 9.00 8.97 (4.63) (1.64)
Adjusted Net Present Value
0.69

WN 1: Taxation (Rs. in million)


Year Amount
Investment 30.6
1 Tax allowable Depreciation @ 25% on reducing balance -7.65
Balance 22.95
2 Tax allowable Depreciation @ 25% on reducing balance -5.74

The Institute of Chartered Accountants of Nepal 54


Suggested Answer- December 2018

Balance 17.21
Tax allowable Depreciation @ 25% on reducing balance -4.3
3
Balance 12.91
Balancing (gain) 0.59
4
13.5

For Subsidies Loan Financing


Year 1 2 3 4
Cash flow before tax -7.6732 11.8268 12.8268 13.1268
Tax allowable depreciation -7.65 -5.74 -4.3 -0.59
Adjusted Cash flow -15.323 6.0868 8.5268 12.5368
Carried forward losses -9.2364 -0.7096 11.8272
Taxable cash flow 0 11.8272
Tax @ 30% 3.54816
Tax payable in Year 5

For Loan Financing


Year 1 2 3 4
Cash flow before tax -8.53 10.97 11.97 12.27
Tax allowable depreciation -7.65 -5.74 -4.3 -0.59
Adjusted Cash flow -16.18 5.23 7.67 11.68
Carried forward losses -10.95 -3.28 8.4
Taxable cash flow 0 8.4
Tax @ 30% 2.52
Tax payable in Year 5

WN 2 : Calculation of working capital (Rs. in million)

Year 1 = 3x0.05=0.15, Year 2=(3+0.15)x0.05=0.16,

Year 3=(3+0.15+0.16)x0.05=0.17 and Year 4 = (3+0.15+0.16+0.17)


=3.48

WN 3 : Issue costs
Debt: (Rs. 30.60 m/0·96) = Rs. 31.88 million

Debt issue costs: Rs. 31.88 million x 0·04 = Rs. 1.28


million

Conclusion: On that basis adjusted NPV as calculated above, the project with the
subsidies loan financing is having highest adjusted NPV, project with the subsidies
loan financing should be accepted. Further, the company should also consider the
financial implication of conditions attached to subsidized loan.

The Institute of Chartered Accountants of Nepal 55


Suggested Answer- December 2018

2.
a) Patan Surgical P. Ltd. (PSPL) is a supplier of medical equipment.
The company projects sales of Rs. 12 million for the year ended
Ashadh end 2075. PSPL‘s Board has set a strategic objective to
increase sales and reduce costs each year. In order to help achieve
these objectives in the year ended Ashadh end 2075, PSPL‘s
management has put forward the following proposals:
Proposal 1: The company‘s Sales Director has estimated that if credit
settlement terms were relaxed by offering customers 90 days
standard settlement terms, sales would increase by one quarter next
year, without the need to reduce prices.
Proposal 2: In a bid to achieve the Finance Department‘s cost
reduction target, PSPL‘s Financial Controller is contemplating
replacing the company‘s Credit Controller, who costs Rs. 1 million
annually, and referring potentially bad debts to a factoring agency
for collection. The factor has indicated that they would charge a
fixed annual fee of Rs. 980,000 for a ‗with recourse‘ factoring
service.
Further relevant details are as follows:
All sales are on credit with all debtors (except bad debts) fully
observing the credit terms of 30 days. Bad debts of 1% of gross sales
are currently experienced. The Sales Director expects that this will
increase to 5% if credit settlement terms are relaxed and the Credit
Controller is replaced by the factoring agency. PSPL‘s gross profit
margin is 25%. PSPL is funded by a bank overdraft, which costs
15% per annum.
Assume a 360-day year.
Required: 10
Evaluate the profitability implications of the above two related
proposals (if taken together) and recommend.
b) A company‘s capital structure consists of the following:
Rs. in million
Equity shares of Rs. 100 each 20
Retained earnings 10
9% Preference share 12
7% Debenture 8
50
The company is planning for an expansion. The expansion involves
Rs. 25 million for which following alternatives are available :
Issue of 200,000 equity shares at premium of Rs. 25 per share.
Issue of 10% pref. shares.
Issue of 8% debentures.
The company‘s EBIT is at the rate of 12% on its capital employed
which is likely to remain unchanged after expansion.

The Institute of Chartered Accountants of Nepal 56


Suggested Answer- December 2018

It is estimated that P/E ratio in the case of equity shares, preference


shares and debentures financing would be 21.4, 17 and 15.7
respectively.
Income tax rate is 50%.
Required: 5
Which of those alternatives of financing would you recommend and
why?
Answer:
a) Present Cost of Debtors:
At present the cost of debtors is Rs. 1.2325 million as follows:
Present Position Rs. in million Calculation
Sales (a) 12.00
Bad Debts @ 1% of Gross Sales (b) (0.12) (12.00 m*1%)
(12.00 m x 0.75 x
Cost of Investment in Debtors (0.1125) 30/360 x 0.15)
Cost of credit controller (d) (1.00)
Net Cost of Present Debtors
(b+c+d) (1.2325)
Proposed Cost of Debtors:
The proposed cost associated with debtors is Rs. 0.4019 million as follows:
Proposed Position Rs. in million Calculation
Sales (a) 12.00
Sales Increase 25% (b) 3.00
Revised Sales Projections (c) 15.00
Gross Profit Increase(d) 0.75 (3.00 m@25%)
Bad Debts @ 5% Gross Sales (e) (0.75) (15.00 m*5%)
(15.00 m x 0.75 x
Cost of Investment in Debtors (f) (0.4219) 90/360 x 0.15)
Net saving of factoring (g) 0.02 (1m-0.98m)
Net Cost of Proposal (d-e-f+g) (0.4019)
This represents an increase of Rs. 0.8306 million (i.e. 1.2325 – 0.4019)
increase in profitability.
Conclusion: The proposal to relax credit terms and to replace credit
controller with a factoring agency could be implemented, as it will improve
profitability by Rs. 0.8306 million per annum.
b) Statement showing evaluation of financial plan
(Rs. in million)
Particulars Under Proposed financial plans
Existing Plan I (Eq. Plan II (Pref. Plan III
Structure share) shares) (Debentures)
Earnings before 6.00 9.00 9.00 9.00
interest & tax (EBIT)
@ 12%
Less: Interest (Old) 0.56 0.56 0.56 0.56
Less: Interest (New) ---- ---- ---- 2.00

The Institute of Chartered Accountants of Nepal 57


Suggested Answer- December 2018

Profit before tax 5.44 8.44 8.44 6.44


(PBT)
Less: Income tax (at 2.72 4.22 4.22 3.22
50%)
Less: Preference 1.08 1.08 1.08 1.08
dividend (Old)
Less: Preference ---- ---- 2.50 ----
dividend (New)
Profit for equity 1.64 3.14 0.64 2.14
shareholders
Number of equity 200,000 400,000 200,000 200,000
shares
EPS (Rs.) 8.20 7.85 3.20 10.70
P/E Ratio --- 21.4 17.0 15.7
Market price per --- 167.99 54.40 167.99
share
(EPS X P/E Ratio)
Conclusion: From the analysis of above three financial plans, raising of
additional finance by issue of shares and issue of debenture will maximize
the market value of share. But in case of issuing debenture the EPS of the
company would be higher. If the company does not foresee any financial
risk with the issue of debenture, it is suggested to choose financial plan III.
3.
a) ABC Ltd. operates four restaurants in Eastern and Western Region of
Nepal. The manager of each restaurant transfers funds daily from the
local bank to the company‘s principal bank in Kathmandu. There are
approximately 250 business days during a year in which transfers
occur. Several methods of transfer are available. A wire transfer
results in immediate availability of funds, but the local banks charge
Rs. 5 per wire transfer. A transfer through an automatic clearing house
involves next-day settlement, or a 1-day delay, and costs Rs. 3 per
transfer. Finally, a mail-based depository transfer cheque arrangement
costs Rs. 0.30 per transfer, and mailing times result in a 3-day delay
on average for the transfer to occur. This experience is the same for
each restaurant. The company presently uses depository transfer
checks for all transfers. The restaurants have the following daily
average remittance:
Restaurant 1 Restaurant 2 Restaurant 3 Restaurant 4
Rs. 3,000 Rs. 4,600 Rs. 2,700 Rs. 5,200

Required: (4+4=8)
i) If the opportunity cost of funds is 10 percent, which transfer
procedure should be used for each of the restaurants?
ii) If the opportunity cost of funds were 5 percent, what would be the
optimal strategy?

The Institute of Chartered Accountants of Nepal 58


Suggested Answer- December 2018

b) The assets of Sona Ltd. consist of fixed assets and current assets,
while its ratio of other current liabilities and bank credit comprise of
2:1.
Following information are also available:
Share capital Rs. 575,000
Working capital (CA - CL) Rs. 150,000
Gross margin 25%
Inventory turnover 5 times
Average collection period 1.5 months
Current ratio 1.5:1
Quick ratio (Quick assets/ Current liabilities.) 0.8: 1
Reserves & surplus to Bank & cash 4 times
Required: 7
Prepare the balance sheet of the company as on 32nd Ashadh 2075.
Answer:
a)
i) If the opportunity cost of fund is 10%

Restaurant
1 2 3 4
Option I - Wire Transfer
Annual Transfer Cost [ 250 Transfer × Rs. 5] 1,250 1,250 1,250 1,250
Cost of Fund Blocked - - - -
Total Cost under Wire Transfer 1,250 1,250 1,250 1,250
Option II - Automated Clearing House [ ACH]
Annual Transfer Cost [ 250 Transfer × Rs. 3] 750 750 750 750
Cost of Fund Blocked (Avg. remittance × 10% 300 460 270 520
Total Cost under ACH 1,050 1,210 1,020 1,270
Option III - Mail Based Transfer Cheque
Annual Transfer Cost [ 250 Transfer × Rs. 0.3] 75 75 75 75
Cost of Fund Blocked (Avg. remittance×3×10%) 900 1380 810 1560
Total Cost Under Mail based transfer system 975 1,455 885 1,635
Option Option Option Option
Preferred Transfer Method for each Restaurant III II III I

ii) If the opportunity cost of fund is 5%

Restaurant
1 2 3 4
Option I - Wire Transfer
Annual Transfer Cost [ 250 Transfer × Rs 5] 1,250 1,250 1,250 1,250
Cost of Fund Blocked - - - -
Total Cost under Wire Transfer 1,250 1,250 1,250 1,250
Option II - Automated Clearing House [ ACH]

The Institute of Chartered Accountants of Nepal 59


Suggested Answer- December 2018

Annual Transfer Cost [ 250 Transfer × Rs 3] 750 750 750 750


Cost of Fund Blocked (Avg. remittance × 5%) 150 230 135 260
Total Cost under ACH
900 980 885 1,010
Option III - Mail Based Transfer Cheque
Annual Transfer Cost [ 250 Transfer × Rs 0.3]
75 75 75 75
Cost of Fund Blocked (Avg. remittance ×3×5%) 450 690 405 780

Total Cost Under Mail based transfer system


525 765 480 855
Preferred Transfer Method for each Restaurant Option Option Option Option
III III III III
b) Balance Sheet of Sona Ltd. as on 32nd Aahadh, 2075
Liabilities & Capital Amount (Rs.) Assets Amount (Rs.)
Share capital 575,000 Fixed assets (Balancing 685,000
figure)
Reserve & surplus 260,000 Current assets:
Current Liabilities: Inventories 210,000
Bank credit 100,000 Debtors 175,000
Other current liabilities 200,000 Cash & bank balance 65,000
Total 1,135,000 Total 1,135,000
Working Note:
1. Current Ratio = Current Assets/ Current Liabilities
1.5 = CA/CL
CA = 1.5CL
Working Capital = CA – CL
150,000 = 1.5 CL – CL
0.5 CL = 150,000
CL = 150,000/0.5 = Rs. 300,000
CA= 1.5*300,000 = Rs. 450,000
2. Other Current Liabilities (OCL)/ Bank Credit (BC) = 2/1
OCL = 2 BC
Current Liabilities = OCL + BC
300,000 = 2BC + BC
BC = Rs. 100,000
OCL = Rs. 200,000
3. Quick Ratio = Quick Assets/ Current Liabilities
0.80 = (Current Assets -Stock)/CL
0.80 = (450,000 – Stock)/300,000
Stock = 450,000 – 240,000 = Rs. 210,000
4. Inventory Turnover = Cost of Goods Sold (COGS)/ Stock
5 = COGS/210,000
COGS = Rs. 1,050,000
5. Gross Profit = 25% of Sales
Gross Profit = Sales- COGS
0.25 Sales = Sales – COGS

The Institute of Chartered Accountants of Nepal 60


Suggested Answer- December 2018

0.75 Sales = COGS


Sales = 1,050,000/0.75 = Rs. 1,400,000
6. Debtors T.O = 12/ACP
12/1.5 =8
Debtors = Sales/ Debtors turnover = 1,400,000/8 = Rs. 175,000
7. Bank & cash = CA – Debtors – Stock
= 450,000 -175,000 – 210,000
= Rs. 65,000
8. Reserve & surplus/ Bank & cash = 4
Reserve & surplus = 4 ×Bank & cash = 4*65,000 = Rs. 260,000
4.
a) Delta Ltd. currently has an equity share capital of Rs. 1,000,000
consisting of 100,000 Equity share of Rs. 10 each. The company is
going through a major expansion plan requiring raising funds to the
tune of Rs. 600,000. To finance the expansion, the management has
following plans:
Plan-I : Issue 60,000 Equity shares of Rs. 10 each.
Plan-II: Issue 40,000 Equity shares of Rs. 10 each and the
balance through long-term borrowing at 12%
interest p.a.
Plan-III: Issue 30,000 Equity shares of Rs.10 each and 3,000
Rs.100, 9% Debentures.
Plan-IV: Issue 30,000 Equity shares of Rs. 10 each and the
balance through 6% preference shares.
The EBIT of the company is expected to be Rs. 400,000 p.a.
Assume corporate tax rate of 40%.
Required: (6+2=8)
i) Calculate EPS in each of the above plans.
ii) Ascertain the degree of financial leverage in each plan and
suggest the better plan.
b) As an investment manager, you are given the following information
of investments:
Investment in Initial Price Return (Rs.) Market price at Beta
(Rs.) year end (Rs.)
Cement Ltd. 25 2 (Dividend) 50 0.80
(Equity share)
Steel Ltd. (Equity 35 2 (Dividend) 60 0.70
share)
Liquor Ltd. 45 2 (Dividend) 135 0.50
(Equity share)
GON Bond. 1,000 140 (Interest) 1,005 0.99
Risk free return may be taken at 14%.
Required: (5+2=7)
i) Calculate expected rate of returns of portfolio by using CAPM.
ii) Calculate average return of the portfolio.

The Institute of Chartered Accountants of Nepal 61


Suggested Answer- December 2018

Answer:
a)
i) Computation of EPS and Financial Leverage
Plan I Plan II Plan III Plan IV
Present Equity Shares 100,000 100,000 100,000 100,000
New Issue 60,000 40,000 30,000 30,000
No. of total equity shares 160,000 140,000 130,000 130,000
Equity share capital (Rs.) 1,600,000 1,400,000 1,300,000 1,300,000
12% Long term loan (Rs.)  200,000  
9% Debentures (Rs.)   300,000 
6% Preference shares (Rs.)    300,000
EBIT (Rs.) 400,000 400,000 400,000 400,000
Interest on 12% Loan (Rs.)  24,000  
Interest on 9% debentures (Rs.)   27,000 
EBT (Rs.) 400,000 376,000 373,000 400,000
Less : Tax@ 40% 160,000 150,400 149,200 160,000
EAT (Rs.) 240,000 225,600 223,800 240,000
Less: Preference Dividends (Rs.)    18,000
(a) Earnings for equity shares (Rs.) 240,000 225,600 223,800 222,000
(b) No. of equity shares 160,000 140,000 130,000 130,000
(c) EPS (a  b) Rs. 1.50 1.61 1.72 1.71
ii) Computation of Financial leverage
Degree of Financial leverage-
 EBIT   EBIT  1.00 1.06 1.07 1.08
  
 EBIT  I   EBT  Or (WN. 1)
EBIT
EBT- PD
(1-t)

Comments: Since the EPS and degree of financial leverage both are highest
in Plan III, the management could accept it.
W.N 1
400000
400000-18,000 = 1.08
(1-0.40)

b) In the given case, market return is not given. Hence, we should calculate the
market return assuming the given securities represent the market as follows:

Investment in Market Price


Initial Price (Rs.) Return (Rs.) at year end (Rs.)
Cement Ltd 25 2 50
Steel Ltd 35 2 60
Liquor Ltd 45 2 135
GON Bond 1,000 140 1,005
Total 1,105 146 1,250

The Institute of Chartered Accountants of Nepal 62


Suggested Answer- December 2018

Market Return (Rm) = Closing price + Return – Opening price


Opening price
= (1250+146-1105)/1105
= 0.2633 i.e. 26.33%
Now,
i) Expected Return on Individual security
Under CAPM, expected return = Rf + (Rm – Rf)β
Cement Ltd 14% +0.8 (26.33%-14%) = 23.86%
Steel Ltd 14% +0.7 (26.33%-14%) = 22.63%
Liquor Ltd 14% +0.5 (26.33%-14%) = 20.16%
GON Bonds 14% +0.99 (26.33%-14%) = 26.21%
ii) Average Return of portfolio
= 23.86%×25/1105 + 22.63%×35/1105 + 20.16%×45/1105 + 26.21%×1,000/1105
= 25.80%
5.
a) SE Ltd., having an equity capital of Rs. 1 billion (face value of Rs.
100 each) earned a net profit of Rs. 150 million during the financial
year 2074/75 and is planning to make new investments of Rs. 600
million during the current financial year 2075/76. In line with the
company's dividend policy, it has a plan of declaring dividend of Rs.
10 per share at the end of current financial year. The firm's
opportunity cost of capital is 15% and is currently traded at Rs. 100
per share.
Required: (3+2=5)
i) Considering the Modigliani and Miller's Dividend Irrelevance
Theorem, what is the price of the share at the end of the current
financial year if:
 the dividend is not declared,
 the dividend is declared.
ii) How many new shares must be issued to finance its investments
assuming that dividend is declared?
b) Suman inherited the following securities on his father‘s death:
Types of security Nos. Annual Maturity Yield %
Coupon % years
Bond A (Par value Rs. 1,000) 20 8 4 10
Bond B (Par value Rs. 1,000) 25 10 5 10
Preference Shares C (Par value 150 11 - 12
Rs. 100)
Preferences Shares D (Par 200 12 - 15
value Rs. 100)

Required: 5
Compute the current value of Suman‘s Investment.

The Institute of Chartered Accountants of Nepal 63


Suggested Answer- December 2018

c) One of your clients has seen many references to the 'Cost of Capital'
in the proposal for lending from banks and has asked you to give him
some guidance on what would be an appropriate figure for his
organization- Crisjan Ltd. The following information is available for
Crisjan Ltd.:

Existing capital structure: Rs.


Issued ordinary shares (120,000) 12,000,000
Retained earnings 4,000,000
6% Preferences shares (20,000) 2,000,000
9% Debenture repayable on 2076
(Par value Rs. 1,000) 6,000,000
9% Debenture was issued in 2075 at par. Its current price is Rs. 920.
A similar issue if made now would require being at Rs. 900.
Preference shares have a par value of Rs. 100 and were originally
issued at Rs. 92 per share. Its current price is Rs. 43. A similar issue
if made now would require to be at Rs. 40 per share.
The market price of an ordinary share is Rs. 700.
Rs. 6 million in dividends was paid this year which represented 75%
of earnings.
Earnings are expected to grow at an annual rate of 5%.
If new ordinary shares were issued now, costs incurred would
represent Rs. 50 per share and a reduction below market value of Rs.
25 per share would also be made.
Corporate tax rate is 25%.
Required: 5
Calculate Crisjan Ltd.'s Weighted Average Cost of Capital.
Answer:
a)
i) Price of the share at the end of the current financial year if the
dividend is not paid
P1= P0 (1 + k) - DIV1
= 100 × (1+0.15) - 0
= Rs. 115
Price of the share at the end of the current financial year if the dividend
is paid
P1= P0 (1 + k) - DIV1
= 100 × (1+0.15) - 10
= Rs. 105
Where
P1= Price at the end of the financial year
P0= Current Price
k= Cost of Capital
DIV1= Dividend at the end of the financial year
ii) The value of share issued for investment is Rs.105 per share (FV

The Institute of Chartered Accountants of Nepal 64


Suggested Answer- December 2018

and share premium). Therefore, no. of shares to be issued:


mP1 = I - (X - nDIV1)
105m = 600 – [150– (10 × 10)]
105m = 550
m = 550 million/105 = 5,238,095 shares
Where
m = Required No. of New Shares
n = Existing No. of Shares
I = Investment Amount
X = Net Profit Attributable to Shareholders
DIV1 = Dividend at the end of the financial year
b) Value of Bond A = Interest (PVIFA 10%, 4 Years) + RV(PVIF 10%, 4th Year)
= 80*3.169+ 1000*0.683
= Rs. 936.52
Value of Bond B = Interest (PVIFA 10%, 5 Years) + RV(PVIF 10%, 5th Year)
= 100*3.79+ 1000*0.621
= Rs. 1,000
(Alternate: Since the Yield of the bond is equal to the coupon rate of the
bond, the fair value of the bond will be equal to the face value of the bond.)

Value of Preference Share C =


= Rs. 11/12% = Rs. 91.67

Value of Preference Share D =


= Rs. 12/15% = Rs. 80
Calculation of Value of Investment of Suman:
Types of Security Number Value per Security Total Value
Bond A 20 936.52 18,730.04
Bond B 25 1,000 25,000
Preference Shares C 150 91.67 13,750.50
Preferences Shares D 200 80 16,000
Total value of Portfolio 73,480.54

c) Marginal cost (after tax) of debenture


Kd = AI(1-t) + Co- CI
n
Co+CI
2
= 90(1-0.25) + 1,000- 900
1
1000+900
2

The Institute of Chartered Accountants of Nepal 65


Suggested Answer- December 2018

= 17.63%
Marginal cost of Preference shares
=120,000
800,000
= 0.15
= 15%
Marginal cost of Ordinary shares
= [ 50(1+0.05)/(700-50-25)]+0.05
= 52.5/625 +0.05
= 13.4%
Weighted Average Cost of Capital:
Capital
Structure
(MV) (in Weigh Component Weight
Source of Finance million) t cost(%) ed Cost
Ordinary shares
84 13.40 12.48
120,000×700 0.9312
5.4 17.63 1.06
Debentures (9@6m) 0.0599
Preference shares
0.8 15 0.13
(4@2m) 0.0089
Total 90.20 1 13.66

OR Alt Solution
Calculation of WACC(Book Value)
Sources of Finance Amount(BV) Weight(a)
Cost(b) (a)×(b)
Ordinary share 120,00,000 0.50
13.40%(WN3) 6.70%
Retained Earnings 40,00,000 0.17
12.50%(WN 4) 2.125%
6% Preference Share 20,00,000 0.08
15%(WN 2) 1.20%
9% Debenture 60,00,000 0.25
17.63%(WN 1) 4.408%
240,00,000 1.00 WACC 14.43%
Working Notes:
(1) Calculation of cost of Debt(kd)
Kd = AI(1-t) + Co- CI
n
Co+CI
2
= 90(1-0.25) + 1,000- 900
1

The Institute of Chartered Accountants of Nepal 66


Suggested Answer- December 2018

1000+900
2
= 17.63%
(2) Calculation of Cost of preference share(KP)
Kp = Pref Dividend ×100%
Net Proceeds
= 6/40×100%
= 15%
(3) Calculation of cost of Equity(ke) as per Gordon's Growth,
Ke = D1/NP + g
= 50(1+5%) + 0.05
700 -50-25
= 13.40%
(4) Calculation of cost of Retained Earnings(Kre)
Kre = D1/Po + g
= 50(1+5%) + 0.05
700
= 12.50%
6. Write short note/ answer on: (4×2.5=10)
a) Debt securitization
b) Virtual banking and its advantage
c) Four kind of floats with reference to cash management
d) Activities covered by Treasury Management
Answer:
a) Securitization is a process in which illiquid assets are pooled into
marketable securities that can be sold to investors. The process leads to the
creation of financial instruments that represent ownership interest in, or are
secured by a segregated income producing asset or pool of assets. These
assets are generally secured by personal or real property such as
automobiles, real estate, or equipment loans but in some cases are
unsecured.
For example, a finance company has issued a large number of car loans. It
desires to raise further cash so as to be in a position to issue more loans. One
way to achieve this goal is by selling all the existing loans, however, in the
absence of a liquid secondary market for individual car loans, this may not
be feasible. Instead, the company pools a large number of these loans and
sells interest in the pool to investors. This process helps the company to
raise finances and get the loans off its Balance Sheet. These finances shall
help the company disburse further loans.
b) Virtual banking refers to the provision of banking and related services
through the use of information technology without direct recourse to the

The Institute of Chartered Accountants of Nepal 67


Suggested Answer- December 2018

bank by the customer. The advantages of virtual banking services are as


follows:
 Lower cost of handling a transaction.
 The increased speed of response to customer requirements.
 The lower cost of operating branch network along with reduced staff
costs leads to cost efficiency.
 Virtual banking allows the possibility of improved and a range of
services being made available to the customer rapidly, accurately and at
his convenience.
c) Four Kinds of Float with reference to Management of Cash are as follows:
 Billing Float: The time between the sale and the mailing of the invoice is
the billing float.
 Mail Float: This is the time when a cheque is being processed by post
office, messenger service or other means of delivery.
 Cheque processing float: This is the time required for the seller to sort,
record and deposit the cheque after it has been received by the company.
 Bank processing float: This is the time from the deposit of the cheque to
the crediting of funds in the seller‘s account.
d) Treasury Management is concerned about the efficient management of
liquidity and financial risk in business. Main activities which are covered by
Treasury Management are as follows:
(i) Cash management: Treasury management in a business organisation
is concerned about the efficient collection and repayment of cash to
both insiders and to third parties.
(ii) Currency management: It manages the foreign currency risk,
exchange rate risks etc. It may advise on the currency to be used
when invoicing overseas sales.
(iii) Funding management: Responsible for planning and sourcing firm‘s
short, medium and long term cash needs. It participates in capital
structure, forecasting of future interest and foreign currency rates
decision-making process.
(iv) Banking: Maintains good relations with bankers and carry out initial
negotiations with them for any short term loan.
(v) Corporate finance: It advises on aspects of corporate finance
including capital structure, mergers and acquisitions.
7. Distinguish between: (4×2.5=10)
a) Venture capital and Private equity
b) Business risk and Financial risk
c) Money market and Capital market
d) Financial distress and Insolvency
Answer:
a) Venture capital is a risk capital and is regarded as a subset of private equity.
It is generally raised to specifically finance startups and small and medium
sized private companies with strong growth potential whereas the private
equity also has component of development capital and buyout capital that is
targeted to mature businesses that are already established.
Development capital refers to the pool of capital raised to finance for
expansion of businesses while buy-out capital refers to pool of fund raised

The Institute of Chartered Accountants of Nepal 68


Suggested Answer- December 2018

through investors and management team, usually together with borrowed


money, to buy a business from its current owners.
Effectively, therefore the private equity consists of venture capital,
development capital and in certain cases buyout capital (when buyouts are
not adequately financed by banks).
The private equity is generally held in private companies or such public
companies that are not traded in stock exchanges.
b) Business Risk refers to the risk associated with the firm‘s operations. It is
the uncertainty about the future operating income (EBIT), i.e. how well can
the operating incomes be predicted. Business risk can be measured by the
standard deviation of the Basic Earning Power Ratio.
Financial Risk refers to the additional risk placed on the firm‘s shareholders
as a result of debt use i.e. the additional risk a shareholder bears when a
company uses debt in addition to equity financing. Companies that issue
more debt instruments would have higher financial risk than companies
financed mostly or entirely by equity.
c) Some of the points of distinction between money market and capital market are as
follows:
Money Market Capital Market
(i) There is no classification There is a classification between primary
between primary market and market and secondary market.
secondary market.
(ii) It deals for funds of short-term It deals with funds of long-term
requirement. requirement.
(iii) Money market instruments Capital Market instruments are shares and
include interbank call money, debt instruments.
notice money up to 14 days,
short-term deposits up to three
months, commercial paper,
treasury bills.
(iv) Money market participants are Capital market participants include retail
banks, financial institution, NRB investors, institutional investors; like
and Government Mutual Funds, Financial Institutions,
Corporate and Banks.
d) Financial Distress is surprisingly hard to define precisely. This is true partly
because of the variety of events befalling firms under financial distress. The
list of events is almost endless, but examples are Dividend Reductions, Plant
Closings, Losses, Layoffs, CEO resignations, Plummeting stock prices and
many more.
Financial distress is a situation where a firm‘s operating cash flows are not
sufficient to satisfy current obligations (such as trade credits or interest
expenses) and the firm is forced to take corrective action. Financial distress
may lead a firm to default on a contract, and it may involve financial
restructuring between the firm, its creditors, and its equity investors. Usually
the firm is forced to take actions that it would not have taken if it had
sufficient cash flow.
Insolvency occurs when an individual or a firm is unable to meet their
financial obligations. Accounting (Balance sheet) insolvency happens when
total liabilities exceed total assets (negative net worth).

The Institute of Chartered Accountants of Nepal 69


Suggested Answer- December 2018

Cash flow insolvency involves a lack of liquidity to pay debts as they fall
due. A business can be cash-flow insolvent but balance-sheet solvent if it
holds market liquidity assets, particularly against short term debt that it
cannot immediately realize if called upon to do so. Conversely, a business
can have negative net assets showing on its balance sheet, making it
balance-sheet insolvent, but still be cash-flow solvent if ongoing revenue is
able to meet debt obligations, and thus avoid default: for instance, if it holds
long term debt. Some large companies operate permanently in this state.

The Institute of Chartered Accountants of Nepal 70


Suggested Answer- December 2018

Specific Comments on the performance of the students


Batch: - December 2018
Level: - CAP-II
Subject: Financial Management
Question No. 1
Many students did not attempt the question properly. Students were not having
knowledge of adjusted NPV. Presentation was very poor. No any students calculated
the NPV properly. Most of the students fail to understand the question. Lack of
conceptual knowledge on capital budgeting.
Question No. 2
(a) Only few students attempted the question properly. Overall performance was not
good. Major students did not calculate the saving while accepting both the
proposals.
(b) Almost all students attempted the question correctly. However, the decision
making part was not attempted correctly. Some students fail to calculate EBIT of
each proposal.
Question No. 3
(a) Only few students attempted the question properly. Overall performance was not
good. Students were not having conceptual clarity. Most of the students did not
understand the question properly.
(b) Many students attempted the question correctly. Few students were having
confusion while calculating Inventory turnover ratio. Students fail to calculate BC
& OCL.
Question No. 4
(a) Many students attempted the question correctly. Few students did not calculate the
financial leverage correctly in case of preference shares. Students fail to calculate
the EPS properly as well as DOF in case of plan IV.
(b) Performance was average. Few students did not attempt the question. Students fail
to calculate the average return of portfolio.
Question No. 5
(a) Almost all students attempted the question correctly. Students fail to calculate the
number of new shares.
(b) Many students attempted the question correctly. However, few students forget to
calculate the overall value of Investment. Some students miscalculated the value.
(c) Many students attempted the question. But were not clear about the calculation of
cost of each component. Most students fail to calculate the cost of capital
properly.
Question No. 6
Overall performance was not good for all sub-section. Students lack of conceptual
knowledge.
Question No. 7
Many students attempted the question without having proper knowledge of the topic.
Students fail to differentiate properly. Lack of conceptual knowledge.

The Institute of Chartered Accountants of Nepal 71


Suggested Answer- December 2018

Paper 5:

Cost & Management Accounting

The Institute of Chartered Accountants of Nepal 72


Suggested Answer- December 2018

Cost & Management Accounting


Suggested
Roll No……………. Maximum Marks - 100
Total No. of Questions: 6 Total No. of Printed Pages - 13
Time Allowed - 3 Hours

Marks
All questions are compulsory. Working notes should form part of the
answer.
Make assumptions wherever necessary.
1. Nepal Milk Chocolates manufactures and distributes chocolate products.
It purchases Cocoa beans and processes them into two intermediate
products:
Chocolate powder liquor base
Milk chocolate liquor base
These two intermediate products become separately identifiable at a
single split off point. Every 500 pounds of cocoa beans yields 20 gallons
of chocolate-powder liquor base and 30 gallons of milk-chocolate liquor
base.
The chocolate powder liquor base is further processed into chocolate
powder. Every 20 gallons of chocolate-powder liquor base yields 200
pounds of chocolate powder. The milk chocolate liquor base is further
processed into milk-chocolate. Every 30 gallons of milk chocolate liquor
base yields 340 pounds of milk chocolate.
Production and sales data for October, 2018 are:
Cocoa beans processed 7,500 pounds
Costs of processing Cocoa beans to split off point Rs. 712,500
(Including purchase of beans)
Production Sales Selling price
Chocolate powder 3,000 pounds 3,000 pounds Rs. 190 per pound
Milk chocolate 5,100 Pounds 5,100 Pounds Rs. 237.50 per pound
The separable costs of processing chocolate powder liquor into chocolate
powder are Rs. 302,812.50 for the month of October, 2018. The separable
costs of processing milk chocolate liquor base into milk chocolate are Rs.
623,437.50 for the month of October 2018.
Nepal Milk processes both of its intermediate products into chocolate
powder or milk chocolate in full. There is an active market for these
intermediate products. In October, 2018, Nepal Milk could have sold the
chocolate powder liquor base for Rs. 997.50 a gallon and the milk-
chocolate liquor base for Rs. 1,235 a gallon.
Required: 20
i) Calculate how the joint cost of Rs. 712,500 would be allocated
between the chocolate powder and milk-chocolate liquor bases under
the following methods:

The Institute of Chartered Accountants of Nepal 73


Suggested Answer- December 2018

a) Sales value at split off point


b) Physical measure (gallons)
c) Estimated net realisable value (NRV) and
d) Constant gross-margin percentage NRV.
ii) What is the gross-margin percentage of the chocolate powder and
milk-chocolate liquor bases under each of the methods in
requirements (i) above?
iii) Could Nepal Milk have increased its operating income by a change in
its decision to fully process both of its intermediate products? Show
your computations.

Answer:
i) Comparison of alternative Joint-Cost Allocation Methods:
a) Sales Value at Split-off Point Method
Chocolate Milk Total
powder liquor base chocolate liquor base

Sales value of
products at Rs. 2,99,250* Rs. 5,55,750** Rs 8,55,000
split off
Weights 0.35 0.65 1.00
Joint cost
Rs. 2,49,375 Rs. 4,63,125 Rs. 7,12,500
allocated
(Rs. 7,12,500 × 0.35) (Rs. 7,12,500 × 0.65)

*(3,000 lbs ÷ 200 lbs) × 20 gallon × Rs. 997.50 = Rs. 2,99,250


** (5,100 lbs ÷ 340 lbs) × 30 gallon × Rs. 1,235 = Rs. 5,55,750
b) Physical Measure Method
Chocolate powder Milk chocolate liquor Total
liquor base base
Output 300 gallon* 450 gallon** 750 gallons
Weight 300/750 = 0.40 450/750 = 0.60 1.00
Joint cost
Rs. 2,85,000 Rs. 4,27,500 Rs. 7,12,500
allocated
(Rs. 7,12,500 x (Rs. 7,12,500 x 0.60)
0.40)
*(3,000 lbs ÷ 200 lbs) × 20 gallon = 300 gallon
** (5,100 lbs ÷ 340 lbs) × 30 gallon = 450 gallon
c) Net Realisable Value (NRV) Method
Chocolate Total
Milk chocolate
powder liquor
liquor base
base

The Institute of Chartered Accountants of Nepal 74


Suggested Answer- December 2018

Final sales value of Rs. 5,70,000 Rs. 12,11,250 Rs. 17,81,250


production (3,000 lbs × Rs. (5,100 lbs × Rs.
190) 237.50)
Less: Separable
Rs. 3,02,812.50 Rs. 6,23,437.50 Rs. 9,26,250
costs
Net value at Rs. 2,67,187.50 Rs. 5,87,812.50 Rs. 8,55,000
realisable
split off
point
Weight 0.3125 0.6875 1.00
(2,67,187.50 ÷ (5,87,812.5 ÷
8,55,000) 8,55,000)
Joint cost allocated Rs. 2,22,656.25 Rs. 4,89,843.75 Rs. 7,12,500
(Rs. 7,12,500 x (Rs. 7,12,500 x
0.3125) 0.6875)

d) Constant Gross Margin( %) NRV method


Chocolate Milk chocolate Total
powder Liquor liquor Base
base
Final sales
value of Rs. 5,70,000 Rs. 12,11,250 Rs. 17,81,250
production
Less:
Gross
Rs. 45,600 Rs. 96,900 Rs. 1,42,500
margin*
8%
Cost of
goods
Rs. 5,24,400 Rs. 11,14,350 Rs. 16,38,750
available
for sale
Less:
Separable Rs. 3,02,812.50 Rs. 6,23,437.50 Rs. 9,26,250
costs
Joint cost
Rs. 2,21,587.50 Rs. 4,90,912.50 Rs. 7,12,500
allocated
*Final sales value of total production = Rs. 17,81,250
Less: Joint and separable cost = Rs. 16,38,750 (Rs. 7,12,500 + Rs. 9,26,250)
Gross Margin = Rs. 1,42,500
Gross margin (%) = 1,42,500/17,81,250× 100 = 8%
ii) Chocolate powder liquor base
(Amount in Rs.)
Sales value at Physical Estimated net Constant
Split off Measure Realisable Value Gross Margin NRV

Final sale
value of 5,70,000 5,70,000 5,70,000 5,70,000
Chocolate

The Institute of Chartered Accountants of Nepal 75


Suggested Answer- December 2018

Powder
Less:
Separable 3,02,812.50 3,02,812.50 3,02,812.50 3,02,812.50
costs
Less: Joint
2,49,375 2,85,000 2,22,656.25 2,21,587.50
costs
Gross Margin 17,812.50 (17,812.50) 44,531.25 45,600
Gross Margin
3.125% (3.125%) 7.8125% 8.00%
%
Milk chocolate liquor base (Amount in
Rs. )
Sales value at Physical Estimated net Constant
split off measure realisable Gross margin
NRV
Final sale value of milk
12,11,250 12,11,250 12,11,250 12,11,250
chocolate
Less: Separable costs 6,23,437.50 6,23,437.50 6,23,437.50 6,23,437.50
Less: Joint costs 4,63,125 4,27,500 4,89,843.75 4,90,912.5
Gross Margin 1,24,687.50 1,60,312.50 97,968.75 96,900
Gross Margin % 10.29% 13.24% 8.09% 8.00%
iii) Further processing of Chocolate powder liquor base into Chocolate
powder
(Amount in Rs. )
Incremental revenue {Rs. 5,70,000 – (Rs. 997.50 x 300
2,70,750
gallon)}
Less: Incremental costs 3,02,812.50
Incremental operating income (32,062.50)
Further processing of Milk Chocolate liquor base into Milk Chocolate.
(Amount in Rs.)
Incremental revenue {Rs. 12,11,250 – (Rs. 1,235 x 450
6,55,500
gallon)}
Less: Incremental cost 6,23,437.50
Incremental operating income 32,062.50
The above computations show that Nepal Milk Chocolates could increase
operating income by Rs. 32,062.50 if chocolate liquor base is sold at split off
point and milk chocolate liquor base is processed further.
2.
a) A company which works at a capacity utilization of 60% expects its
turnover for the year 2017/18 at Rs. 86.40 lakhs. If the company
works at 100% capacity, the sales-cost relationship will be as follows:
Factory cost: Two-third of sales value
Prime cost: 75% of factory cost
Selling and administrative expenses (75% variable): 20% of sales value.
The factory overheads will vary according to the operating capacity in
the following manners:

The Institute of Chartered Accountants of Nepal 76


Suggested Answer- December 2018

Operating Capacity 60% 80% 100% 120%


Factory Overheads (Rs. in lakhs) 19.80 21.60 24.00 30.00
The company receives an offer from abroad for a value of Rs. 19.80
lakhs. The prime cost of this order is estimated at Rs. 12 lakhs and the
selling and administration expenses applicable to this order are Rs.
90,000. This order will occupy 40% of the capacity of the plant.
The Marketing Director estimates that by the time the new order
materializes, the company‘s own sales will increase to 80% of the
capacity.
Required: 10
Prepare statements to show:
i) Profitability at the present capacity utilization of 60%.
ii) Profitability based on Marketing Director‘s estimate of increases
in the company‘s own sale to 80% capacity.
iii) Evaluation of the export order with advice as to whether the
company should accept the export order or not.
b) Muktishree Limited operates a system of standard costing in respect
of one of its products which is manufactured within a single cost
centre. The Standard Cost Card of a product is as under:
Standard Unit cost (Rs.)
Direct material 5 kgs @ Rs. 4.20 21.00
Direct labour 3 hours @ Rs. 3.00 9.00
Factory overhead Rs. 1.20 per labour hour 3.60
Total manufacturing cost 33.60
The production schedule for the month of March, 2018 required
completion of 40,000 units. However, 40,960 units were completed
during the month without opening and closing work-in process
inventories.
Purchases during the month of March, 2018, 225,000 kgs of material
at the rate of Rs. 4.50 per kg. Production and Sales records for the
month showed the following actual results.
Material used 205,600 kgs.
Direct labour 121,200 hours; cost incurred Rs. 387,840
Total factory overhead cost incurred Rs. 100,000
Sales 40,000 units
Selling price to be so fixed as to allow a mark-up of 20 per cent on
selling price.
Required: 10
i) Calculate material variances based on consumption of material.
ii) Calculate labour variances and the total variance for factory
overhead.
iii) An incentive scheme is in operation in the company whereby
employees are paid a bonus of 50% of direct labour hour saved at
standard direct labour hour rate. Calculate the Bonus amount.

The Institute of Chartered Accountants of Nepal 77


Suggested Answer- December 2018

Answer:
a)

Statement of Profitability (Rs. In lakhs)


Capacity
100% 60% 80% 40%(Export order)
Sales (a) 144 86.4 115.2 19.8
Prime Cost 72 43.2 57.6 12
Factory Overheads 24 19.8 21.6 8.4
Factory Cost 96 63 79.2 20.4
Selling and Admin. Costs 28.8 20.16 24.48 0.9
Cost of Sales (b) 124.8 83.16 103.68 21.3
Profit(Loss) (a-b) 19.2 3.24 11.52 -1.5
The above analysis shows that export order instead of increasing the profit is
giving a loss. Hence, it should not be accepted.
WORKING NOTES:
a) Computation of Cost and Sales at 100% Capacity
(Rs. In lakhs)
Sales at 60% 86.4
Sales at 100% = 86.40 / 0.6 144
Factory Cost 2/3 of Sales = 144*2/3 96
Factory Overheads (Given) 24
Prime Cost 72
Selling and Administrative expenses = 144*20/100 28.8
b) Computation of Cost and Sales at 60% Capacity
(Rs. In lakhs)
Sales 86.4
Prime Cost = 72 * 60/100 43.2
Selling and administrative costs:
Variable : 28.8 *0.75 * 0.6 12.96
Fixed: 28.8 * 0.25 7.2
c) Computation of Cost and Sales at 80% Capacity
(Rs. In lakhs)
Sales (144 * 0.8) 115.2
Prime Cost = 72 * 80/100 57.6
Selling and administrative costs:
Variable : 28.8 *0.75 * 0.8 17.28
Fixed: 28.8 * 0.25 7.2

d) Computation of Factory Overheads for additional 40% capacity


(Rs. In lakhs)
Factory Overheads : at 120% 30

The Institute of Chartered Accountants of Nepal 78


Suggested Answer- December 2018

Less: Factory Over Head at 80% Capacity 21.6


Factory Over Head at 40% Capacity 8.4
b)
i) Material variances:
(a) Direct material cost variance = Standard cost - Actual cost
= 40,960 x 21 - 2,05,600 x 4.50
= 8,60,160 - 9,25,200 = Rs. 65,040 (A)
(b) Material price variance = AQ (SP - AP)
= 2,05,600 (4.20 - 4.50) = Rs. 61,680 (A)
(c) Material usages variance = SP (SQ - AQ)
= 4.20 (40,960 x 5 - 2,05,600) = Rs. 3,360 (A)
ii) Labour variances and overhead variances:
(a) Labour cost variance = Standard cost -Actual cost
= 40,960 x 9 - 3,87,840 = Rs. 19,200 (A)
(b) Labour rate variance = AH (SR - AR)
=1,21,200 (3 - 3.20) = Rs. 24,240 (A)
(c) Labour efficiency variance = SR (SH -AH)
= 3 (40,960 x 3 - 1,21,200) = Rs. 5,040 (F)
(d) Total factory overhead variance
= Factory overhead absorbed - factory
overhead incurred
= 40,960 x 3 x 1.20 – 1,00,000 = Rs. 47,456 (F)
iii) Bonus Amount:

Labour hour saved Hours


Standard labour hours 40,960 x 3 1,22,880
Actual labour hour worked 1,21,200
Labour hour saved 1,680
Bonus for saved labour = 0.50 (1,680 x 3) = Rs. 2,520.
3.
a) 'Jharna View Lodging' home is being run in a small hill station of
Bhedetar of Dhankuta District with 50 single rooms. The home offers
concessional rates during six off- season months in a year. During this
period, half of the full room rent is charged. The management‘s profit
margin is targeted at 20% of the room rent. The following are the cost
estimates and other details for the year ending on 31st March 2018.
[Assume a month to be of 30 days].
i) Occupancy during the season is 80% while in the off- season it is
40% only.
ii) Expenses:
 Staff salary [Excluding room attendants] Rs. 3,55,000
 Repairs to building Rs. 1,30,500
 Laundry and linen Rs. 45,000
 Interior decoration Rs. 1,05,500

The Institute of Chartered Accountants of Nepal 79


Suggested Answer- December 2018

 Sundry expenses Rs. 95,400


iii) Annual depreciation is to be provided for buildings @ 5% and on
furniture and equipments @ 15% on straight-line basis.
iv) Room attendants are paid Rs. 5 per room day on the basis of
occupancy of the rooms in a month.
v) Monthly lighting charges are Rs. 120 per room, except in four
months in winter (off-season) when it is Rs. 30 per room and this
cost is on the basis of full occupancy for a month.
vi) Total investment in the home is Rs. 100 lakhs of which Rs. 80
lakhs relate to buildings and balance for furniture and equipment.
You are required to work out the room rent chargeable per day both
during the season and the off-season months on the basis of the
foregoing information. 8
b) The following figures have been extracted from the financial accounts
of V-Wash ltd. for the first year of its operations:
Direct Material consumption Rs. 50,000
Productive wages 30,000
Factory overheads 16,000
Administrative overheads 7,000
Selling & distribution overheads 9,600
Bad debt written off 800
Preliminary expenses written off 400
Legal charges 100
Dividend received 1,000
Interest received on bank deposits 200
Sales (12,000 units) 120,000
Closing stock:
Finished goods (400 units) 3,200
Work in progress 2,400
The cost account for the same period reveals that direct material
consumption was Rs. 56,000. Factory overhead is recovered at 20%
on prime cost. Administration overhead is recovered at 60 paisa per
unit of production. Selling & distribution overheads at 80 paisa per
unit sold.
Prepare: 8
Profit & Loss Accounts both as per financial records and as per cost
records. Also reconcile the profits as per the two records.
c) What is CVP Analysis? Explain how CVP based Sensitivity Analysis
can help Managers cope with uncertainty. 4
Answer:
a)
i) Computation of Estimated Fixed Cost for the year ending 31st
March, 2018
Particulars Amount (Rs.)
Salary 3,55,000
Repairs 1,30,500
Laundry and linen 45,000
Interior decoration 1,05,500

The Institute of Chartered Accountants of Nepal 80


Suggested Answer- December 2018

Depreciation:
5% on Rs. 80 lakhs: Rs. 4,00,000
15% on Rs. 20 lakhs: Rs. 3,00,000 7,00,000
Sundry expenses 95,400
Total Fixed Cost 14,31,400
ii) Number of room days in a year:
Occupancy during season for 6 months @ 80% = (50 x 0.80 x 6 x 30) = 7,200
Off-season occupancy for 6 months @ 40% = (50 x 0.40 x 6 x 30) = 3,600
Total number of room days during a year = 10,800
iii) Attendant‘s salary For 10,800 room days @ Rs. 5 per day = Rs.
54,000
iv) Light charges for 8 months
@ Rs. 120 per month per occupied room, i.e. Rs. 120/30 = Rs. 4 per room day.
Light charges for 4 months
@ Rs. 30 per month per occupied room, i.e. Rs. 30/30 = Rs. 1 per room day
Total lighting charges:
During season 6 months @ Rs. 4 for 7,200 room days = Rs.
28,800
During off season 2 months @ Rs. 4 for 1,200 days (2/6 x 3,600) = Rs.
4,800
During off season 4 months winter @ Re. 1 for 2,400 days (4/6 x 3600) = Rs.
2,400
Rs.
36,000
v)
Statement of Total Estimated Cost
Particulars Amount (Rs.)
Total Fixed Expenses as shown in (i) above (a) 14,31,400
Variable Running Cost (b)
Attendant‘s salary as shown in (iii)above 54,000
Lighting charges as shown in (iv) above 36,000
9000
Total cost (a+b) 15,21,400
vi) Computation of total Full Room Days
During season : 7,200
Off-season : 1,800 (Equivalent to 50% rate of 3,600 days)
Total Full Room Days : 9,000
Computation of Room Rent
Cost per room day : Rs. 15,21,400 / 9,000 = Rs.169.04
Add: Profit margin at 20% of rent or 25% of cost = Rs. 42.26
Room Rent = Rs. 211.30
Therefore, during season, room rent of Rs. 211.30 is to be charged while in
the off-season room rent of Rs.105.65 (Rs. 211.30/2) is to be charged.
b)
Financial records
Profits and loss account
Dr Cr
Particulars Amount Particulars Amount

The Institute of Chartered Accountants of Nepal 81


Suggested Answer- December 2018

To materials 50,000 By sales (12,000 units) 1,20,000


To wages 30,000 By closing stock of F.G.(400 3,200
units)
To factory overheads 16,000 By W.I.P. 2,400
To gross profit 29,600
1,25,600 1,25,600
To admn. Overheads 7,000 By gross profit 29,600
To S & D overheads 9,600 By dividend 1,000
To bad debts 800 By interest received 200
To prel. Expenses w/o 400
To legal charges 100
To net profit 12,900
30,800 30,800
Cost records
Statement showing cost of sales and profit Rs.
Direct materials
56,000
Direct wages
30,000
Prime cost
86,000
Works overhead (20% on prime cost)
17,200

1,03,200
Less: closing works in progress
24,00
Factory cost
1,00,800
Administration overhead (12,400X0.60)
7,440
Cost of production
1,08,240
Less: finished goods (closing) (1,08,240/12,400X400)
3,492
Cost of production of goods sold
1,04,748
Selling overheads (12,000 X 0.80)
9,600
Cost of sales (12,000 units)
1,14,348
Profit as per cost accounts
5,652
Sales (12,000 units)
1,20,000
Reconciliation statement
Profit as per financial records Rs.
12,900

The Institute of Chartered Accountants of Nepal 82


Suggested Answer- December 2018

Add: difference in valuation of closing stock (Rs. 3,492-3200)


292
Expenses not recorded in cost accounts :
Bad debts 800
Preliminary expenses 400
Legal charges 100
1,300

14.492
Less: Materials over valued in the cost accounts 6,000
Works overheads over-absorbed in cost accounts 1,200
Administration overheads over-absorbed 440
Non-operating income 1,200
8,840
Profit as per cost accounts
5,652
c) Cost-Volume-Profit Analysis (CVP Analysis) is the analysis of three
variables, viz. Cost, Volume and Profit which explores the relationship
existing amongst Cost, Revenue, Activity Levels and resulting Profit. It aims
at measuring variations of Profits and Costs with Volume, which is
significant to business profit planning. CVP analysis makes use of the
Marginal Costing principles for planning and for making short-run decisions.
Sensitivity Analysis refers to analysis of the change in one factor on the other
related factors. For example, what will be the effect of a 10% increase in
selling price on sales volume and profits? It focuses on how a result will be
changed if the original estimates of the underlying assumptions change.
CVP based Sensitivity Analysis will help top management to get answers to
questions like – what will be the total profit if the sales mix is changed to
include more of one product and less of other product? or what will be the
profit if fixed cost increase by 30% and variable cost decline by 5%? etc.
CVP based Sensitivity Analysis can be performed in a spreadsheet package,
i.e. computerized CVP models. Computers will quickly show changes both
graphically and numerically based in data keyed in. Managers can study
various combinations of changes in selling prices, Fixed Cost, Variable Cost
and product mix, and can react quickly without waiting for formal MIS
reports from the financial officer.
4.
a) A product passes from Process-I and Process-II. Materials issued to
Process-I amounted to Rs. 80,000, Wages Rs. 60,000 and
manufacturing overheads were Rs. 54,000. Normal loss anticipated
was 5% of input. 9,100 units of output were produced and transferred
from Process-I. There were no opening stocks. Input raw material
issued to Process I were 10,000 units. Scrap has realizable value of
Rs. 4 per unit.
You are required to show: 5
i) Process-I account,
ii) Value of normal loss,
iii) Value of abnormal loss and
iv) Value of units transferred to Process-II

The Institute of Chartered Accountants of Nepal 83


Suggested Answer- December 2018

b) A firm makes special assemblies to customers‘ orders and uses job


costing. The data for a particular period are:
Particulars Job Number Job Number Job number
AA10 (Rs.) BB15 (Rs.) CC20 (Rs.)
Opening work in progress 26,800 42,790 0
Material added in period 17,275 0 18,500
Labour for period 14,500 3,500 24,600
The budgeted overheads for the period were Rs. 126,000.
Required: 5
i) What is the most logical basis for absorbing the overhead job
costs?
ii) Calculate the overhead to be added to job number CC20 for the
period?
iii) Job number BB15 was completed and delivered during the period
and the firm wishes to earn 33% profit on sales. What is the
selling price of job number BB15?
iv) What was the approximate value of closing work-in-progress at
the end of the period?
c) M/s. Shyam Sunder Bros. wants to ascertain the profit lost during the
year 2017/18 due to increased labour turnover. They have given you
the following information:
i) Traning period of the new recruits is 60,000 hours. During this
period their productivity is 75% of the experienced workers. Time
required by an experienced worker is 10 hours per unit.
ii) 20% of the output during training period was defective. Cost of
rectification of a defective unit was Rs. 40.
iii) Potential productive hours lost due to delay in recruitment were
120,000 hours.
iv) Selling price per unit is Rs. 400 and P/V ratio is 20%.
v) Settlement cost of the workers leaving the organisation was Rs.
400,000.
vi) Training cost was Rs. 200,000.
vii) Recruitment cost was Rs. 160,000.
You are required to calculate the profit lost by the company due to
increased labour turnover during the year 2017/18. 5
Answer:
a)
i) Process I Account
Dr. Cr.
Particulars Units Amount Particulars Units Amount
(Rs.) (Rs.)
To Material 10,000 80,000 By Normal loss (5%) @4 500 2,000
To Wages - 60,000 By Abnormal Loss 400 8,084
(Qty.- Bal. Fig) @
20.2105
To Overhead - 54,000 By Process II @20.2105 9,100 183,916
10,000 1,94,000 10,000 1,94,000

The Institute of Chartered Accountants of Nepal 84


Suggested Answer- December 2018

ii) Cost/Unit = 194000 - 2000


10000 - 500
= 20.2105

b)
i) The most logical basis for absorbing the overhead job costs is to use a
percentage of direct labour cost. If materials cost is used as the basis for
overhead absorption, it would give erroneous result as this would not be
equitable because job number BB15 incurred no material cost and would
therefore absorb no overhead. If Prime cost (material plus labour) is used
as the basis for overhead absorption the same disadvantage would arise.
Thus it is best to use direct labour hour as the basis for overhead
absorption.
ii) Overhead to be added to job number CC20 (absorbed on the basis of
direct labour hours)
= 24,600/(14,500 + 3,500 + 24,600) × 126,000 = 72,761
iii) Calculation of Selling Price to be quoted for Job BB15
Particulars Rs.
Opening WIP 42,790
Labour for the period 3,500
Overheads (35,00/42,600) × 126,000 10,352
Total Cost 56,,642
Profit @ 33% 27,898
Selling Price 84,540
iv) Calculation of Closing WIP (Considering point ii which states that Job
BB 15 has been delivered).
Job Workings WIP (Rs.)
Number
AA 10 (26,800 + 17,275 + 14,500) + (14,500/42,600) × 101,462
126,000
CC 20 (18500 + 24600 + 72761 [as calculated in ii]) 115,861
Total closing WIP 217,323
c)
i) Output by experienced workers in 60,000 hours = (60,000/10) , = 6,000
units
Output by new recruits = 75% of 6,000 = 4,500
units
Loss of output = 6,000 – 4,500 = 1,500
units
ii) Total loss of output = Due to delay recruitment + due to inexperience
= (1,20,000/10) + 1500 = 12,000 + 1,500 = 13,500
units
iii) Contribution per unit = 20% of Rs. 400 = Rs. 80
Total contribution lost = Rs. 80 × 13,500 units = Rs.
10,80,000
iv) Cost of repairing defective units = 4,500 units × 0.2 × Rs. 40 = Rs.
36,000

The Institute of Chartered Accountants of Nepal 85


Suggested Answer- December 2018

Profit forgone due to labour turnover


Particulars Amount (Rs.)
Loss of Contribution 10,80,000
Cost of repairing defective units 36,000
Recruitment cost 1,60,000
Training cost 2,00,000
Settlement cost of workers leaving 4,00,000
Profit forgone in 2017-18 18,76,000
5.
a) The following figures are related to Dabur Limited for the year ending
31st Ashadh, 2074:
Sales - 24,000 units @ Rs. 200 per unit;
P/V Ratio 25% and Break-even Point 50% of sales
You are required to calculate: 7
i) Fixed cost for the year
ii) Profit earned for the year
iii) Units to be sold to earn a target net profit of Rs. 1,100,000 for a
year.
iv) Number of units to be sold to earn a net income of 25% on cost.
v) Selling price per unit if Break-even Point is to be brought down by
4,000 units.
b) What are the aims of Material Requirement Planning (MRP)? 4
c) Explain briefly advantages of perpetual inventory system. 4
Answer:
a) Break- even point (in units) is 50% of sales i.e. 12,000 units.
Hence, Break- even point (in sales value) is 12,000 units x Rs. 200 = Rs. 24,00,000
i) We know that Break even sales = Fixed Cost / PV ratio
or Rs. 24,00,000 = Fixed Cost / 25%
or Fixed Cost = Rs. 24,00,000 x 25%= Rs. 6,00,000
So Fixed Cost for the year is Rs. 6,00,000
ii) Contribution for the year = (24,000 units x Rs. 200) x 25% = Rs. 12,00,000
Profit for the year = Contribution - Fixed Cost
= Rs. 12,00,000 – Rs. 6,00,000 = Rs. 6,00,000
iii) Target net profit is Rs. 11,00,000
Hence, Target contribution =Target Profit + Fixed Cost
= Rs. 11,00,000 + Rs. 6,00,000 = Rs. 17,00,000
Contribution per unit = 25% of Rs. 200 = Rs. 50 per unit
No. of units = Rs. 17,00,000/Rs. 50 per unit = 34,000 units
So, 34,000 units to be sold to earn a target net profit of Rs. 11,00,000 for a year.
iv) Net desired total Sales (Number of unit x Selling price) be X, then desired profit
is 25%
on Cost or 20% on Sales i.e. 0.2 X
Desired Sales = (Fixed Cost + Desired Profit) / PV ratio
X = (6,00,000 + 0.2 X) / 25%
or, 0.25 X = 6,00,000 + 0.2 X
or, 0.05 X = 6,00,000
or, X = Rs. 1,20,00,000

The Institute of Chartered Accountants of Nepal 86


Suggested Answer- December 2018

No. of units to be sold = 1,20,00,000 / 200


= 60,000 units
v) If Break- even point is to be brought down by 4,000 units then Break-even point
will be
12000 units - 4000 units = 8000 units
Fixed Cost = Rs. 6,00,000
Required Contribution per unit = 6,00,000 / 8,000 units
= Rs. 75
Selling Price = Contribution per unit / PV Ratio
=Rs. 75 / 25%
= Rs. 300 per unit
Hence, selling price per unit shall be Rs. 300 if Breakeven point is to be
brought down by
4,000 units.
b) The aims of material requirement planning to make use of computer in order to :
1. Determine for final products namely,what should be produced and at what
time.
2. Ascertaining the required units of production of sub-assemblies.
3. Determining the requirement for materials based on an up-to -date bill of
materials file.
4. Computing inventories, work-in -progress ,batch sizes and manufacturing and
packaging lead times.
5. Controlling inventory by ordering bought -in components and raw materials
in relation to the orders received or forecast rather than the more usual
practice of ordering from stock-level indicators.
c) Advantages of Perpetual inventory system
1. The system serves as a moral check on the stores staff. They keep the stores
record up-to-date and are differed from committing dishonest acts.
2. Interim financial accounts can be prepared with regular convenience because
the long and costly work of actual stock taking is avoided.
3. Over -investment in stock can be avoided because quantity and value of
materials in stock is always known.
4. In case of destruction of stock by fire, the system helps in settling the
insurance claim as correct stock figures can be readily obtained.
5. loss of stock due to pilferage or obsolence etc,is detected at an early stage and
suitable steps can be taken to prevent its recurrence.
6. Timely replenishment of stock is facilitated as the management may be
informed daily of the number of units and the value of each kind of material
on hand. This tends to eliminate delays and shut down in production
activities.

6. Write short notes: (4×2.5=10)


a) Explicit Cost and Implicit Cost
b) Product Cost and Period Cost
c) Absorption and Variable Costing
d) Engineered Cost and Differential Cost
Answer:
a) Explicit costs, which are also known as out of pocket costs, refer to costs

The Institute of Chartered Accountants of Nepal 87


Suggested Answer- December 2018

involving immediate payment of cash. Salaries, wages, interest on capital,


etc. are some of the examples of explicit costs. They can be easily measured.
Implicit costs (also known as economic costs) do not involve any immediate
cash payment.
The main points of difference between Explicit and Implicit costs are:
Implicit costs do not involve immediate cash payment whereas Explicit costs
involves immediate cash outflow.
Implicit costs are not recorded in the books of account whereas explicit costs
are entered in the books of account.
b) The costs which are a part of the cost of a product rather than an expense for
the period in which they are incurred are called as product cost. They are
included in inventory values.
The costs which are not associated with production are called period costs.
They are treated as an expense of the period in which they are incurred. They
may also fixed as well as variable. Such costs include general administration
costs, salaries of salesmen and commission, depreciation on office facilities
etc. They are charged against the revenue of the relevant period.
c) The basic difference between absorption and variable costing relates to the
handling of fixed manufacturing overhead. Under absorption costing, fixed
manufacturing overhead is treated as a product cost and under Marginal
costing it is treated as period cost. The other differences are:
1. The value of closing stock under absorption costing will be more as it
includes portion of fixed manufacturing over head.
2. The cost of manufacturing under variable costing is less than the
absorption costing
3. Variable costing takes contribution margin as the basis for decision
making and absorption costing takes the profit as the basis
4. The Over/under absorbed over head generally arises in absorption costing
where as under variable costing question of over/under absorption does
not arise.
d) Engineered costs are costs that result specifically from a clear cause and
effect relationship between inputs and outputs. The relationship is usually
personally observable. Examples of inputs are direct material cost, direct
labour cost. Examples of output are car, computer etc.
Differential costs represent the change in total cost due to change in activity
level, technology, process or method of production, etc. Example, if total
cost under alternative I is Rs. 30,000 and alternative II is Rs. 50,000, then
differential cost is Rs. 20,000.

The Institute of Chartered Accountants of Nepal 88


Suggested Answer- December 2018

Specific Comments on the performance of the students


Batch: - December 2018
Level: - CAP-II
Subject: Cost & Management Accounting
Question No. 1
Overall performance of the students was satisfactory. However, some students could
not calculate the gross manufacture percentage under different method of allocation of
joint cost. Student should focus on proper decision making and gross profit
computation need to be done properly.
Question No. 2
(a) Most of the students could not answer the question properly. Students did not
understand the cost relationship outlined in the question. Students are not able to
segregate the cost for each level of capacity utilization. Export order calculation
was difficult.
(b) Well attempted and answered by the students. Variances were easy to compute.
Question No. 3
(a) Most students could not calculate the lighting charges correctly. Computation of
room day and equivalent room day are not done properly. Performance of most of
the students was poor.
(b) Well answered by most of the students. Computation of C/S of F-goods is not
satisfactory.
(c) Answers were too sketchy and irrelevant. Students showed negligence on
theoretical part.
Question No. 4
(a) Most of the students well attempted the question.
(b) Lack of preparation was evident. Some students could not work out the overheads
applicable to different jobs.
(c) Most of the students could not calculate contribution foregone.
Question No. 5
(a) Well answered by the students.
(b) Lack of conceptual knowledge was evident.
(c) Answers were sketchy and irrelevant to the subject.
Question No. 6
Lack of conceptual knowledge of the students was assured. Students did not prepare
for the theory part and provided irrelevant details in answering the question. Part (d)
of the question regarding engineered cost has not been understood by the students.

The Institute of Chartered Accountants of Nepal 89


Suggested Answer- December 2018

Paper 6:

Business Communication & Marketing

The Institute of Chartered Accountants of Nepal 90


Suggested Answer- December 2018

Business Communication
Suggested
Roll No……………. Maximum Marks – 100
Total No. of Questions - 8 Total No. of Printed Pages -5
Time Allowed – 3 Hours
Marks
All questions are compulsory.
Section -'A'
1. Read the following case carefully and answer the questions given below: (4×5=20)
Generational differences abound in the workplace, but few are quite as
visible as body art: tattoos, piercings (other than ear lobes), and hair dyes
in unconventional colors. According to survey data from the Pew Research
Center, people younger than 40 are much more inclined than those over 40
to display some form of body art. For example, people 26 to 40 years old
are four times more likely to have tattoos than people who are 41 to 64
years old.
With such profound differences, it‘s no surprise that body art has become a
contentious issue in many workplaces, between employees wanting to
express themselves and employers wanting to maintain particular
standards of professional appearance. As employment law attorney
Danielle S. Urban notes, the issue gets even more complicated when
religious symbolism is involved. Who is likely to win this battle? Will the
body art enthusiasts who continue to join the workforce and who are now
rising up the managerial ranks force a change in what is considered
acceptable appearance in the workplace? Or will they be forced to cover
up in order to meet traditional standards? So far, most companies appear to
be relying on the judgment of their employees and managers, rather than
enforcing strict guidelines.
Many seem to accept that tastes and norms are changing and that body art
has become a widespread form of self-expression rather than a mode of
rebellion. Job seekers are still advised to be discreet, however, particularly
with facial piercings and large, visible tattoos. The nonverbal signals you
think you are sending might not be the signals a hiring manager receives.
Questions:
a) Should companies have stricter standards of appearance?
b) Should there be difference for ―customer-facing‖ employees than for
employees who do not interact with customers in terms of appearance?
c) Should companies allow their employees the same freedom of
expression and appearance latitude as their customer's exhibit? For
example, if a firm‘s clientele tends to be heavily tattooed, should
employees be allowed the same freedom?

The Institute of Chartered Accountants of Nepal 91


Suggested Answer- December 2018

d) Do people in Nepal have different perspectives and attitude on a


colleague who has tattoo or dreadlock hair or both?
Answer:
a) Companies according to their nature of jobs make different organizational
culture. Appearance of the employees depend on the type of job they perform.
Also, it depends on the space where they work, the culture and the age group
they belong to. The service industries like education, banking and health in
some part of the world might set strict standards of appearance for uniformity
and to assert the company identity. While imposing the stricter standard of
appearance the organizations should also take care of the types of clients.
Because organizations and companies are to satisfy their clients, they should
very carefully set organizational standard as far as uniformity and appearance
is concerned. Formality is considered as one of the important parts of an
organization. Now the question is what is ‗formal‘. So, the formal is defined
by the community and it is very sensitive for an organization to evaluate the
standard the community thinks and expects. If the community the organization
serves think formality as decent quality (in its perception) such as not having
tattoos or unusual piercing except in the ear lobe and nose, the company
should have the stricter standards of appearance where employees should not
look different with their body ‗fabricated‘ in tattoos and piercing or extremely
colored hairs in multiple colors. In such situation, the companies must have
stricter standards of appearance to meet the prospects of its clients otherwise
the companies may allow their employees to be casual.
b) The subject of appearance is considered as a personal freedom, however, once
an employee enters into the organization s/he should abide by its custom. It‘s
very difficult for a company to make two different sets of regulations in the
same company. The differences in terms of regulations might create conflict
among the colleagues. There is division between those who feel privileged and
the other forced. On the other hand, sometimes the subject of appearance
depends on the type of job one performs: such as an IT officer or a consultant
who doesn‘t need to be in the office like the other regular staff might be given
leverage and they can have relative freedom. But the employees who are in the
interface reflect the company image and they have the ‗face value‘ by which
the client judge company‘s formality in a way. Therefore, making the
difference in appearance for customer-facing employees and employees who
don‘t interact with the customers is totally the matter of emotional intelligence
of a manager.
c) The freedom of expression and appearance of latitude as the customer exhibit
depends on the type of service the company offers. If it is a company related to
fashion or cosmetics appearance latitude is almost mandatory for the
employees because it serves like an advertorial for the clients. The clients can
be attracted with the appearance of the employee and can demand similar
service. Or it is even easier for the clients to customize and improvise the type
of service or product they want to consume whether it is tattooing a body or
choosing a dress. However, the organization like bank and insurance where
various fashionable people receive service can‘t practice the similar trend
because the client might have different perception towards the organization.
Hence, the company at first should recognize its audience/consumers
expectation and perform accordingly.

The Institute of Chartered Accountants of Nepal 92


Suggested Answer- December 2018

d) Individual perspectives are never the same. The perspectives are determined
by culture, generation and location. Usually heavily tattooed body and
dreadlock hair are considered informal by the older generation in Nepali
community. Therefore, the employees with such exposes are not expected to
be decent and gentle. They are not even recruited in the offices where
formality is strictly followed. Most of the workers born and brought up in the
Nepali community have different perspective and attitude on a colleague with
tattoo or dreadlock hair. This may be because of the films, TV serials and
media representation which project such people as villain and evil power. No
matter people with customized body are intelligent, sincere and decent, first
impression and their nonverbal gesture does not communicate formality and
decency for some people. But the case might be different with an employee
who grew up in hip-hop generation with high influence of rock music. They
don‘t have different attitude such as negative towards tattooed body and
dreadlock hair. Hence, people belonging to different generation, location and
culture in Nepal have different perspectives and attitude on a colleague who
has tattoo or dreadlock hair or both.

2. Encoding and decoding are the basic processes of communication. What


do they refer? Discuss in brief the relationship between these processes. 10
Answer:
Encoding and decoding are the basic processes of communication. They are
concerned with how the sender frames the intended sense and how the receiver
perceives the sense framed by the sender. Business communication involves a
number of linguistic and non-linguistic elements such as words, phrases, discourse
markers, graphic tools, paralanguage features, instruments and so on which are
used by the participants to give a specific meaning. It is human mind that is
essentially important in making the meaning of what we have used as linguistic or
nonlinguistic device for communication.
The speaker‘s mind gives or determines particular meaning of the linguistic or
non-linguistic devices. The sender‘s mind is responsible for giving the meaning of
the language used. The process is known as encoding. With the help of linguistic,
socio- cultural, and experiential knowledge, a speaker encodes the meaning of his
or her speech, and intends that the receiver will also decode the meaning in the
same way. To be more specific, encoding is a sender‘s mental process of
presenting ideas or information in oral or written form, using sounds, letters,
words, figures or symbols.
Decoding is the receiver‘s mental process or act of assigning the meaning to the
words and symbols used by the speakers or writers. While encoding is concerned
with production, decoding is concerned with perception of discourse meaning in
context. When the message encoded by the sender is decoded properly by the
receiver, the process of communication is successful. Encoded messages are
decoded in the effective communication.
3.
a) What is business ethics? Discuss in brief economic, legal and
philosophical perspectives of business ethics. 5
b) Write a follow up letter after you faced an interview for the position of
credit manager at Chaudhari Group, Sanepa Lalitpur on 18th October,
2018. 5

The Institute of Chartered Accountants of Nepal 93


Suggested Answer- December 2018

Answer:
a) Ethics refers to the principles, rules, norms or code of conducts to govern
people. It is essential in every sector of life, profession and activity. In the
sector of business, ethics plays a vital role for effective conduct of
organization and transaction. Business ethics means obeying rules and
respecting customer values. Transparence and truth are also important aspects
of business ethics. It may incorporate social values, cultural norms, morality
and legal regularity too. It usually has three perspectives: economic, legal and
philosophical.
 Economic perspective refers to the tendency of avoiding financial abuses,
unfair business competencies, and unethical dealings. It encourages fair
business, accurate pricing, regular tax payment, and so on.
 Legal perspective refers to abiding in state rules, following regulatory
norms and conducting according to the established law and regulations.
 Philosophical perspective refers to the tendency of respecting others‘
cultural values, honor of self-esteem, mutual co-operation, etc. it
encourages truth, honesty, and loyalty in business.
b)
345 Kadaghari Marga
Kageswari Manohara, Kathmandu
19 October, 2018
Ms. Sajani Maharjan
Human Resource Manager
Chaudhari Group
Sanepa, Lalitpur
Dear Ms. Sajani:
Thank you for the opportunity yesterday to interview for the position of credit
manager for Chaudhari Group. I appreciated the time that you and other
members of your staff spent with me during the morning.
After meeting with you and your sales team, I am enthusiastic about the
position and the growth opportunities that it offers. My experience in retail
credit and previous problem-solving accomplishments, both with volunteer
work and work experience while earning my Business Administration degree
and a parallel course of CA match well with the responsibilities of the credit
manager position.
I look forward to hearing your hiring decision for the position. If you need
additional information, please phone me at 98510-76125 or send an e-mail to
shanti_sharma@gmail.com.
Sincerely,
Shanti Sharma
Shanti Sharma
4. Write short notes on: (5×2=10)
a) Functional resume
b) Executive summary of report
c) Ineffective listening
d) Kinesics

The Institute of Chartered Accountants of Nepal 94


Suggested Answer- December 2018

e) Characteristics of a good team


Answer:
a) A functional résumé, sometimes called a skills résumé, emphasizes
candidate‘s skills and capabilities, identifying employers and academic
experience in subordinate sections. This arrangement stresses individual areas
of competence rather than job history. The functional approach also has three
advantages: (1) Without having to read through job descriptions, employers
can see what you can do for them, (2) you can emphasize earlier job
experience, and (3) you can deemphasize any lengthy unemployment or lack
of career progress.
b) An executive summary is a brief version of the report; it restates each section
of the report in abbreviated form with an emphasis on findings, conclusions,
and recommendations. Other common names for an executive summary are
summary, abstract, overview, and synopsis. The summary saves readers time
by providing an overview of the report‘s contents. Reports that include a
synopsis in the letter of transmittal generally do not contain an executive
summary.
c) Failure to listen is a common barrier to successful oral communication.
Listening effectively is not easy. One reason listening is challenging is that
most people speak 150 to 200 words a minute but are capable of listening at
400 to 500 words a minute. This difference allows listeners‘ minds to wander
to topics other than the message. In addition, listeners may tune out a speaker
and begin thinking about how they will respond to the message. Senders can
use several methods to overcome the barrier posed by a receiver‘s poor
listening skills. Including phrases such as ―Take note of this next point; it is
particularly important‖ alerts receivers to listen carefully. Asking questions
periodically will help determine the extent of the listener‘s comprehension. In
some circumstances, a poor listener may be encouraged to improve her or his
listening skills. One of the most effective ways to remove poor listening as a
barrier to communication is to improve the quality of the message and the way
in which it is conveyed. Thoroughly analyzing the audience before designing
the message will help a sender plan, organize, and deliver an appropriate oral
message.
d) Kinesics is the study of body physical movements. It is the way our body
communicates without words and it‘s occupying a major aspect of non-verbal
communication process. When we communicate our thought process and
attitude is transmitted in the form of body movements, which reflects our inner
state of mind in our movements. Thus you can realize these expressions by
face and eyes, gestures, posture and physical appearance.
 Facial expression is the first way to communicate particularly eyes and eye
movement. We can express happiness, surprise, fear, anger and sadness
everything through our eyes.
 The gesture is our body parts especially arms, legs, hands and head convey
meaning. All these movements are made rationally along with our
speaking and not made with any intention.
 Body shape is related to our biological factor and it is natural but poster is
how we stretch our body in different styles. Body shapes are classified in
to three they are ectomorph which is thin, youthful, and tall then the

The Institute of Chartered Accountants of Nepal 95


Suggested Answer- December 2018

mesomorph is muscular fit body and finally endomorph which is fat,


round, and soft.
 Appearance includes clothes, hairstyle, jewelry, cosmetics and such styles,
which we include in our daily life by which we communicate how we want
to project our self.
e) Here are some characteristics of a good team:
 Good teams are adaptable to one another.
 Good teams have high energy.
 Good teams stay focused on their long-term goals.
 A good team learns the importance of being selfless.
 Its members are committed.
 Good teams communicate well with one another.

The Institute of Chartered Accountants of Nepal 96


Suggested Answer- December 2018

Marketing
Suggested Answer
Roll No……………. Maximum Marks –50
Total No. of Questions - 4 Total No. of Printed Pages -7
Time Allowed –3 Hours
Marks
All questions are compulsory.

5. Read the following case carefully and answer the questions given below: (45=20)
Saugat, a MBA from School of Management, Tribhuvan University,
thinking about running own business rather than being employed. After
graduating from University, he had meeting with his school friend
Prakash, graduated from Rampur Campus. Both are from middle class
family. They thought instead of doing a job, they will launch fresh
vegetables in market. Having learnt of the future conventional foods, they
decided to venture into cultivation of mushrooms.
Mushrooms are known to be the best alternative food for vegetarians. For
them fund raising was a serious handicap for mass production. However,
the first trial batch of mushrooms that they produced was bought by Stupa
Hotel in Kathmandu.
Further, the hotel placed orders for supply of 20 kgs every day. Now
mushroom industry is run by small entrepreneurs, like Saugat and Prakash.
Another big player M/s Thapa Mushrooms, equipped with cold storage
facility was more interested in the export market. Saugat and Prakash have
set their sights high. They aim to sell mushrooms in a very big way all
over the country. Mushrooms have a great market potential and is a
perishable food.
Questions
a) How do you support that choosing for mushroom was good product
choice?
b) How will you advise them, as how to increase the consumer awareness
about this food?
c) How do you suggest them to differentiate their consumer awareness to
two different market segments: Institutional sale and Individual sale?
d) What would be your suggestions for distribution channel for
mushrooms?

Answer:
a) Mushroom farming is good product choice they have made for their new business. Some
of the reasons are listed below, which show the choice for mushroom is right decision.
Firstly, Nepal has very suitable climate for mushroom farming. Many local farmers are
involved in farming of mushroom with lack of technical knowledge. In this sense,
Prakash who is agriculture graduated can fulfill this gap and can be updated. Secondly,
this farming can be run with low capital, which is suitable for the initial stage of
business. Third, the demand of Mushroom in Nepalese market has been growing in
increasing trend in recent years. This is additional product for vegetarian. In addition,
Nepalese society is gradually being aware from disadvantages of being non-vegetarian.

The Institute of Chartered Accountants of Nepal 97


Suggested Answer- December 2018

b) Consumer awareness can be created by test marketing. Through sales persons and
customer response to the product.
stores. Awareness can also be created through outdoor publicity such as wall hoardings,
banners, insertions in news-papers etc. Brand name of the company along with the
product can also be highlighted to the customer by using the concept of event marketing.

c) Institutional sales may include hotel, restaurants and industrial/college canteens; and
Individual sale refers to household.
For institutional sales, the possible awareness is approaching to hotel industry can be
made and product benefit can be shown to convince the customer. Mushroom related
recipe booklet can be given to them for use.
For individual sales, they can approach the T.V programs for ―Food show‖ to show
different recipes of Mushrooms in their shows.
Furthermore, dealer push can be done through sales promotion campaign. Press meetings
can be a way to consumer awareness. Editors, journalists of newspapers having maximum
circulation can be contacted and samples to be distributed to them (such as 250 gm or 100
gm packs). Finally, packaging should be attractive.
This can be suggested that many of awareness program can be used for both sales.

d) Mushroom is perishable product, company should go for faster and effective distribution
network h
local market and distribution through road transport to urban markets.
Distribution channel can be more specific for different market segment. For instance, zero
layer channel could be used for industrial sale and others channels for household.

6. What is marketing concept? Discuss basic principles of modern marketing


concept. (4+6=10)

Answer:
The marketing concept is the key to achieve organizational goals through the determination
of needs and wants of target markets and to deliver the desired satisfaction more effectively
and efficiently than the competitors.
Principles of marketing concept:
 Target Market: A target market is defined as a set of actual and potential buyers of a
product, service, or idea. A buyer, who has interest in the product, income and willingness
to buy, can broadly be called as potential buyer. However, it might not be possible for the
marketer to target all of them. Thus, a small portion of potential market might become part
of the target market.
 Customer Oriented: In this approach, product-design, features, quality, price, production
schedules, advertising, distribution channels, promotion of the product and personal selling
are all aimed at satisfying the customer. Customer orientation approach requires the
gauging of needs of customer from customers‘ point of view and not from the view of
marketing personnel.
 Integrated Marketing: It requires coordination and cooperation within the department.

The Institute of Chartered Accountants of Nepal 98


Suggested Answer- December 2018

Satisfaction of the customer should be the main concern of the organization and all efforts
should be geared to this end. All marketing functions viz, marketing research, advertising,
pricing, promotion, distribution etc. are organizationally coordinated under the marketing
department to satisfy customer needs. Marketing department cannot do its task effectively
and efficiently if production department does not produce goods of the required quality at
a feasible cost and if finance department does not provide adequate capital. The
interdependence of various departments in an organization cannot be overlooked.
 Profitability: Business organizations are set up to earn profit. Huge funds are invested in
them, some of which are borrowed and are to be repaid. Therefore, a business firm must
get a reasonable return on the investment. Profit is one of the main objectives of an
organization. Therefore, notion of satisfaction of customer needs, which is the most
important principle of the marketing concept, does include the prospects of earning
reasonable profits. Therefore, the marketing concept holds that the firm should seek profit
making opportunities and strive to find profitable ways to satisfy customer needs.
7.
a) What is new product? Why is it important? Briefly explain the new
product development process. (1+1+3=5)
b) What are the functions of promotion? Discuss the factors affecting the
determination of the promotion mix. (3+2=5)

Answer:
a) New product refers to original products, improved products, modified products and
new brands that the firm develops. It is that, which organization produce for the first
time and other way new product is that which customer perceives as a new by its
different characteristic.
It is important for every organization as single product cannot satisfy customer
forever. Because customer‘s needs, desire, taste and preferences are always
changeable in nature, new product is essential to be competitive in market.
New product development process set outs a series of stages that new product
typically goes through, beginning with idea brainstorming and ending with preparing
for launch.
Product Idea Brainstorming
The first step is to generate an idea for the product. Employees, especially those who
deal with customers regularly, are major source for product ideas. In addition,
customer survey could be conducted for feedback on existing products. Marketer has
to examine the industry to see whether there are areas where useful products do not
exist. Online survey for customers or social media fans would be the good techniques
for brainstorming. And all ideas for a new product are to be listed.
Evaluate the Ideas
Listed product ideas are to be shared with the appropriate decision-makers in the
company, such as the management team. Pros and cons of each idea are to be
discussed and narrow the list to just a handful of the best ideas, based on their
potential to generate revenue, as well as the time and resources available to actually
create the products.
Market Evaluation
Feedbacks are to gathered from customers, employees and partners on which idea is
most appealing. Whittle the list to just one or two product ideas.

The Institute of Chartered Accountants of Nepal 99


Suggested Answer- December 2018

Analyze the Competitive Situation


Remaining product idea from a business perspective is to be analyzed. In this course,
competition between similar products, and demand for the product are to be
determined. In addition, there is need of estimation of costs affiliated with the
product; such as development costs and operational costs, to help determine the profit
margin.
Prototype and Marketing
Developing a prototype of the product is next step, which is to be shared with a
handful of good customers and key partners aiming to gain feedback. The marketing
team should use that feedback to craft marketing messages and developing marketing
campaign ideas, such as email campaigns, websites, billboards or posters.
Market Testing
In market testing adjustments to the prototype is expected. Then, prototype is
modified into a new version, if necessary. In fact, a small product is released in select
areas aiming to examine potential sales status, evaluate the price and effectiveness of
the marketing message. A small launch, expected in market testing, helps determine
what needs to be done before an official launch.
Prepare for Launch
Prior to launch, volume of products to produce are to be determined based on market
testing and demand for the product. In addition, advertise and speak to product
distributors about ordering the product to be done, if the product will be sold in stores.

b) Functions of Promotion
So, main functions of promotion are as follows:
a) Informing
b) Persuading
c) Reminding
d) Reassuring
a) Informing
The first function of promotion is to give information about their goods or services
to middlemen, final consumers and industrial users. Even a very useful product
cannot get success in market if nobody gets any information about it. So, giving
information about the product to prospective consumers is one of the important
functions of promotion.
b) Persuading
Persuading is to motivate customers to buy products. It becomes essential for
producers or sellers to make persuasion program under promotion due to intense
competition among different industries producing similar types of products. Only
information about goods or services can do nothing. The sellers should give
information about the benefits, quality, utility, price etc. of products to make
consumers confident that their needs will be satisfied with the goods or services.
c) Reminding

The Institute of Chartered Accountants of Nepal 100


Suggested Answer- December 2018

The consumers should be reminded from time to time of the availability,


satisfaction, utility, benefits etc. of products. Because of cut-throat competition in
production and marketing, every firm should always conduct promotional
programs to remind the customers of their products and exist in markets.
d) Reassuring
After buying a product, the customers may not be confident in their buying
decision. They may feel whether they took right or wrong decision to buy a certain
product. So, promotion plays an important role to make them confident about the
quality, utility and benefits of the goods they have bought. This makes them free
from worry about the brand of the product, increases brand loyalty in them and
improves image of the firm. This is called reassuring.

Factors Affecting Determination of Promotion Mix

Following factors affect promotion mix.

 If the objective is to build awareness level among the buyers, the promotion mix is
dominated by advertising and publicity.
 If buyers are already aware of the product but do not exhibit liking, preference and
conviction for the brand, the marketer needs to put more emphasis on personal
selling which is more persuasive than advertising.
 If buyers have favorable attitude toward the brand, but do not show clear purchase
intentions, sales promotions may provide incentives to undertake immediate
actions. Buyers can be better reminded by display advertisements.

The following factors affect the determination of promotion mix:


a) Size of the promotion budget
The size of the promotion budget determines the selection of the promotion mix.
Advertising requires heavy expenses as it has to be launched for a longer time period.
Similarly, maintaining large sales force for aggressive personal selling campaign is
suitable for financially strong firms. Firms with small promotion budget prefer to use
a combination of dealer promotion, on the spot displays, pamphlets etc.
b) Nature of the Market
The structure of the market, particularly its geographical coverage affects the design
of the promotion mix.
 In a small market, personal selling, consumer promotions and local level
advertising is a good promotional blend.
 In large markets, a blend of advertising, dealer promotions and consumer
promotions are more effective.

The Institute of Chartered Accountants of Nepal 101


Suggested Answer- December 2018

 Trade customers such as wholesalers and retailers are better approached with
personal selling while consumers can be accessed through advertising.
 In industrial and institutional markets, advertising is less important than personal
selling
c) Nature of the Product
 Convenience products are generally mass merchandised and supported with heavy

advertising by the manufacturer.


 Shopping products require more emphasis on personal selling than on advertising.
 Specially goods and services are mostly sold through good public relations.
 Specially goods and services are mostly sold through good public relations.
 Although most of the industrial products are largely sold through personal selling,
operating suppliers are extensively advertised.
 Level of social risk involved in the purchase and ownership of the product also
affects the promotion blend. Purchase of convenience items involves no social risk
for the buyer. In high social risk products, such as jewelry and clothing some
personal selling has to be added to high image advertising.
 Items that are a relatively small part of the organization's budget or the buyer's
budget do not require salesperson to pursue and close the sale. If the product is
expensive, technically complex and requires after sales services, personal selling
is more suitable and productive.
d) Product life Cycle
Promotion blend changes as the product moves from one stage to another stage of the
product life cycle.

 In the introduction stage, personal selling and advertising go together.


 During the growth stage, personal selling has little use while advertising is
continued with a change in its focus from awareness creation to building
conviction among buyers.
 In the maturity stage, sales promotion plays the dominant role and advertising is
used only to support the sales promotion campaigns.
 During decline stage, all forms of promotion become irrelevant.

8. Briefly explain the following: (5×2=10)


a) E-marketing
b) Marketing mix
c) Market Research Process
d) Zero Level channel structure for consumer good
e) Macro environment

The Institute of Chartered Accountants of Nepal 102


Suggested Answer- December 2018

Answer:
a) E-marketing:
Electronic Marketing (E-Marketing) can be viewed as a new philosophy and a modern
business practice involved with the marketing of goods, services, information and
ideas via the Internet and other electronic means.
In this respect most of the researchers misused the term E-Marketing; majority of
researchers are using the term: E-Marketing/ Internet Marketing / E-Commerce/E-
business as equivalents or a different wording for the same meaning, which is
incorrect because they are different.
For example, E-Marketing has broader scope than internet marketing since Internet
Marketing (IM) refers only to the Internet, World Wide Web, e-mails. While E-
Marketing includes all of that plus all other E-Marketing tools like: Intranets,
Extranets and mobile phones to satisfy customers and ultimately to get profits.

b) Marketing mix
Marketing mix is a set of controllable, tactical tools,-product, price, place and promotion-
that the firm blends to produce the response it wants in the target market. The marketing
mix consists of everything the firm can do to influence the demand for its product.

c) Market Research process:


The marketing research process is a six-step process are as follows:
a) Identifying and defining the problem/opportunity.
b) Planning the research design
c) Selecting the sampling procedure
d) Data Collection
e) Evaluating the data
f) Preparing and presenting the research report.
Involving the definition of the problem being studied upon, determining what
approach to take, formulation of research design, field work entailed, data preparation
and analysis, and the generation of reports, how to present these reports, and overall,
how the task can be accomplished.
d) Zero Level channel structure for consumer good
The channel in which no intermediaries exist between producer and consumer is
called Zero level channel. Producers distribute their goods to consumers through
multiple shop and chain stores. Goods are distributed through sales representatives‘
mail order, tele-marketing and online marketing.
e) Macro environment
The macro environment includes all the factors that can influences the organization
and its marketing mix but that are out of their direct control. A company does not
generally influence any laws, population or economy. It is continuously changing
and the company needs to be flexible to adapt. According to Philip Kotler, ―Macro
Environment consists of the large social forces that affect the micro environments –
demographic, economic, natural, technological, political and cultural forces. Macro
environment provides opportunities and threats to the marketing.

The Institute of Chartered Accountants of Nepal 103


Suggested Answer- December 2018

Specific Comments on the performance of the students


Batch: - December 2018
Level: - CAP-II
Subject: Business Communication
Question No. 1
Overall performance is ok, but requires more clarity in terms of opinion and
restatement of the case time and again needs to be reduced. Many students are found
writing either lengthy or very short answer. There should words limit given.
Question No. 2
Most of the students have correctly addressed this question, but could have been
improved by adding diagrams.
Question No. 3
Students are found failing/performing very poor at writing a follow up letter. Students
have not more clarity in 'Business ethics'. There was lacking of formalities in follow
up letter.
Question No. 4
Some of the students are found not familiar with the term 'Kinethesis' instead of '
Kinethesis' the question should have been given non – verbal language. Due to the
nature of the question and weightage of marks, the answers were not in detail as the
candidate had to give short answer. In general, there is lacking of clarity in the
specifications of different resumes.

Subject: Marketing
Question No. 5
In general, dealing with case can be more improved. Candidates are supposed to relate
the specified problem with existing theory to deal case precisely. The students are not
conscious about the case studies. Majority of them are not serious about the issue of
the cases. So they are weak.
Question No. 6
Candidates are found answering superficially for the asked questions having higher
weight. Most of the students did not understand about principles of modern marketing
concept.
Question No. 7
Though the answer seems satisfactory with respect to concept, presentations of
dealing question can be improved. Students are confident in this section but confused
in factor affecting the determination of promotion mix. They are clear about the
process of new product development.
Question No. 8
Concept is clear. However, there is still room for improvement in answering.

The Institute of Chartered Accountants of Nepal 104


Suggested Answer- December 2018

Paper 7:

Income tax & VAT

The Institute of Chartered Accountants of Nepal 105


Suggested Answer- December 2018

Income Tax & VAT


Suggested
Roll No……………. Maximum Marks - 100
Total No. of Questions - 8 Total No. of Printed Pages -14
Time Allowed - 3 Hours
Marks
Attempt all questions. Working note should form part of the answer.
8. Mega Vision Pvt. Ltd., Pokhara, a manufacturer and exporter of garments;
has the following incomes and expenses for Income Year (IY) 2074/75:
Particulars Amount (Rs.)
Export sales 50,000,000
Misc. income 2,850,000
Dividend income 950,000
Opening stock 15,000,000
Raw material import 14,000,000
Freight for raw material 900,000
Custom duty paid for raw material 1,500,000
Custom agent fee 150,000
VAT paid for import 1,500,000
Wages for production 3,000,000
Overhead cost (production) 2,000,000
Administrative expenses 1,500,000
Selling and distribution expenses 1,000,000
Taxes paid to state and local government 1,500,000
Interest paid to financial institution 300,000
Penalties paid to metropolitan office 200,000
Donation 1,100,000
Depreciation 1,200,000
The following information are relevant for income tax calculation:
a) Sales include VAT refund of Rs. 1,500,000 which was paid at the
customs point for import of raw materials.
b) Miscellaneous income Rs. 2,000,000 is the incentive given by the
Government of Nepal (GON) for export of previous year and is
received during this year through Nepal Rastra Bank as the right to
such incentive was established during the current income year.
Remaining miscellaneous income Rs. 850,000 is a net of tax amount
received from the bank deposit incidental to the business.
c) Dividend income is also a net of tax amount received from a subsidiary
company.
d) The opening stock and closing stock have been recorded 1,000 pieces
and 1,200 pieces respectively. The opening stock includes fixed factory
overhead Rs. 10 per piece and machineries repair and maintenance Rs.
15 per piece. During the year, the Company produced 2,000 pieces of

The Institute of Chartered Accountants of Nepal 106


Suggested Answer- December 2018

garments and the overhead cost includes equal rate of previous years'
fixed overhead and repair and maintenance cost.
e) Administrative expenses include employees' travelling expenses Rs.
100,000 during the import of raw materials and also include
entertainment expenses Rs. 50,000 provided to the Custom's
employees.
f) The Company has made employees' bonus provision Rs. 2,200,000 as
per the Bonus Act. During the year, it has paid Rs. 400,000 as full
settlement of previous year's bonus. The administrative expense
presented in the above table includes the previous year's bonus but
excludes the current year's bonus.
g) The Company has made provision for Corporate Social Responsibility
(CSR) for Rs. 187,000 as 1 percent of profit after tax as per Section 48
of Industrial Enterprises Act, 2073. The amount is included in the
administrative expenses.
h) Selling and distribution expenses were paid to the cargo agent for
export, invoices amounting Rs. 300,000 from the cargo agent were
received, and the remaining amount pertains to goods transportation
from the factory to depot point as per details given by the agent.
i) Interest expense is charged against the loan taken to purchase the
factory machinery costing Rs. 2,000,000. The machinery was
purchased on 1st Baishakh, 2075 and has been put to use from 20th
Bhadra, 2075.
j) Donation includes Rs. 100,000 directly paid to the flood victims; the
remaining amount was deposited into Reconstruction Fund established
by the GON for earthquake victims.
k) Opening balance of depreciable assets as on 2074.04.01 as per tax
return records are as follows:
Building Rs. 6,000,000
Cars Rs. 3,000,000
Plant & Machinery Rs. 8,000,000
l) The company disposed a Santro car for Rs. 800,000 during the year and
the written down value of the car at the disposal time was Rs.
1,000,000.
You are requested to ascertain: 20
i) Taxable income
ii) Tax liability
iii) Dividend tax that must be paid by the Company, if the Company
has proposed to declare dividend of Rs. 3,000,000.
Solution:
Calculation of Taxable Income & Tax Liability of Mega Vision (Pvt.) Ltd. for
IY 2074/75

Particulars Sec. Note Amount Rs.


Inclusions:
Sales 7 (2) 1 48,500,000

The Institute of Chartered Accountants of Nepal 107


Suggested Answer- December 2018

Miscellaneous Income 7 (2)


Export Incentive 2.1 2,000,000
Interest income directly related to business 2.2 1,000,000
Dividend Income 7 (3) & 92 3 0.00
Total Inclusions (A) 51,500,000
Deductions:
Interest Expense 14 4
Cost of Trading Stock 15 5 23,593,000
Depreciation 19 6 2,586,940
Repair & Improvement Cost 16 7 30,000
Administrative expenses 13 8 863,000
Selling & Distribution Expense 13 9 300,000
Provision for Employee Bonus 13, 21 10 2,200,000
Penalties paid to Metropolitan Office 11 0.00
Tax Paid to State and Local Government 12 1,500,000
Total Deductions (B) 31,072,940
Assessable Income from Business (A-B) 20,427,060
Less: Allowable Reductions
Donation to Exempt Entities 12 13 0
Contribution to National Reconstruction Fund of
GON 12B 13 1,000,000
Taxable Income 19,427,060
Calculation of Tax Liability
Taxable Income 19,427,060
Tax Rate 14 15%
Tax Liability 2,914,059
Can't be
computed
Calculation of dividend tax 54 15 correctly
Notes & Working Notes:
1. Calculation of Sales as per Tax
Particulars Amount (Rs.)
Export Sales (given) 50,000,000
Less: VAT refund included in Sales (Note 1.1) -1,500,000
Sales u/s 7 (2) (b) 48,500,000
1.1. VAT is refundable duty and does not form part of either cost of
sales when paid or income when refunded.
2. Miscellaneous Income
2.1. Export Incentive:
As per Sec. 22 of Income Tax Act, 2058; company must follow accrual basis
of accounting. However, as the right to such incentive was established during
the current income year, it is income of this year and is taxable in the current
income year.
2.2. Interest income, though return of investment in nature, but is related to
business objective of the company and hence included in income as per Sec.
7 (2) (g) of the Income Tax Act.

The Institute of Chartered Accountants of Nepal 108


Suggested Answer- December 2018

Since, interest income has been recorded net of tax, it must be grossed up.
Total interest income is included in taxable income and the TDS thereon is
adjusted with tax liability.
Interest income = Rs. 850,000/0.85= 1,000,000
TDS= Rs. 850,000*15/85 = Rs. 150,000
3. Dividend income received from resident company is a final withholding
payment as per Sec. 92 of the Act. and is not included as income in
calculation of taxable income.
4. As per Sec. 14 (1), if interest is accrued on loan borrowed for the
purchase/creation of asset during the income year, such interest expense shall
be claimed only when the related asset is used during the income year. Since,
the asset is not used during the year; the interest expense is not deductible.
5. Calculation of Cost of Trading Stock
Particulars Amount
Opening Stock 15,000,000
Less: Adjustment for Repair & Maintenance Expense (15*1000) (15,000)
Value of opening stock (A) 14,985,000
Add: Cost of Production
Import of Raw Material 14,000,000
Freight for Raw Material 900,000
Custom duty paid for raw material 1,500,000
Custom agent fee 150,000
Wages 3,000,000
Overhead Expense (assumed to be production overhead) 2,000,000
Less: Adjustment for Repair & Maintenance Expense in cost of production
(2000*15) -30,000
Total cost of production during the year (B) 21,520,000
Less: Value of Closing Stock (Assumed FIFO) (C) 12,912,000
(Cost of Production divide by 2000 multiplied by 1200)
Cost of Trading Stock (D=A+B-C) 2,359,3000
5.1. Refundable duties do not form part of cost of inventories. As such, VAT paid on
import is not part of cost of trading stock, and hence, not included.
6. Calculation of Depreciation Expense
Particulars Pool A Pool C Pool D
I. Depreciation Rate 6.67% 26.67% 20.00%
II. Opening Depreciation Base 6,000,000 3,000,000 8,000,000
III. Absorbed Additions
IV. Disposals 800,000
V. Depreciation Base 6,000,000 2,200,000 8,000,000
VI. Depreciation Expenses 400,200 586,740 1,600,000
VII. Total Depreciation Expense 2,586,940
Land is non-depreciable asset.
Additional 1/3rd depreciation is provided to
special industries.
7. Calculation of Eligible Repair & Improvement

The Institute of Chartered Accountants of Nepal 109


Suggested Answer- December 2018

Cost
Particulars Pool A Pool C Pool D
I. Depreciation Base 6,000,000 2,200,000 8,000,000
II. 7% of Depreciation Base 420,000 154,000 560,000
III. Actual Repair & Improvement Cost - 0 30,000
IV. Eligible (Lower of II or III) - - 30,000
8. Eligible Administrative Expenses
Particulars Amount
Given 1,500,000
Less: Entertainment Expense Note 8.1 -50,000
Less: Bonus Paid for previous year erroneously
charged as expense Note 8.2 -400,000
Less: CSR Provision Note 8.3 -187,000
863,000
8.1. Since the amount is paid to customs'
employee without proper receipt of service and
proper justification for such payment, the amount
is a cost not for business purpose, thus, is not a
deductible expense u/s 13 (c) and Sec. 21 (1) (f).
8.2. Company must follow accrual basis of
accounting, as such, the bonus expense accrued
for last year and paid during the year included as
part of administrative expense is not deductible
expense u/s 13. (It is an accounting error and must
be rectified, which decrease administrative
expense for accounting purpose as well).
8.3 CSR is appropriation of profit and hence, as
per accrual basis, not deductible.
9. Eligible Selling & Distribution Expenses
Particulars Amount
Given 1,000,000
Less: Ineligible due to absence of supporting (700,000)
Given 300,000
9.1. The expense without supporting is ineligible
as the purpose of expense cannot be justified due
to lack of evidence justifying the purpose.
10. Calculation of Bonus and CSR expenses
Companies follow accrual basis of accounting, accordingly, the
provision for expenses are deducted as per Sec 24.
Bonus provision of this year is allowed as expense, while bonus of
last year is not allowed as noted in point 8.

11. Penalty is not a deductible expense as it is paid as a result of


infringement of law u/s 21 (1) (b).
12. Tax paid to municipality and local government is allowed for
deduction as per provision to Sec. 21(1)(b)

The Institute of Chartered Accountants of Nepal 110


Suggested Answer- December 2018

13. Since direct payment to flood victim is not a


payment of donation to exempt organization, thus
the amount is not deductible. The contribution to
National Reconstruction Fund of GON is eligible
for deduction u/s 12B.
14. Calculation of Effective Tax Rate
Since it is a special industry, the applicable tax
rate as per Schedule 1, Section 2 (3) is 20%. As it
is export oriented industry, it is entitled to
exemption of 25% on tax rate as per Sec. 11
(3Nga) and as such, the effective tax rate is 15%.

9. Mr. Shyam is an Engineer working since 1 Jestha, 2075 in an Airlines


Company. The contract period is for 4 years. The Company has been
charging expenses related to his service. Calculate the tax to be deducted
by the Company on his employment income for the month of Ashwin,
2075. 10
Charging Monthly Expenses:
a) Monthly remuneration Rs. 80,000 (60% is basic salary and the
remaining amount is allowance). Festival allowance Rs. 80,000 for the
year.
b) The Company contributes 10% of his salary and he also contributes the
same amount and is deposited into an approved provident fund. In
addition, the Company deposited Rs. 10,000 per month to Citizen
Investment Trust as per his request.
c) The Company makes a provision for gratuity @ 8.33% on his basic
salary.
Facilities Provided:
a) He has been provided a car along with a driver by the Company for
both official and personal use. Actual petrol expenses Rs. 30,000
pertaining to the fiscal year for upto Ashwin end, 2075 was reimbursed
by the Company and no such facility shall be provided further.
b) He has been provided a chair for physically disabled people for his
convenience; annual maintenance cost Rs. 6,000 is also borne by the
Company.
c) He has got a Samsung S 9 Mobile phone from the Company on 5
Bhadra, 2075. Purchase price of the phone was Rs. 90,000. Monthly
total telephone expenses amounting to Rs. 1,500 is reimbursed to him.
The phone is used equivalent to 33.33 percent for personal calls.
d) The Company is providing free lunch during office hours, the cost of
which is Rs. 100 per meal to all employees. He has been receiving Rs.
150 per meal due to his special work. Assume 252 working days in a
year.
e) Annually, the Company pays a premium of Rs. 25,000 against his
health insurance.

The Institute of Chartered Accountants of Nepal 111


Suggested Answer- December 2018

f) The Company has been providing bonus to its employees, who served
at least 6 months in the company. Such bonus is equivalent to 3 month's
basic salary.
Other evidence and information submitted by him to the Company for tax purpose are:
a) He has paid premiums of Rs. 15,000 and Rs. 7,000 respectively against
his and his wife's life insurance policies.
b) He and his wife declared that they are couple in the Income Year
2075/76 and his wife does not involve in any income generating
activities.
Solution:
Computation of Taxable Income, Tax Liability and Monthly TDS amount of Mr. Shyam
(couple status) for IY 2075/76
Particulars Amount in Rs.
Salary (80,000 x 12) 960,000.00
Dashain allowance 80,000.00
Contribution to Provident Fund (PF)
57,600.00
(960000*.6*.1)
Gratuity (note a) -
Vehicle facility (0.5% of (960,000*.6) (note b) 2,880.00
Bonus (note c) -
Petrol expenses -
Maintenance cost of Handicapped (note d) -
Purchase price of phone (Rs. 90,000) 90,000.00
Personal communication expenses (Rs. 500*11)
5,500.00
(note e)
Meal allowances (150*252) (note f) 37,800.00
Health Insurance premium 25,000.00
Bonus (note g) -
Assessable income 1,198,780
Less
Deductible Amount against contribution to PF 235,200
Which is lower of
Actual Contribution (57,600*2 +12*10,000) 235,200.00
1/3rd of Assessable Income 399,593
Maximum Limit 300,000.00
Taxable Income 963,580
Deduction against life Insurance premium
(22,000.00)
lower of
Actual payment 22,000
Maximum Limit 25,000
Deduction against health Insurance premium
(20,000.00)
lower of
Actual payment 25,000.00
Maximum Limit 20,000

The Institute of Chartered Accountants of Nepal 112


Suggested Answer- December 2018

Deduction for disabled person (50 % of Rs.


(200,000.00)
400,000) (note h)
Balance Taxable Income 521,580
Computation of Tax Liability
Up to Rs. 400000 @1% Social security 4,000.00
Next Rs. 100000@10 % 10,000.00
Next Rs. 21,580@20 % 4,316.00
Total annual tax 18,316.00
Monthly tax to be deducted (18,316/12) 1,526.00
Note
a) Remuneration tax is calculated on cash basis, provisions of the gratuity amount
by the company are not considered payment to the employees, so it is not
included in the taxable income.
b) Vehicle facility provided by the company is quantified at 0.5% of basic salary
under Rule 13 of Income Tax Rules, 2059.
c) Employees are entitled to receive the bonus amount to the year on which they
have worked, during the last year. He has worked only 2 months, so he cannot
get the bonus amount from the profit of the last year.
d) Chair facility is provided to him as a basic official requirement, the chair also
belongs to the company as per the information given in the question, so
maintenance cost is not considered while calculating the taxable remuneration
income under Section 8.
e) Personal benefits out of Purchase price and phone expense paid by the
employer are only included in the taxable income. The Phone is purchased on
Bhadra, so phone expense is considered since Bhadra, 2075.
f) If the employer provides free lunch on equal basis to all employees, such
amount is not taxable. But in this case, special free lunch is provided to him,
and then such amount is included in the taxable income.
g) As per the schedule 1 of Income Tax Act, a disabled person is entitled to
deduct 50 % of first slab amounting to Rs. 400,000. The company provided the
handicapped chair, so he is considered a disabled person.
10.
a) M/S Sonica Pvt. Ltd. having registered office at Kathmandu,
exclusively deals in computer trading wholesale business. It recently
extended its branches to all 77 districts of Nepal. The Company imports
computers for wholesale business @ Rs. 85,000 per computer. After
opening the branches, the Company sent the imported computers for
office use one piece to each branch. Is there any tax implication for the
above transaction as per the provision of Income Tax Act, 2058? 5
b) Saina and Safina decided to jointly invest Rs. 2,000,000 and Rs.
3,000,000 respectively. Out of the collected amount, they bought land
for Rs. 4,500,000 with additional registration and other expenses Rs.
500,000. They sold the land for Rs. 8,600,000, out of which, they
spent Rs. 600,000 for commission and other expenses. Calculate their
taxable income as per Income Tax Act, 2058? 5

The Institute of Chartered Accountants of Nepal 113


Suggested Answer- December 2018

Solution:
a) When the trading stock of M/S Sonica Pvt. Ltd. is changed to depreciable
assets, it is deemed as disposal as per Section 40 (3) (Gha) of IT Act. So, the
company should calculate Incomings, Outgoings, Gain and Net Gain
respectively as per Sections 39, 38, 37 and 36. If there appears any net gain,
the amount should be included as inclusion in the income from business as per
Section 7 (2).

b) As per Section 29 of the Income Tax Act, 2058, the amount to be included or
deducted for determining income from investment of two or more persons shall
be performed on the basis of the ratio of their contribution i.e. contribution
ratio. In the given case, Saina and Safina shall calculate their taxable income
by dividing the overall income and expenses in the ratio 2:3 i.e their
investment ratio.
Partiulars Total Rs. For Saina Rs. For Safina Rs.
Expenses cost of land
Other charge 45,00,000
Total 2:3) 5,00,000
Outgoings (A) 50,00,000 20,00,000 30,00,000
Selling price
Less: Expenses
Net Selling price 86,00,000
Net profit (B-A)
6,00,000
Incomings (B) 80,00,000 32,00,000 48,00,000
Gain/ Net Gain = (B-A) 30,00,000 12,00,000 18,00,000

11.
a) A company operating in Bhutan is engaged in production of Jam
products. It conducted its AGM at Thimpu and elected its board
members. The majority of elected board members represent from
Nepal. During FY 2074/75, most of the board meetings took place in
Kathmandu. The company has earned Rs. 10,000,000 from
Bhutan, Rs. 30,000,000 from India, Rs. 20,000,000 from Bangladesh
and Rs. 10,000,000 from Nepal during FY 2074/75. Decide the
residential status and taxable income of the Company as per the
provision of Income Tax Act, 2058? 5
b) Horizon Pvt. Ltd. entered into a publication copyright contract on 1st
Chaitra, 2072 with Florish Pvt. Ltd. to publish a book. Royalty is fixed
Rs. 1,200,000 for 2 years and 10 months with maximum copies of
10,000 per year. Find the deductible expenses for three years for tax
return. 5
c) Quantify the value of consideration as per Section 27 of Income Tax
Act, 2058 under the following circumstances: (5×1=5)

The Institute of Chartered Accountants of Nepal 114


Suggested Answer- December 2018

i) Mr. Ram gave his smart TV for settlement of payables to a


shopkeeper. Mr. Ram had purchased the TV for Rs. 45,000 three
years back, and the market value at the time of handing over to the
shop was Rs. 25,000.
ii) XYZ limited provides free motor vehicle costing Rs. 15,000,000 to
Mr. Ganpath, a government official.
iii) PQR Limited provides two security personnel free of cost to Mr.
Mohan, a marketing manager, for which the company pays Rs.
240,000 for a year.
iv) K&K Ltd. provides free accommodation to a public director of the
Company, for which the Company pays annual rent Rs. 500,000.
v) K&K Ltd. provides its own bungalow costing Rs. 20,000,000 as
free accommodation to a local leader, normal rent for the
bungalow is Rs. 100,000 per month.
d) Write the tax implications on the following cases as per Income Tax
Act, 2058 : (5×1=5)
i) Smooth Telecom Pvt. Ltd. deposits all gratuity amounts into an
Approved Retirement Fund. On 30 Bhadra, 2075, the Fund paid
for gratuity Rs. 30 lakh to Mr. Ram upon termination of his job.
ii) Mr. Shayam owns a truck for providing transportation services. He
is paying annual tax as per Income Tax Act, 2058. During Ashwin
2075, he has not issued any invoice but has earned transportation
services fee amounting to Rs. 2 lakh.
iii) Kathmandu Metropolitan City (KMC) has an agreement with
Kalimati Users Committee for drain construction; the agreement
was made on 30 Baishakh, 2075. KMC paid Rs. 70 lakh to the
Committee on 10 Ashwin, 2075 after completion of the works.
iv) An NGO having tax exempt certificate, has booked income from
house rent Rs. 500,000 during the year 2074/75.
v) A Pvt. Ltd. has booked as income from sale of Car Rs. 300,000 on
Ashwin, 2075. The Company is registered with the objectives of
providing transportation services.

Solution:
a) As per Section 2 (Ka Nga) of IT, Act 2058, a company registered in Nepal is
always resident in Nepal. However, for companies outside Nepal whose
management is effective from Nepal during the income year are also resident.
Board meeting is the supreme body of the company, where major decisions
related to the companies are taken. The control and management of a unit is
usually situated at a place where the directing powers are situated.
Hence for FY 2075-76 the Bhutanese Company is resident in Nepal and its
worldwide income i.e. Rs. 70,000,000 is taxable income as per the provision of
IT Act, 2058.

The Institute of Chartered Accountants of Nepal 115


Suggested Answer- December 2018

b) It is a case of intangible asset. It's tax rate is to be computed based on straight-


line formula as:
Depreciation per Year: =1/Period of the contract near to half year
Rate of depreciation = (1/3)*100= 33.33%
Computation of depreciation base and depreciation:

Amount Rs.

Pool E Year 1 Year 2 Year 3


Purchase price 1,200,000
Opening Depreciation 0 933,333 533,333
base
Addition absorbed 800,000 0 0
Depreciation Base 800,000 933,3333 533,333
Terminal depreciation 0 0 0
Rate of depreciation 33.33% 33.33% 33.33%
Depreciation Expenses 266,667 400,000 400,000
Opening Depreciation 12,00,000 12,00,000 12,00,000
for the next year
Expiry of terms of an asset is disposal under Section 40(1) of Income Tax Act,
2058. Hence, the remaining balance of Rs. 12,00,000 will be treated as
depreciation of 4th year as the intangible asset will be use in fourth year as
well.
c) As per Section 27, the value of consideration shall be as follows:
i) Market value of TV i.e. Rs. 25,000 at the time of delivery is the
value of consideration paid.

ii) 1% per annum of value of motor vehicle i.e. Rs. 150,000 is the
value of consideration received by Mr. Ganpath.

iii) Security personnel fee Rs. 240,000 paid by the company shall be
the value of consideration received by Mr. Mohan.

iv) Value of free accommodation is 25% of rent paid i.e. Rs. 125,000
for the public director.
v) Value of free accommodation is 25 % of market rent i.e. Rs. 25,000
shall be the value of consideration for local leader.
d)
i) If any payment was made from non-contributory fund, the tax should be
deducted @ 15 % under Section 88(1) on the payment amount which is
final-withholding. The Fund should deduct Rs. 450,000 (Rs.30 lakh*15%)
from the amount and make payment amounting Rs. 26.50 lakh to him.
ii) Since Income Year 2075/76, the withholding tax on payment of rent in
relation to transport service shall be 2.5%. In the case of Shayam, he can
receive after deducting 2.5 % from the registered companies.

The Institute of Chartered Accountants of Nepal 116


Suggested Answer- December 2018

Alternatively, if the student shows the acquaintance with provision of Sec.


1 (13) of Schedule 1 of the Act, marks shall be awarded. The vehicle
owner needs to pay tax of Rs. 8,000 for the year.
iii) Since Income Year 2075/76, payment exceeding Rs. 5 Million where the
payment is related to work to be conducted through Users Committee,
there shall be tax withholding @ 1.5%. The agreement was concluded on
Income Year 2074/75, however, the provision is applicable for this
payment.
iv) In case of tax exempt entity, the following incomes shall be exempted as
per the Section 10(Chha) of income Tax Act, 2058:
 Donation, gift,
 Other contributions directly related with an organization entitled to
exemption as referred to in Clause (d) of Section 2 without having
consideration or without hoping for such contribution.
The rent income is not exempted by virtue of this provision, so, the NGO
should pay the tax on such rent income with filing the tax return.
v) If, in disposing the depreciable property of the business made by that
person, the incomings to be received exceed the remaining value
comprising the outgoings made for the property of the group of
depreciable property pursuant to Section 4(2) of Schedule 2, the excess
amount shall be considered to have been derived from the disposal of
depreciable property of the business, then included in the income as per
Section 7 of Income Tax Act, 2058. So, tax implications can be
determined on the basis of the value of the vehicles (Block C). It may
impact on depreciation expenses or income of the company. The objective
is not relevant in this case.
12. Write short notes: (4×2.5=10)
a) Approved medical expenses
b) Deductible research and development expenses
c) Tax on agricultural income
d) Withholding agent
Solution:
a) The following medical treatment expenses shall be deemed as the approved
medical treatment expenses:
 The amount as per the bill including the expenses for medicines incurred in
doing treatment of any natural person by a recognized hospital, nursing
home, health centre or a doctor.
However, the following costs should not be treated as approved medical cost:
 Costs incurred for cosmetic surgery
 if costs incurred in filing prescriptions of diagnosis, or payments for filing
prescription bills including the purchase of medicine as described above
are reimbursed by an insurance company under a health insurance premium
paid or claim as per Schedule 1(1)(16), they cannot be claimed as a
medical tax credit because the taxpayer does not directly pay them.

b) Section 18 has made provision for eligible expenses of research and


development expenses, it is explained "research and development expenses"
means the expenses incurred by any person for the purpose of controlling

The Institute of Chartered Accountants of Nepal 117


Suggested Answer- December 2018

developing his business and improving commercial production and process.


Provided that, such expenses shall not include the cost at the time of acquiring
any property referred to in Sub-section (3) of Section 1of Schedule-2.
As per the provision, research and development expenses may be deducted to
the extent incurred in the operation of that business in that year, the deductible
limit shall not be exceed 50 % of the adjusted taxable income of that person
computed without deducting research and development expenses of all
businesses operated by that person.
Any excess expense or part thereof which is not deductible in excess of the
limit may be capitalized and depreciated pursuant to Schedule-2.

c) Agricultural income derived by a natural person from farming within the


prescribed limit under Land Related Act, 2021 is exempt from tax. But the
same income is taxed if earned by a partnership firm or Company.

d) As per Section 2 (Ka) of Income Tax Act, 2058, a withholding agent means a
person who is required to withhold tax according to Chapter 17 of the Act
while making payment in consideration of employment, investment returns,
service fees, and contract payments.

13. Not Only For Profit Pvt. Ltd., Kathmandu imports luxury cars and track-
laying tractors and sells in Nepal. Following details pertain to Ashwin,
2075.
a) The showroom price of luxury car is Rs. 5 million. Two percent
Dashain discount was offered on sale of the cars. 10 cars were sold to
VAT registered customers and 6 cars were sold to individual customers
not registered in VAT.
b) The showroom price of Track-laying tractors is Rs. 2.5 million. Two
percent Dashain discount was offered on the tractors. 10 tractors were
sold to individual customers who were not registered for VAT.
c) The company offers additional 5% discount for prompt payment. 5
customers purchasing luxury car and 6 customers purchasing tractor
made prompt payment and benefitted from the discount offer.
d) Import price of luxury cars was Indian Rupees (INR) 2 million per car.
Total transportation expenses upto custom point was INR 200,000
(paid to Indian transporter) and total insurance premium Rs. 200,000
was paid to an insurance company in Nepal. 16 cars were imported in
the same month and sold as above.
e) Import price of per unit Track-laying tractors was INR 1.5 million.
Total transportation expenses up to custom point was INR 100,000
(paid to Indian transporter) and total insurance premium Rs. 50,000
was paid to an insurance company in Nepal. 10 tractors were imported
in the same month and sold as above.
f) During the month, Rs. 90,000 was spent on mobile telephone calls, of
which 40% relates to private calls.
g) On 30 Ashwin 2075, office equipment was purchased for Rs.
1,000,000.

The Institute of Chartered Accountants of Nepal 118


Suggested Answer- December 2018

h) Custom duty was Rs. 41,376,000 for the luxury cars in total.
i) The company also constructed a showroom in the month of Ashwin,
2075. The total cost of construction was Rs. 6 million, which was
constructed by a VAT unregistered builder.
j) All of the figures are exclusive of VAT and use exchange rate @ Rs.
1.6015 per INR.
Calculate the amount of VAT payable/receivable by the company for
Ashwin 2075. 10
Solution:
Calculation of VAT payable/receivable for the month ended Bhadra 2075
Particulars Amount Rs.
Output tax (WN 1) 10,192,000
Less: Input tax for imported cars (WN 2) (12,108,759)
Input tax credit on expenses (WN 3) (130,396)
Net VAT receivable (2,047,155)
Reverse charge (VAT) on construction (WN 4- to be
paid) 780,000

WN 1: Calculation of output tax


Tractors sale VAT
Particulars Cars sale Rs. Rs. amount Rs.
Showroom price 5,000,000 2,500,000
Dashain Discount (trade discount) 100,000 50,000
Sales price 4,900,000 2,450,000
Sales to VAT registered Customers (10 cars) 49,000,000 0.00 6,370,000
Sales to Unregistered Customers (6 cars&10 tractors) 29,400,000 24,500,000 3,822,000
Total 78,400,000 24,500,000 10,192,000
Proportionate sales ratio 76.19% 23.81%
* Track-laying Tractor is VAT exempt as per schedule 1 of VAT Act, 2052.

WN 2: Calculation of VAT on Import at custom point for Luxury cars:


Particulars Amount
Qty
Import price (16*2,000,000*1.6015) 51,248,000

Transportation expenses IRs. 200,000 * 1.6015 320,300


Insurance premium 200,000
custom duty 41,376,000
Total taxable value for VAT 93,144,300
VAT paid at custom point 12,108,759
* Track-laying tractors are VAT exempt items so no need to calculate input VAT on import

WN 3: Calculation of input tax credit on expenses:

The Institute of Chartered Accountants of Nepal 119


Suggested Answer- December 2018

Amount
Particulars Rs.
Full tax credit:
Insurance for Car Rs. 200,000 * 13% 26,000
Proportionate tax credit:
Rs. 90,000*13%*
Mobile expenses 60%* 76.19% 5,349
Rs. 1,000,000 *
Office Equipment 13%*76.19% 99,047
Total 130,396

WN 4: Reverse VAT on construction of structures for commercial use through VAT


Unregistered builders.
As per section 8(3), if a building, apartment or similar structures are constructed for
commercial purposes, from VAT-unregistered builders, then VAT should be deposited
as if it was constructed from VAT-registered person.
Total cost of construction 6,000,000.00
VAT 780,000.00

14.
a) Banmali Traders has following sales & purchase transactions exclusive
of VAT for the following months of 2075:
Month Sales (Rs.) Purchase (Rs.) Export % of Total Sales
Baishakh 500,000 600,000 43%
Jeshtha 825,000 785,000 65%
Ashadh 680,000 675,000 38%
Shrawan 450,000 500,000 40%
Bhadra 300,000 275,000 55%
Ashwin 400,000 500,000 45%
Kartik 600,000 490,000 25%

How much refund can Banmali Traders claim for? 5

b) Manaslu Enterprises, Kathmandu is trading in clothes, and has the


following transactions in the last twelve months from the end of
Bhadra, 2075. The Enterprises is not registered in VAT.
i) Export sales Rs. 20 lakh (Kartik 2074 to Magh 2074)
ii) Sales in foreign currency from the Shop: equivalent Rs. 10 lakh
(Bhadra 2075)
iii) Sales in local currency Rs. 25 lakh (Ashwin 2074 to Bhadra 2075)
iv) Purchase Rs. 40 lakh
v) Loan from bank Rs. 15 lakh
vi) Loan payment Rs. 15 lakh
vii) Salary and other expenses Rs. 10 lakh
viii) Rent expenses Rs. 2 lakh

The Institute of Chartered Accountants of Nepal 120


Suggested Answer- December 2018

ix) Telephone expenses Rs. 1 lakh


Mention the VAT implication and registration in the above transaction. 5

Solution:
a)
Amt. Rs.
Period VAT on VAT on Debit/(Credit)
2075 Purchase Local Sales Export Purchase Sales
Baishakh 600,000 285,000 215,000 78,000 37,050 (40,950)
Jeshtha 785,000 288,750 536,250 102,050 37,537.50 (64,512.50)
Ashadh 675,000 421,600 258,400 87,750 54,808 (32,942)
Shrawan 500,000 270,000 180,000 65,000 35,100 (29,900)
Bhadra 275,000 135,000 165,000 35,750 17,550 (18,200)
Ashwin 500,000 220,000 180,000 65,000 28,600 (36,400)
Kartik 490,000 450,000 150,000 63,700 58,500 (5,200)
Since, Banmali Traders' export sales in the month of Baishakh, Jeshtha,
Shrawan, Bhadra and Aswin exceeds 40% of total sales, it may file an
application for refund while filing the VAT return of respective month.

The excess input VAT over output VAT for the month of Ashad and Kartik is
carried forward for set off in next six months. If the same cannot be consumed
by any payable VAT in succeeding six months, it may file an application for
refund after the elapse of such six months.

It is assumed that all the purchase is qualifying for credit.

b) Conditions for compulsory registration:


 In case the turnover exceeds Rs. 50 lakhs where dealing in taxable goods,
and Rs. 20 lakhs while dealing in taxable services or both taxable goods
and taxable services
 In case of specific transactions all over Nepal, regardless of turnover
 In case of specific transactions in specific area, regardless of turnover
 In case a person borrows the loan more than Rs. 10 lakh for business
purpose, the person shall compulsorily register for VAT (Rule 7 (5) of
VAT Rules, 2053)
 In case of stock exceeding limit prescribed by IRD at the time of inspection
by tax officer
The enterprise has borrowed the loan Rs. 15 lakh during the year. So, it is
required to register in VAT.
As per Rule 6, exemption from registration is given to small vendors having
five millions or less taxable transaction of goods during the previous twelve
months. If the turnover of transaction carried on by any person exceeds Rs.
five million, the person carrying on such transaction has to make an application
in the format referred to in Schedule-1 to the concerned Tax Officer to get the
transaction registered within Thirty days of the date of such excess. So, it has
to register within 30 days from Ashwin 30, 2075. The enterprise has total sales
of Rs. 55 lakh (20+10+25) including export sales. The excess sales than

The Institute of Chartered Accountants of Nepal 121


Suggested Answer- December 2018

prescribed amount Rs. 50 lakh are related to local sales as per the given
information.
According to Section 15, a person who is not registered shall not issue an
invoice or other document showing the collection of tax and shall not recover
the tax. By these provision, the enterprise cannot collect vat amount on sales
above Rs.5,000,000 before registration in VAT.

15. Write short notes on the basis of VAT Act, 2052: (4×2.5=10)
a) Market Value
b) Collection of tax from other than registered person
c) Zero rated and VAT exempted goods/services
d) Valuation at market price by tax officer
Solution:
a) As per Section 13 of the Value Added Tax Act, 2052, Market value of goods
and services supplied is determined as the consideration between unrelated
parties at arms-length transaction for the supply of goods or services would be
generally be agreed on if the transaction were made on similar circumstances.
b) As per Section 15 of the VAT Act. Federal Government offices, local
government offices, public enterprises, and Nepal based international NGOs
should collect VAT on disposal of any VAT attractive items despite they are
unregistered for VAT.
c) Zero rated goods/services are within the scope of VAT, Zero rated goods &
services are listed in Schedule-II, person dealings with zero rated
goods/services shall get registered, required to issue invoice as per the
requirements of VAT Act and normally related to exports. Whereas VAT
exempted goods/services our outside the scope of VAT Act, such goods and
services are listed in schedule-I, no registration is required for person
exclusively dealing with VAT exempted goods/services. Exempted goods and
services basically include basic goods & services.
d) In the following transactions tax officer may be reassessed the value of goods
or services at market value:
i) If tax is not levied on market value by tax payers themselves as per under
section 20 and Rule 29.
ii) In case of under invoicing as per Section 12(6), 20 and Rule 29.
iii) In case the stocks appearing in the books cannot be shown physically at
the time of inspection by tax officer Rule 40.

The Institute of Chartered Accountants of Nepal 122


Suggested Answer- December 2018

Specific Comments on the performance of the students


Batch: - December 2018
Level: - CAP-II
Subject: Income Tax & VAT
Question No. 1
Majority of students attempted the question, however very few students get half of the
marks allotted for the question. Majority of students secured marks in between 5 to 8.
Students need to understand the concepts of deductions relating to provision for CSR,
R&M expenses etc. Only few students were able to answer near correctly. Most
students even not able to compute depreciation correctly by applying correct rate.
Students are also unable to understand that the entity is special industry inadequate.
Question No. 2
Majority of students attempted the question and secured average marks and very less
students secured 5 to 7 marks. Not well prepared by students to compute TDS of
employee as given in the question. Students need to understand that for TDS annual
income shall compute first.
Question No. 3
Majority of students do not attempted the question with disclosing the provision of IT
Act, general attempt by students and secured very less marks.
Question No. 4
Majority of students attempted the question but unable to secure good marks. Only
very less student secured average marks. Majority of students secured very less
marks. Students have Lack of understanding of knowledge about residential status,
withholding tax and recent amendment. Students were more confused in the concepts
of Residential status of company and its taxability.
Question No. 5
Majority of students attempted the question but unable to answer the short notes in
line with the provision of IT Act and secured average marks. Students need to
elaborate in theory questions.
Question No. 6
Majority of students try to attempt the question one/two students try to solve the
question but majority students confused to answer the question and secured very less
marks. Students showed poor performance due to lack of knowledge about
proportionate VAT credit. Lack of knowledge of proportionate credit was verdict.
Question No. 7
Majority of students attempted the question and unable to answer as per the spirit of
question and provision of VAT Act. They secured average marks based on step
markings. Students attempt is good but need to response on specific matter asked by
question.
Question No. 8
Majority of students attempted the question and very less students able to secure good
marks. Majority of students gives general answer that is without disclosing the
provision of Act. Students need to be elaborate in theory questions.

The Institute of Chartered Accountants of Nepal 123

You might also like